You are on page 1of 161

GENERAL STUDIES 1 - 50 PAGES

1. ANCIENT INDIA
1.

2.

3.

4.

5.

1995
Who among the following was.a Brahmavadini who composed some hymns of
the Vedas?
(a) Lopamudra
(b) Gargi
(c) Leelavati
(d) Savitri
Ans. (a)
Explanation:
In vedic period, women called Brahmavadini composed many hymns of
Rigveda. Lopamudra, Ghosa, Nivavari and Vishwaware were prominent among
them.
Match List I with List II and select the correct answer using the codes given
below the lists:
List I (Eras)
List II (Reckoned from)
A. Vikrama era
1. 3102 B.C.
B. Saka era
2. 320 A.D.
C. Gupta era
3. 78 A.D.
D. Kali era
4. 58 B.C.
5. 248 A.D.
A B C D
A B C D
(a) 2 4 5 1
(b) 1 3 2 4
(c) 4 5 2 3
(d) 4 3 2 1
Ans. (d)
Explanation:
King Vikramaditya started Vikram era in 58 B.C. to mark his victory over
Shakas. A.D. 78 is regarded as Saka era and it is most probably the date of
Kanishka's accession. Chandragupta-I was the first independent king in Guptas
with the title Maharajadhiraja and he started the Gupta era in 319-20 A.D. Kali
era started in 3102 B.C.
In Sanskrit plays written during the Gupta period women and sudras speak
(a) Sanskrit
(b) Prakrit
(c) Pali
(d) Sauraseni
Ans. (b)
Explanation:
The position of women and Sudras was reflected in the Sanskrit plays written in
the Gupta period. They depicted as the males of upper three varnas spoke
Sanskrit whereas the lower varna-sudra and women of all Varnas spoke in
Prakrit.
The name by which Ashoka is generally referred to his inscriptions is
(a) Chakravarti
(b) Dharmadeva
(c) Dharmakirti
(d) Priyadarsi
Ans. (d)
Explanation:
In his inscriptions, the king Ashoka is generally referred to as Priyadarshi
(Piyapins) and Devanampriya (Qeva nom piya).
Who among the following anticipated Newton by declaring that all things
gravitate to the earth ?
(a) Aryabhatta
(b) Varahamihira
(c) Buddhagupta
(d)
Brahmagupta
Ans. (d)
Explanation:

Brahmagupta in his books Brahmasphuta Siddhanta and Khanda Khadyaka,


postulated that all things gravitate to the earth, long before than Newton.
6.
According to Mimamsa system of philosophy liberation is possible by means of
(a) Jnana
(b) bhakti
(c) yoga
(d) karma
Ans. (d)
Explanation:
Mimamsa System of Philosophy recognises the Vedas as the final authority. It
stresses on
Karma Kanda (ritualism) for the liberation Along with that
pursuit of Vedic knowledge is also important to perform Karmas wholistically.
7.
The term Yavanapriya5 mentioned in ancient Sanskrit texts denoted
(a) a fine variety of Indian muslim
(b) ivory
(c) damsels sent to the Greek court for dance performance
(d) pepper
Ans. (d)
Explanation:
Pepper which was grown in India and surrounding parts (S.Asia and S.E. Asia)
had a lot of demand in Ancient Roman Empire. By exporting this commodity
India earned a lot of gold from Rome, that is why in Sanskrit texts Pepper was
called "yavana priya" (the favourite of Greeks).
8.
The concept of Arruvrata was advocated by
(a) Mahayana Buddhism
(b) Hinayana Buddhism
(c) Jainism
(d) the Lokayukta school
Ans. (c)
Explanation:
The 5 cardinal principles of Jainism, if followed by a monk, is called Mahavrata
and if observed by a lay follower, then it was called Anuvrata.
9.
Zero was invented by
(a) Aryabhatta
(b) Varahamihira
(c) Bhaskara I
(d) an unknown
Indian
Ans. (d)
Explanation:
Zero was invented by an unknown Indian. The zero can be traced to the Indian
Mathematician Aryabhatta who, about 520 A.D., deviced a positional decimal
number system that contained word "Kha" for the idea of a placeholder. By 876
A.D., based on an existing tablet inscription with that date, the 'Kha' had
become the symbol "0". Later "Brahmagupta" developed the concept of the
zero as an actual independent number.
1996
10. According to ancient Indian cosmogonic ideas the sequential order of the cycle
of four aeons (yugas) is
(a) Dvapara, Krita, Treta and Kali
(b) Krita, Dvapara, Treta and Kali
(c) Krita, Treta, Dvapara and Kali
(d) Treta, Dvapara, Kali and Krita
Ans. (c)
Explanation:
Krita (4 times of Kaliyuga), Treta (3 times of Kaliyuga), Dvapara (twice of
Kaliyuga), Kaliyuga, is the sequential order according to Ancient Indian
Cosmogonic ideas.
11. The river most mentioned in early Vedic literature is
(a) Sindhu
(b) Sutudri
(c) Saraswati
(d) Ganga
Ans. (a)
Explanation:

12.

13.

14.

15.

16.

Sindhu is the most mentioned river, where as Saraswati is considered to be the


most important and sacred.
Which one of the following is not a part of early Jaina literature ?
(a) Therigatha
(b) Acarangasutra (c) Sutrakritanga
(d)
Brihatkalpasutra
Ans. (a)
Explanation:
Among these Therigatha is a part of Buddhist literature, and was compiled by
Buddhist nuns, with the poems on Lord Buddha.
Which of the following were common to both Buddhism and Jainism ?
1. Avoidance of extremities of penance and enjoyment
2. Indifference to the authority of the vedas
3. Denial of efficacy of rituals
4. Non-injury to animal life
Select the correct answer using the codes given below :
(a) 1, 2, 3 and 4
(b) 2, 3 and 4
(c) 1, 3 and 4
(d) 1 and 2
Ans. (b)
Explanation:
In Buddhism, Madhyam marga was stressed on. So the followers were
advocated to choose a middle path. So avoidance of extremities was stressed
on in Buddhism, whereas in Jainism, extremism and penance were advocated.
Who among the following is known for his work on medicine during the Gupta
period ?
(a) Saumilla
(b) Sudraka
(c) Shaunaka
(d) Susrutha
Ans. (d)
Explanation:
Susrutha was considered as the best surgeon of ancient India during Gupta
rule. He wrote "Susruta Samhita" - a treatise on medicine. Susrutha belongs to
4th century A.D.
Match List I with List II and select the correct answer using the codes given
below the lists:
List I
List II
A. Visakhadatta
1. Medicine
B. Varahamihira
2. Drama
C. Charaka
3. Astronomy
D. Brahmagupta
4. Mathematics
A B C D
A B C D
(a) 1 3 4 2
(b) 2 1 3 4
(c) 2 3 1 4
(d) 3 4 1 2
Ans. (c)
Explanation:
Vishakhadatta wrote a great Sanskrit play "Mudra Rakshasam". Varahamihira
was a famous astronomer. Charaka wrote Charakasamhita and contemporary
of king Kanishka. Brahmagupta was a mathematician who propounded that all
things gravitate to the earth.
1997
Match List I with List II and select the correct answer using the codes given
below the lists:
List I
List II
A. Gupta
1. Badami
B. Chandella
2. Panamalai

17.

18.

19.

20.

21.

22.

C. Chalukya
3. Khajuraho
D. Pallava
4. Deogarh
A B C D
A B C D
(a) 4 3 1 2
(b) 4 2 3 1
(c) 2 3 4 1
(d) 3 4 1 2
Ans. (a)
Explanation:
Gupta's capital was Deogarh, and Chandellas had their capital at Khajuraho,
Chalukya's capital was Badami (Vatapi) and Pallava's capital was Panamalai.
Which one of the following statements regarding Asokan stone pillars is
incorrect ?
(a) These are highly polished
(b) These are monolithic
(c) The shaft of pillars is tapering in shape (d) These are parts of architectural
structures
Ans. (d)
Explanation:
Asoka's rock pillars are not architectural structures, they were intended to
spread Ashoka's message to the people. They were erected with Asokan
edicts.
Which one of the following scripts of ancient India was written from right to left ?
(a) Brahmi
(b) Nandnagari
(c) Sharada
(d) Kharoshti
Ans. (d)
Explanation:
The Brahmi script was used mostly in India is written from left to right whereas,
Kharoshti script found in now Pakistan region is written from right to left, which
is evolved on the Varnamala system of Indian language.
The famous dialogue between Nachiketa and Yama is mentioned in the
(a) Chhandogyopanishad
(b) Mundakopanishad
(c) Kathopanishad
(d) Kenopanishad
Ans. (c)
Explanation:
The famous dialogue between Nachiketa and Yama (Lord of death) about
death and transmigration of Soul is mentioned in Kathopanishad.
Milindapanho is in the form of a dialogue between the king Menander and the
Buddhist monk
(a) Nagasena
(b) Nagarjuna
(c) Nagabhatta
(d)
Kumarilabhatta
Ans. (a)
Explanation:
The dialogue between the king Menander (Milinda) and the Buddhist monk
Nagasena was compiled in the form of Milindapanho in 1 century B.C.
Which one of the following edicts mentions the personal name of Asoka ?
(a) Kalsi
(b) Rummindei
(c) Special Kaling Edict (d) Maski
Ans. (d)
Explanation:
A The personal name of Ashoka is mentioned in Maski and Gurjara edicts.
The following map shows four of the sixteen mahajanapadas that existed in
ancient India :
The places marked A, B, C and D respectively are

23.

24.

25.

26.

27.

(a) Matsya, Cedi, Kosala, Anga


(b) Surasena,
Avanti,
Vatsa,
Magadha
(c) Matsya, Avanti, Vatsa, Anga
(d) Surasena,
Cedi,
Kosala,
Magadha
Ans. (c)
Explanation:
The given order is Matsya-Avanti-Vatsa-Anga. According to 16 Mahajanapadas
mentioned in Anguttaranikaya.
In the Mahayana Buddhism, the Bodhisatva Avalokitesvar was also known as
(a) Vajrapani
(b) Manjusri
(c) Padmapani
(d) Maitreya
Ans. (c)
Explanation:
In Mahayana Bodhisatva Avalokiteshvara is known as "padmapani". He was
considered as the God of Sex.
The silver coins issued by the Guptas were called
(a) rupaka
(b) karshapana
(c) dinar a
(d) pana
Ans. (a)
Explanation:
Gupta rulers issued Silver coins known as "rupaka". These coins have been
found in most of the northern parts of the country and maximum number in
Bharatpur of Rajasthan.
Which one of the following was a corporation of merchants in ancient India?
(a) Chaturvedimangalam(b) Parishad
(c) Ashtadikgaja
(d) Manigrama
Ans. (d)
Explanation:
Manigrama was a corporation of merchants in Ancient South India.
1998
Directions : The following items consists of two statements, one labelled as
Assertion (A) and the other labelled as Reason (R). You are to examine these
two statements carefully and decide if the Assertion (A) and the Reason (R) are
individually true and if so, whether the Reason R is a correct Explanation: of
Assertion. Select your answer to these items using the codes given below and
mark your answer sheet accordingly.
(a) Both A and R are true and R is the correct Explanation: of A
(b) Both A and R are true but R is not the correct Explanation: of A
(c) A is true but R is false
(d)
A is false but R
is'true
Assertion A: The Gandhara School of Art bears the mark of Hellenistic
influence.
Reason R : Hinayana form was influenced by that art.
Ans. (c)
Explanation:
Assertion is correct as the Gandhara School of Art was influenced by Greeks.
But, it was Mahayana form of Buddhism that was influenced by that art.
Assertion A : According to Asoka's edicts social harmony among the people
was more important than religious devotion.
Reason R : He spread ideas of equity instead of promotion of religion.
Ans. (a)
Explanation:

28.

29.

30.

31.

Ashoka spread ideas of equity instead of religion, that is why, his edicts stress
on social harmony among the people rather than religious devotion.
Which of the following pairs are correctly matched ?
1. Lothal
: Ancient dockyard
2. Sarnath
: First Sermon of Buddha
3. Rajgir
: Lion capital of Ashoka
4. Nalanda
: Great seat of Buddhist learning
Select the correct answer using the codes given:
(a) 1, 2, 3 and 4
(b) 3 and 4
(c) 1, 2 and 4
(d) 1 and 2
Ans.
(c)
Explanation:
Sarnath is the place where Buddha gave his first sermon and also lion capital of
Ashoka is existed. Rajgir is famous pilgrimage centre and is famous for its hot
water springs.
Which one of the following ancient Indian records is the earliest royal order to
preserve food-grains to be utilised during the crises in the country ?
(a) Sohagaura Gopper-plate
(b) Rummindei pillar-edict of Asoka
(c) Prayaga-Prasasti
(d) Mehrauli Pillar inscription of
Chandra
Ans. (a)
Explanation:
The earliest royal order to preserve food-grains to be utilised during the crisis in
the country is Sohagaura copper-plate inscription of Ashoka.
The concept of Eight-fold path forms the theme of
(a) Dipavamsa
(b) Divyavadana
(c) Mahaparinibban Sutta
(d) Dharma Chakra Pravartana Sutta
Ans. (d)
Explanation:
The concept of eight-fold path forms the theme of 'Dharma Chakra pravartana
sutta' which means sermon of the turning of the wheel of law. This gives 'Noble
Eight fold path' and also 'Four Noble Truths' which were accepted as basic
categories of all Buddhist sects.
The given map relates to :

(a) Kanishka at the time of his death


(b) Samudragupta after the close of his South Indian campaign
(c) Ashoka tDwards close of his reign
(d) Empire of Thaneswar on the eve of Harsha's accession
Ans. (c)
Explanation:
The given map relates to the Ashoka's empire towards close of his reign.
32. Many of the Greeks, Kushanas and Shakas embraced Buddhism rather than
Hinduism because
(a) Buddhism was in the ascendant at that time
(b) they had renounced the policy of war and violence
(c) caste-ridden Hinduism did not attract them
(d) Buddhism provided easier access to Indian society
Ans. (d)
Explanation:

33.

34.

35.

36.

37.

38.

Greeks, Kushanas and Shakas embraced Buddhism rather than Hinduism


because Buddhism provided easier access to Indian society.
The Ashokan major rock edicts which tell us about the Sangam Kingdom
include rock edicts
(a) I and X
(b) I and XI
(c) II and XIII
(d) II and XIV
Ans. (c)
Explanation:
Ashokan major rock edicts II and XIII tells us about the Sangam Kingdom.
I, II - Gave account of Ashoka's concern for the comforts of men and beasts.
X, XI - talk about Dharma.
XIV - is a post script to 13 rock edicts.
What is the correct chronological order in which the following appeared in India
?
1. Gold coins.
2. Punch-marked silver coins.
3. Iron plough.
4. Urban culture.
Select the correct answer using the codes given:
(a) 3, 4, 1, 2
(b) 3, 4, 2, 1
(c) 4, 3, 1, 2
(d) 4, 3, 2, 1
Ans. (b)
Explanation:
Iron plough led to increased agricultural production which helps in flourished
trade and growth of urban culture. Then the punch-marked silver coins have
appeared which were succeeded by Gold coins minted by Indo-Greek rulers.
1999
The term 'Aryan denotes
(a) an ethnic group (b) a nomadic people (c) a speech group (d) a superior
race
Ans. (c)
Explanation:
'Aryans' a speech group called after their language, which spread from Central
Asia to other places.
Which one of the following ports handled the north Indian trade during the
Gupta period ?
(a) Tamralipti
(b) Broach
(c) Kalyan
(d) Cambay
Ans. (a)
Explanation:
Tamralipti port in Bengal handled the north-Indian trade with SE Asian
countries during the Gupta period.
The following persons came to India at one time or another
1. Fa-Hien
2. I-Tsing
3. Megasthenese
4.
Hieun-Tsang
The correct chronological sequence of their visits is :
(a) 3, 1, 2, 4
(b) 3, 1, 4, 2
(c) 1, 3, 2, 4
(d) 1, 3, 4, 2
Ans. (b)
Explanation:
Megasthenese - Mauryan period (4th century B.C.)
Fa-Hien - Gupta period (5th century A.D.)
Hieun-Tsang - Post-Gupta period (7th Century A.D.)
I-Tsing - After Harsha (8th century A.D.)
Which one of the following was initially the most powerful city state of India in
the 6th century B.C.?

39.

40.

41.

42.

(a) Gandhar
(b) Kamboj
(c) Kashi
(d) Magadh
Ans. (c)
Explanation:
Initially Kashi was most powerful. In course of time, Kosala grew and finally
Magadha dominated the scene under Bindusara and Ajatasatru.
The Indo-Greek kingdom set up in north Afghanistan in the beginning of the
second century B.C. was
(a) Bactria
(b) Scythia
(c) Zedrasia
(d) Aria
Ans. (a)
Explanation:
The Indo-Greek kingdom set up in north Afghanistan in the beginning of the
second century B.C. was Bactria. Hence, Indo-Greeks were also called
Bactrians.
From the third century AD when the Hun invasion ended the Roman Empire,
the Indian merchants relied more and more on the
(a) African trade
(b) West-European trade
(c) South-East Asian trade
(d) Middle-Eastern trade
Ans. (c)
Explanation:
Hun invasion ended the Roman empire. It forced the Indian merchants to look
for developed markets in the eastern areas. So South-East Asian trade availed
them of opportunities.
2000
Consider the following statements regarding the Chakiarkoothu form of dance:
1. It is performed by Chakiar caste.
2. It cannot be traditionally witnessed by the higher caste Hindus.
3. Mizhavu is the accompanying instrument.
4. Its theatre form is called koothambalam.
Which of these statements are correct ?
(a) 1, 3 and 4
(b) 1, 2 and 3
(c) 2, 3 and 4
(d) 1, 2 and 4
Ans. (d)
Explanation:
Chakiarkoothu is a famous folk dance of Kerala performed by Chakiar caste.
Mizhavu is not associated with this dance.
Directions : The following items consists of two statements, one labelled as
Assertion (A) and the other labelled as Reason (R). You are to examine these
two statements carefully and decide if the Assertion (A) and the Reason (R) are
individually true and if so, whether the Reason R is a correct Explanation of
Assertion. Select your answer to these items using the codes given below and
mark your answer sheet accordingly.
(a) Both A and R are true and R is the correct Explanation of A
(b) Both A and R are true but R is not the correct Explanation of A
(c) A is true but R is false
(d)
A is false but R
is true
Assertion A : The origin of Feudal system in Ancient India can be traced to
Military campaigns.
Reason R : There was considerable expansion of the Feudal system during the
Gupta period.
Ans. (d)
Explanation:

43.

44.

45.

46.

47.

48.

49.

The origin of feudal system can be traced back to landgrants given by


Satavahanas. The feudal system considerably expanded during the Gupta
period.
Assertion A : Ashoka annexed Kalinga to the Mauryan Empire.
Reason R : Kalinga controlled land and sea routes to South India.
Ans. (a)
Explanation:
Kalinga controlled land and sea routes to South India hence it was annexed to
Mauryan empire by Ashoka.
Assertion (A) : The emphasis of Jainism on nonviolence (ahimsa) prevented
agriculturalists from embracing Jainism.
Reason (R) : Cultivation involved killing of insects and pests.
Ans. (a)
Explanation:
Agriculturists were prevented from embracing Jainism as cultivation involves
killing of insects and pests which is against the prinifple of non-violence
(ahimsa) of Jainism.
The practice of military governorship was first introduced in India by the
(a) Greeks
(b) Shakas
(c) Parthians
(d) Mughals
Ans. (a)
Explanation:
The practice of military governorship was first introduced in India by Greeks.
Alexander in his return journey from India, distributed his possessions among
his military generals. The military governors are called 'strategos'.
Which one of the following dynasties was ruling over North India at the time of
Alexander's invasion ?
(a) Nanda
(b) Maurya
(c) Sunga
(d) Kanva
Ans. (a)
Explanation:
Nanda dynasty was ruling over North India at the time of Alexander's invasion.
Nanda's were later deposed by Chandragupta Maurya with the help of Kautilya.
2001
Who among the following presided over the Buddhist Council held during the
reign of Kanishka at Kashmir ?
(a) Parsva
(b) Nagarjuna
(c) Sudraka
(d) Vasumitra
Ans. (d)
Explanation:
1st Council (Rajagriha) - Mahakassapa (Ajatashatru)
2nd Council (Vaisali) - Sabakami (Kalashoka)
3rd Council (Pataliputra) - Mogaliputta Tissa (Ashoka)
4th Council (Kashmir) - Vasumitra during the reign of Kanishka.
Which one of the following animals was NOT represented on the seals and
terracotta art of the Harappan culture ?
(a) Cow
(b) Elephant
(c) Rhinoceros
(d) Tiger
Ans. (a)
Explanation:
Cow was not represented on the seals and terracotta art of the Harappan
Culture,
Which one of the following pairs is correctly matched ?
(a) Harappan Civilization :Painted Grey Ware

(b) The Kushans


: Gandhara School of Art
(c) The Mughals
: Ajantha Paintings
(d) The Marathas
: Pahari School of Painting
Ans. (b)
Explanation:
Gandhara School of Art flourished during Kushan period. Harappan Civilization
- Black on red pottery.
Vedic Period - Painted Grey ware.
Guptas - Ajanfa paintings.
Mughals - Pahari school of painting.
Directions : The following items consists of two statements, one labelled as
Assertion (A) and the other labelled as Reason (R). You are to examine these
two statements carefully and decide if the Assertion (A) and the Reason (R) are
individually true and if so, whether the Reason R is a correct Explanation of
Assertion. Select your answer to these items using the codes given below and
mark your answer sheet accordingly.
(a) Both A and R are true and R is the correct Explanation of A
(b) Both A and R are true but R is not the correct Explanation of A
(c) A is true but R is false
(d)
A is false but R
is true
50. Assertion A : Harshavardhana convened the Prayag Assembly.
Reason R : He wanted to popularise only the Mahayana form of Buddhism.
Ans. (b)
Explanation:
.Harshavardhana convened the Prayag Assembly to popularise himself among
his people. To popularise Mahayana form of Buddhism at the instinct of HuenTsang, he called on a Conference at Kannauj.
2002
51. Match List I with List II and select the correct answer using the codes given
below the lists:
List I (Ancient site)
List
II
(Archaeological finding)
A. Lothal
1. Ploughed field
B. Kalibangan
2. Dockyard
C. Dholavira .
3. Terracotta replica of a plough
D. Banawali
4. An inscription comprising
ten large sized signs of the Harappan script
A B C D
A B C D
(a) 1 2 3 4
(b) 2 1 4 3
(c) 1 2 4 3
(d) 2 1 3 4
Ans. (b)
Explanation:
Ancient Site Archaeological Finding Lothal - Dockyard
Kalibangan
- Ploughed field
Dholavira - An inscription comprising ten large sized signs of the Harappan
script
Banawali - Terracotta replica of a plough.
52. In ancient Indian Buddhist monasteries, a ceremony called Pavarana used to
be held. It was the

10

53.

54.

55.

56.

(a) occasion to elect the Sanghaparinayaka and two speakers one on Dhamma
and the other on Vinaya
(b) confession by monks of their offences committed during their stay in the
monasteries during the rainy season
(c) ceremony of initiation of new person into the Buddhist Sangha in which the
head is shaved and when yellow robes are offered
(d) gathering of Buddhist monks on the next day to the ful 1-moon day of
Ashadha when they take up a fixed abode for the next four months of rainy
season
Ans. (b)
Explanation:
Confession by monks of their offences committed during their stay in the
monasteries during the rainy season.
The ancient Indian play Mudrarakshasa of Visakhadutt has its subject on
(a) a conflict between Gods and Demons of ancient Hindu lore
(b) a romantic story of of an Aryan prince and a tribal woman
(c) the story of the power struggle between two Aryan tribes
(d) the court intrigues at the time of Chandragupta Maurya
Ans. (d)
Explanation:
Mudrarakshasa of Visakhadutt deals with the court intrigues at the time of
Chandragupta Maurya.
2003
Emperor Harshas southward march was stopped on the Narmada river by
(a) Pulakesin-I
(b) Pulakesin-II
(c) Vikramaditya-I
(d)
Vikramadity-ll
Ans. (b)
Explanation:
Emperor Harsha's southward march was stopped on the Narmada river by
Chalukya King of Vatapi-Pulakesin-II.
Consider the following statements :
1. The Cholas defeated Pandya and Chera rulers and established their
domination over peninsular India in the early medieval times.
2. The Cholas sent an expedition against Sailendra empire of South East Asia
and conquered some of the areas.
Which of these statements is/are correct ?
(a) Only 1
(b) Only 2
(c) Both 1 and 2
(d) Neither 1
nor 2
Ans. (c)
Explanation:
Chola's established their domination in peninsular India by defeating Pandya
and Chera rulers. Naval expedition by Rajendra-I of Chola's against Sailendra
empire of South East Asia as Srivijaya rulers created difficulties for Indian
merchants.
Mrichchhakatika an ancient Indian book written by Shudraka deals with
(a) the love affair of a rich merchant with the daughter of a courtesan
(b) the victory of Chandragupta II over the Shaka Kshatrapas of western India
(c) the military expeditions and exploits of Samudragupta
(d) the love affair between a Gupta King and a princess of Kamarupa
Ans. (a)

11

57.

58.

59.

60.

Explanation:
'Mrichhakatikam' of Sudraka deals with the love affair of a rich merchant called
'Charudatta' and the daughter of a courtesan called 'Vasanthasena'.
'Mrichchakatika' is also known as 'Little clay cart'.
Consider the following statements :
1. Vardhaman Mahaviras mother was the daughter of Lichchhavi chief
Chetaka.
2. Gautama Buddhas mother was a princess from the Koshalan dynasty.
3. Parshvanatha, the twenty-third Tirthankara, belonged to Banaras.
Which of these statements is/are correct ?
(a) Only 1
(b) Only 2
(c) 2 and 3
(d) 1,2 and 3
Ans. (c)
Explanation:
Trisala, the mother of Vardhaman Mahavira, was the sister of Lichchhavi Chief
Chetaka.
1st Tirthankara
: Rishahhnath
23rd Tirthankara
: Parshvanath
24th & last Tirthankara : Vardhamana Mahavira
Consider the following statements :
1. The last Mauryan ruler, Brihadratha was assassinated by his commander-inehief, Piishyamitra Sunga.
2. The last Sunga king, Devabhuti was assassinated by his Brahmana minister
Vasudeva Kanva who usurped the throne,
3. The last ruler of the Kanva dynasty was deposed by the Andhras,
Which of these statements is/are correct ?
(a) 1 and 2
(b) Only 2
(c) Only 3
(d) 1, 2 and 3
Ans. (d)
Explanation:
Pushyamitra Sunga killed Brihadratha to establish Sunga' dynasty. Vasudeva
Kanva established Kanva dynasty by killing the last Sunga ruler, Devabhuti.
The last ruler of Kanva dynasty 'Susharma' was killed by Andhra Simuk to
establish Satavahana dynasty'.
2004
Consider the following statements:
1. The Chinese pilgrim Fa - Hien attended the fourth Great Buddhist Counsil
held by Kanishka.
2. The Chinese pilgrim Hiuen - Tsang met Harsha and found him to be
antagonistic to Buddhism.
Which of the statements given above is/are correct?
(a) 1 only
(b) 2 only
(c) Both 1 and 2
(d) Neither 1
nor 2
Ans. (d)
Explanation:
Fa-Hien came to India during Chandragupta-II reign (399-414 A.D ). Harsha
was not antagonistic to Buddhism when Hiuen-Tsang met hirn. He was a
Shaiva in the beginning hut embraced Buddhism gradually.
With reference to ancient Jainism, Which one of the following statements Is
correct ?
(a) Jainism was spread in South India under the leadership of Sthulabahu

12

61.

62.

63.

64.

65.

66.

(b) The Jainas who remained under the leadership of Bhadrabahu were called
Shvetambaras after the Council held at Pataliputra
(c) jainism enjoyed the patronage of the Kalinga king Kharavela in the first
century BC
(d) In the initial stage of Jainism, the Jainas worshipped images unlike
Buddhists
Ans, (c)
Explanation:
Jainism enjoyed the patronage of the Kalinga King Kharvela in the first century
B.C. Bhadrabahu went to South to lead the Digambaras. Sthulabahu was the
leader of the Swetambaras. Jainism worshipped images in its later phase.
Which one of the following four Vedas contains an account of magical charms
and spells ?
(a) Rig-veda
(b) Yajur-veda
(c) Atharva-veda
(d) Sama-veda
Ans. (c)
Explanation:
Atharva Veda contains Magical charms, spelis and folk traditions.
Who among the following was not a contemporary of the other three ?
(a) Bimbisara
(b) Gautama Buddha (c) Milinda
(d) Prasenjit
Ans. (c)
Explanation:
Milinda was not a contemporary of Bimbisara, Gautama Buddha and Prasenjit
who were belong to pre-Mauryan period whereas Milinda to posi-Mauryan
period. Milinda an Indo-Greek King was famous for his conversation with 'Naga
Sena1, which was recorded as Milind-Panho.
2006
The initial design and construction os which massive temple took place during
the reign of Suryavarman II ?
(a) Sri Mariamman Temple
(b) Angkor Vat
(c) Batu Caves Temple
(d) Kamakhya Temple
Ans. (b)
Explanation:
The initial design and construction of a massive temple Angkor Vat" took place
during the reign of Surya Varman II (1113-1150 A.D.) at Angkoc Cambodia.
Who among the following laid the foundation of Rashtrakuta Empire ?
(a) Amoghavarsha I (b) Dantidurga
(c) Dhruva
(d) Krishna I
Ans. (b)
Explanation:
Dantidurga laid the foundation for Rashtrakuta empire by overthrowing the last
Chalukyan ruler of Badami, Kirti Varman-II. He built Dasavatara caves at Ellora.
With reference to the invaders in ancient India, which one of the following is the
correct chronological order ?
(a) Greeks - Sakas - Kushans
(b) Greeks - Kushans - Sakas
(c) Sakas - Greeks - Kushans
(d) Sakas - Kushans - Greeks
Ans. (a)
Explanation:
Indo-Greeks or Bactrians - 190 B.C.
Sakas or Scythians - 90 B.C.
Kushanas or Yueh-Chis - 45 A.D.
Consider the following statements :

13

1. The Ikshvaku rulers of Southern India were antagonistic towards Buddhism.


2. The Pala rulers of Eastern India were patrons of Buddhism.
Which of the statements given above is/are correct?
(a) 1 only
(b) 2 only
(c) Both 1 and 2
(d) Neither 1
nor 2
Ans. (b)
Explanation:
The Ikshvakus of Andhra might have been a distant branch of main Ikshvaku
family of Ayodhya established their own kingdom in the Krishna-Guntur region
with Amaravati as their capital. They were patrons of Amaravati school of Art
centering on Buddhist theme. Buddhism spread to Tibet, Java, Sumatra and
Malaysia during the Pala period. 'Devapala' permitted the Sailendra ruler of Sri
Vijaya (Indonesia) 'Balaputradeva' to construct vihara at Nalanda. Dharmapala
founded Vikramasila University and revived Nalanda University.
67. The Allahabad Pillar inscription is associated with which one of the following ?
(a) Mahapadma Nanda
(b) Chandragupta Maurya
(c) Ashoka
(d) Samudragupta
Ans. (d)
Explanation:
The Allahabad pillar inscription is associated with 'Samudragupta'.
68. Match List - I with List - II and select the correct answer using the code given
below the lists:
List - I (Finding/Invention/Calculation)
List - II (Ancient Indian Scholar)
A. Time Taken by the Earth to orbit the Sun
1. Aryabhatta
B. Calculation of the value of (pi)
2. Bhaskaracharya
C. Invention of the digit zero
3. Budhayana
D. The game of snakes & ladders
4. Gyandev
A B C D
A B C D
(a) 2 4 1 3
(b) 1 3 2 4
(c) 2 3 1 4
(d) 1 4 2 3
Ans. (c)
Explanation:
Time taken by the Earth to orbit the Sun - Bhaskaracharya
Calculation of the value of (pi) - Budhayana
Inventiion of the digit zero - Aryabhatta
The game of snake and ladders - Gyandev.

2. MEDIEVAL INDIA
1.

1995
The Nagara, the Dravida, the Vesara are
(a) the three main racial group of the indian subcontinent
(b) the three main linguistic divisions into which the languages of India can be
classified
(c) the three main styles of Indian temple architecture
(d) the three main musical gharanas prevalent in India
Ans. (c)
Explanation:
The Nagara, the Dravida, the Vesars are the three main styles of Indian temple
architecture. The Nagara style was prevalent in North India in the region
between the Himalayas and the Vindhyas, The Dravida style was prevalent in
14

2,

3.

4.

5.

6.

7.

the far south-in the region between Krishna and Kanyakumari. The Vesara style
(Cbaiukyan style) was the hybrid or mixture of Dravidian and Nagara styles
emerged under the rule of later Chalukyas but flourished under Hoyasalas of
Dwarasamudram (Halebid),
The Mughal school of painting formed the spinal column of the various schools
of Indian miniature art. Which one of the following painting styles was not
affected by Mughal painting ?
(a) Pahari
(b) Rajasthani
(c) Kangra
(d) Kalighata
Ans. (d)
Explanation:
Unlike Pahari, Rajasthani and Kangra; Kalighata school of painting was
indigenous and not affected by Mughal school of painting.
Who among the following were famous jurists of medieval India ?
1. Vijnanesvara
2. Hemadri
3. Rajasekhara
4.
Jimutavahana
(a) 1, 2 and 3
(b) 2, 3 and 4
(c) 1, 2 and 4
(d) 1 and 4
Ans. (c)
Explanation
Hemadri, Vijnaneshwar and Jimutavahana are famous jurists in the medieval
India. Vijnaneshwar and jimutavahana wrote books 'Mitakshara' and
'Dayabhaga' respectively.
Bronze icons of Nataraja cast during the chola period invariably show the deity
with
(a) eight hands
(b) six hands
(c) four hands
(d) two hands
Ans. (c)
Explanation
Bronze icons of Nataraja cast during the chola period invariably show the deity
with four hands, which is one of the finest example of Chola sculpture.
Which one of the following monuments has a dome which is said to be one of
the largest in the world ?
(a) Tomb of Sher Shah, Sasaram
(b) Jama Masjid, Delhi
(c) Tomb of Ghiyas-ud-din Tughlaq, Delhi (d) Gol Gumbaz, Bijapur
Ans. (d)
Explanation:
Gol Gumbaz of Bijapur is said to be the largest dome in the world built by 'Adil
Shah'.
Ashtapradhan was a Council of ministers
(a) in the Gupta administration
(b) in the Chola administration
(c) in the Vijayanagar administration
(d) in the Maratha administration
Ans. (d)
Explanation:
'Ashtapradhan' was a Council of eight ministers constituted by Shivaji in
Maratha administration.
Consider the map given below :
The route indicated in the map was followed, during the course of his military
expedition, by
(a) Chandragupta II (b) Harshavardhana (c) Rajendra ChoSa (d) Malik Kafur
Ans. (d)
Explanation:

15

8.

9.

10.

11.

12.

13.

14.

The route indicates the military expeditions by Malik Kafur who was sent by
Allauddin Khalji for Southern expedition who reached up to Kanya Kumari.
The Modi script was employed in the documents of the :
(a) Wodeyars
(b) Zamorins
(c) Hoysalas
(d) Marathas
Ans. (d)
Explanation
In order to keep the revenue and administrative records, the Modi script was
employed In the documents of Marathas.
1996
Nastaiiq was
(a) a oersian script used in medieval India (b) a raga composed by Tansen
(c) a cess levied by the Mughal rulers (d) a manual of code of conduct for the
Ulemas
Ans. (a)
Explanation :
Nastaiiq was a persian script used in Medieval India during the Mughal period.
The sufi saint who maintained that devotional music was one way of coming
close to God was
(a) Muin-ud-din Chishti
(b) Baba Farid
(c) Saiyid Muhammad Gesudaraz
(d) Shah Alam Bukhari
Ans. (b)
Explanation:
Baba Farid Ganjshakkar of Chisti Silsilah, a Sufi saint maintained that
devotional music was one way of coming close to God.
Mughal painting reached its zenith under
(a) Humayun
(b) Akbar
(c) Jahangir
(d) Shahjahan
Ans. (c)
Explanation:
Mughal painting reached its zenith under Jahangir. Jahangir claimed that he
could identify the painter just by looking at his work.
In medieval India, Mansabdari system was introduced mainly for
(a) making recruitment to the army
(b) facilitating revenue collection
(c) ensuring religious harmony
(d) effecting clean administration
Ans. (d)
Explanation:
In Medieval India, the Mansabdari System was introduced by Akbar for
effecting dean administration and to bring stability in his kingdom.
Prem Vatika, poems on the life of Krishna, was composed by :
(a) Bihari
(b) Surdas
(c) Raskhan
(d) Kabir
Ans. (c)
Explanation:
Prem Vatika', poems on the life of Krishna was composed by 'Raskhan', a
great devotee of Lord Krishna.
1997
After consolidating his power, Balban assumed the grand title of
(a) Tute-i-Hind
(b) Kaisr-i-Hind
(c) Zil-i-Ilahi
(d) Din-i-Ilahi
Ans. (c)
Explanation:
After consolidating his power, Balban assumed the grand title of Zil-i-Iiabi
(Shadow of God).

16

15. The head of the military department under the recognised central machinery of
administration during Akbars reign was
(a) Diwan
(b) Mir Bakshi
(c) Mir Saman
(d) Bakshi
Ans. (b)
Explanation:
'Mir Bakshi' was the head of the military department under the recognised
central machinery of administration during Akbar's reign.
16. Consider the following statements :
Assertion (A) : The sponsor and the most prominent figure of the Chisti order
of Sufis in India is Khwaja Moinuddin Chisti.
Reason (R) : The Chisti order takes its name from a village Chisti in Ajmer.
In the context of the above two statements, which one of the following is correct
?
(a) Both A and R are true and R is the correct explanation of A
(b) Both A and R are true but R is not the correct explanation of A
(c) A is true but R is false
(d)
A is false but R
is true
Ans. (c)
Explanation:
Khwaja Moinuddin Chisti introduced "the Chisti Siisilah" into India who came
along with Mohammad Ghori in 1192. Chisti is the name of place in Afghanistan
where the founder of Chisti Siisilah "Khwaja Abu Ishak Siyami of Syria" had
died.
17. The medieval Indian .writer who refers to the discovery of America is
(a) Malik Muhammad Jayasi
(b) Amir Khusrau
(c) Raskhan
(d) Abul Fazl
Ans. (d)
Explanation:
'Abul Fazl is the Medieval Indian writer who refers to the discovery of America.
He called it as 'Alumnau'.
18. The Badami rock inscription of Pulakesin I is dated in the Saka year 465. If the
same were to be dated in Vikraraa Samvat, the year would be
(a) 601
(b) 300
(c) 330
(d) 407
Ans. (a)
Explanation :
To convert Badami rock inscription as per Vikram Samvat, the year would be
'601' (465 + 78 + 58), where 78 A.D. is the starting year of Saka Era and 58
B.C. is the starting year of Vikram Samvat.
1998
Directions : The following items consist of two statements, one labelled as the
Assertion (A) and the other as Reason (R). You are to examine these two
statements carefully and select the answers to these items using the code
given below :
(a) Both A and R are true and R is the correct explanation of A
(b) Both A and R are true but R is not the correct explanation of A
(c) A is true but R is false
(d)
A is false but R
is true
19. Assertion A : At first the Turkish administration in India was essentially
military.

17

20.

21.

22.

23.

Reason R : The country was parcelled out as 'Iqtas' among leading military
leaders.
Ans. (a)
Explanation:
Initially, the Turkish administration in India was essentially military and the
country was parcelled out as 'Iqtas' among leading military leaders.
Assertion A : During the reign of Shahjahan, Dara Sikoh was sent on
expedition to Balkha, Badakh shan and Qandahar.
Reason R : The expedition sent by Shahjahan to the Middle-East was a
marvellous success.
Ans. (c)
Explanation:
Shahjahan's Balkha campaign was a failure,
Consider the following :
1. Tughlaquabad fort.
2.
Lodi garden.
3. Qutub Minar.
4. Fatehpur Sikri
The correct chronological order in which they were built is :
(a) 3, 1, 4, 2
(b) 3, 1, 2, 4
(c) 1, 3, 2, 4
(d) 1, 3, 4, 2
Ans. (b)
Explanation:
Tughlaqabad fort (1414-1451 A.D.), Lodi Garden (1451-1526 A.D.), Qutub
Minar (1193-1320 A.D.), Fatehpur Sikri (1569 A.D.)
Match List-I with List-II and select the correct answer using the codes given
below the lists:
List I
List II
A. 1556
1. Battle of Haldi Gnati
B. 1600
2. Nadir Shah's capture of Delhi
C. 1686
3. Death of Shivaji
D. 1739
4. Grant of Charter to East India
Company
5. Accession of Akbar
A B C D
A B C D
(a) 3 4 2 1
(b) 5 4 3 2
(c) 5 2 1 4
(d) 1 5 3 2
Ans. (b)
Explanation:
1556
- Accession of Akbar
1600
- Grant of charter to East India Company
1686
- Death of Shivaji
1739
- Nadir Shah's capture of Delhi
In the given map, the shaded part represents Akbar's empire at a certain
juncture : A stands for an independent country and !B! marks the site of a city.
Whieh one of the following alternatives given all correct information ?
(a) Akbar in 1557
(b) Akbar in 1557
(c) Akbar in 1605
(d) Akbar in 1605
Ans. (d)
Explanation

:
:
:
:

(A) Golkonda, (B) Lahore


(A) Khandesh, (B) Multan
(A) Gondwana, (B) Multan
(A) Gondwana, (B) Lahore

18

24.

25.

26.

27.

28.

The shaded part represents Akbar's empire at the time of his death. 'A' stands
for 'Gondwana' an independent country, while 'B' marks the city of 'Lahore'.
The member of Shivaji's Astha Pradhana who looked after foreign affairs was :
(a) Peshwa
(b) Sachiv
(c) Pandit Rao
(d) Sumant
Ans. (d)
Explanation:
'Sumant' looks after foreign affairs. Shivaji's 'Ashtapradhan' consisted of a
council of eight ministers to advise him.They are : 1. Peshwa, 2. Pandit Rao, 3.
Sumant, 4. Amatya, 5. Mantri, 6. Sachiv, 7. Senapati, 8. Nyayadish.
The loss of Qandhar was a big blow to the Mughal empire from the view point
of
(a) natural resources
(b) buffer territory
(c) communication
(d) strategic stronghold
Ans. (d)
Explanation:
The loss of Qandhar was a big blow to the Mughal empire from the view point
of 'Strategic hold'.
Fawazii in the Sultanate period means
(a) extra payment made to the nobles
(b) revenue assigned in lieu of salary
(c) excess amount paid to the exchequer by the iqtadars
(d) illegal exactions extracted from the peasants
Ans. (c)
Explanation:
The excess amount paid to the exchequer by the Iqtadars in the Sultanate
period is called "Fawazii".
The Sultan of Delhi who is reputed to have built the biggest network of canals in
India was
(a) lltutmish
(b) Ghiyasuddin Tughlaq
(c) Feroze Shah Tughlaq
(d) Sikandar Lodi
Ans. (c)
Explanation:
Feroj Shah Tughlaq was reputed to have built the biggest network or canals in
India during sullanaie period. Some of the canals he built were 1. Sirsa to
Hansi, 2. Sutlej to Dipaipur, 3. Yamuna to Sirmaur.
Consider the following statements :
Ahadis were those troopers who
1. offered their services singly.
2. did not attach themselves to any
chief.
3. has the emperor as their immediate colonel.
4. attached themselves to
Mirzas.
Of these statements
(a) 1, 3 and 4 are correct
(b) 1, 2 & 3 are correct
(c) 2 and 3 are correct
(d) 1 and 4 are correct
Ans. (b)
Explanation:
Ahadis were those troopers who were appointed directly by Mughal emperor,
had the emperor as their immediate Colonel. They were not responsible to any
Chief/Mirza and were responsible and highly loyal to the emperor.
1999

19

29. One consistent feature found in the history of southern India was the growth of
small regional kingdoms rather than large empires because of
(a) the absence of minerals like iron
(b) too many divisions in the social
structure
(c) the absence of vast areas of fertile land (d) the scarcity of manpower
Ans. (c)
Explanation:
Growth of small regional kingdoms rather than large empires in South India was
due to absence of vast areas of fertile land.
30. 'The king was freed from his people and they from their king.' On whose death
did Badauni
comment thus ?
(a) Balban
(b) Ala-ud-din Khalji
(c) Muhammad-bin-Tughlak
(d) Feroze Shah Tughlak
Ans. (c)
Explanation:
This statement was quoted by Badauni in his work "Muntkhab-ul-lubad" on the
death of 'Muhammad-bin Tughlaq'.
31. Consider the following statements :
The striking feature of the Jama Masjid in Kashmir completed by Zain-ul-Abidin
include(s)
1. turret
2. similarity with Buddhist pagodas
3. Persian style
Which of the above statements is/are correct?
(a) 1 alone
(b) 1, 2 and 3
(c) 2 and 3
(d) 1 and 3
Ans. (b)
Explanation:
Jama Masjid in Kashmir built by Zain-ul-Abdin is similar to Buddhist pagoda
with persian style and also includes 'turret' in its architecture.
32. Which one of the following pairs is not correctly matched ?
(a) Jahangir
: William Hawkins
(b) Akbar
: Sir Thomas Roe
(c) Shahjahan
: Travemier
(d) Aurangzeb
: Manucci
Ans. (b)
Explanation:
Sir Thomas Roe visited Jahangir durbar in the year 1608.
33. The first writer to use Urdu as the medium of poetic expression was
(a) Amir Khusrau
(b) Mirza Ghalib
(c) Bahadur Shah Zafar (d) Faiz
Ans. (a)
Explanation:
Amir Khusrau was the first writer to use 'L)rdu! as the medium of poetic
expression,
34. To which Lodi Sultan does the given map relate and what town does the site
marked A on the map represent ?
(a) Behlol Lodi, Jaunpur
(c) Ibrahim Lodi, Jaunpur
Aligarh
Ans. (a)
Explanation:

(b) Sikandar Lodi, Aligarh


(d)
Ibrahim

20

Lodi,

35.

36.

37.

38.

The given map is related to Behiol Lodi and the town marked represents
Jaunpur.
Directions : The following items consist of two statements, one labelled as the
Assertion (A) and the other as Reason (R). You are to examine these two
statements carefully and select the answers to these items using the code
given below :
(a) Both A and R are true and R is the correct explanation of A
(b) Both A and R are true but R is NOT the correct explanation of A
(c) A is true but R is false
(d) A is false but R is true
Assertion (A) : During the time of Akbar, for every ten cavalrymen, the
mansabdars had to maintain twenty horses.
Reason (R) : Horses had to be rested while on march and replacements were
necessary in times of war.
Ans. (d)
Explanation:
This was started during Jahangir's period, who introduced 'Duh Aspa Sih Aspa'
in Mansabdari.
2000
Who among the following streamlined the Maratha administration after
Sjambhaji ?
(a) R.aja Ram
(b) Balaji Viswanath (c)GangaBai
(d)
Nanaji
Deshmukh
Ans. (b)
Explanation:
Peshwa Balaji Viswanath streamlined the Maratha administration after
Sambhaji.
Match List-1 wlxia ListII and select the correct answer using the codes given
below the lists:
List-I
List-II
A. Iqta
1. Marathas
B. jagir
2. Delhi Sultans
C. Amaram
3. Mughals
D. Mokasa
4. Vijayanagara
A B C D
A B C D
(a) 3 2 1 4
b) 2 3 4 1
(c) 2 3 1 4
(d) 3 2 4 1
Ans. (b)
Explanation:
These were all land grants.
Iqta - Delhi Sultans, Jagir - Mughals, Amaram - Vijayanagara rulers, Mokasa Marathas
Which one of the following Muslim rulers was hailed as the iagadguru' by his
Muslim subjects because of his belief in secularism ?
(a) Husain Shah
(b) Zain-ul-Abidin
(c) Ibrahim Adil Shah (d) Mahmud
II
Ans. (c)
Explanation:
Ibrahim Adil Shah was called 'Jagadg'uru' and 'Adilbaba' by his people because
of his belief in Secularism.
2001

21

39. Which one of the Chola kings conquered Ceylon ?


(a) Aditya I
(b) Rajaraja I
(c) Rajendra
(d) Vijayalaya
Arts. (b)
Explanation:
Rajaraja-I attacked and conquered Ceylon and later by Rajendra-I under whom
the Ceylon was completely subjugated.
40. The shaded area in the above map shows the empire of :

41.

42.

43.

44.

(a) Ala-ud-din Khalji (b) Mohammed Tughlaq (c) Shahjahan


(d) Aurangzeb
Ans. (c)
Explanation:
The shaded area was the empire of Shahjahan. The territories of Golkonda and
Bijapur were later annexed into Mughal empire by Aurangzeb.
Hoysaia monuments are found in
(a) Hampi & Hospet
(b) Haiebid & Belur
(c) Mysore & Bangalore
(d) Sringeri & Dharwar
Ans. (b)
Explanation:
Hoysaia monuments were found in Haiebid and Belur. Hoysalas capital city
was 'Haiebid'.
Match List I with List II and select the correct answer using the codes given
below the lists:
List I (Bhakti Saint)
List
II
(Profession)
A. Namdev
1. Barber
B. Kabir
2. Weaver
C. Ravid as
3. Tailor
D. Sena
4. Cobbler
A B C D
A B C D
(a) 2 3 1 4
(b) 3 2 4 1
(c) 3 2 1 4
(d) 2 3 4 1
Ans. (b)
Explanation
Namdev - Tailor, Ravidas - Cobbler, Kabir - Weaver, Sena - Barber.
All of the above Bhakti saints were disciples of Swami Ramanand.
The Mongols under Chengiz Khan invaded India during the reign of
(a) Balban
(b) Feroz Tughlaq
(c) Htutmish
(b)Muhammad bin
Tughlaq
Ans. (c)
Explanation:
The Mongols under Chengiz Khan invaded India during the reign of Iltutmish
but did not enter deep in India as the persian king was not given shelter by
Iltutmish, whom Chengiz Khan was chasing.
Which among the following ports was caiied Babul Makka (Gate of Makka)
during the Mughal Period ?
(a) Calicut
(b) Broach
(c) Cambay
(d) Surat
Ans. (c)
Explanation:
Cambay was called Babul Makka (Gate of Makka) during the Mughal period
because the pilgrimage to Makka was made from here.

22

45. Which of the following pairs is correctly matched ?


(a) Dewan-i-Bandagani : Tughlaq
(b) Dewan-i-Mustakhraj : Balban
(c) Dewan-i-Kohi
: Alauddin Khilji
(d) Dewan-i-Arz
: Muhammad Tughlaq
Ans. (a)
Explanation:
Dewan-i-Bandagani : by Feroz- Tughlaq (Department of Slaves)
Dewan-i-Mustakhraj : Allaudin Khalji Dewan-i-Kohi : Muhammad Bin Tughlaq
Dewan-i-Arz : Balban
Directions ; The following items consist of two statements, one labelled as the
Assertion (A) and the other as Reason (R). You are to examine these two
statements carefully and select the answers to these items using the code
given below :
(a) Both A and R are true and R is the correct explanation of A
(b) Both A and R are true but R is NOT the correct explanation of A
(c) A is true but R is false
(d) A is false but R is true
46. Assertion (A) : The Battle of Khanha was certainly more decisive and
significant than the First Battle of Panipat.
Reason (R) : Rana Sanga, the Rajput hero, was certainly a more formidable
adversary than Ibrahim Lodi.
Ans. (a)
Explanation:
Rana Sanga was more formidable adversary than Ibrahim Lodi as he had once
defeated Ibrahim Lodi. So battle of Khanva was certainly more significant and
decisive than first battle of Panipat.
2002
47. Assertion (A) : Muhammad bin Tughlaq left Delhi, and, for two years lived in a
camp called Swarga-dwari.
Reason (R) : At that time, Delhi was ravaged by a form of plague and many
people died.
Ans. (a)
Explanation:
Because of plague in Delhi, Mohammad Bin Tughlaq left Delhi and lived in a
camp called Swarga-dwari for two years.
48. The historian Barani refused to consider the state in India under Delhi Sultans
as truly Islamic because
(a) the majority of the population did not follow Islam
(b) the Muslim theologists were often disregarded
(c) the Sultan supplemented the Muslim lav/ by framing his own regulations
(d) religious freedom was accorded to non-Muslims
Ans. (c)
Explanation:
Because that, the Sultans had supplemented the Muslim Law by framing their
own regulations, the historian 'Barani' has refused to consider the state in India
under Delhi Sultans as truly Islamic.
49. With reference to Sufism In Indian history, consider the following statements ;
1. Shaikh Ahmad Sarhandi was a contemporary of Ibrahim Lodi.
2. Shaikh Nasiruddin Chirg-i-Dehlavi was a disciple of Shaikh Nizarnuddin
Auliya.

23

50.

51.

52.

53.

3. Aurangzeb was a contemporary of Shaikh Salim Chisti.


4. The Qadiri order of Sufis was first introduced in India by Shaikh Niamtullah
and Makhdum Muhammad Jilani.
Which of these statements are correct ?
(a) 1 and 2
(b) 1 and 3
(c) 2 and 3
(d) 2 and 4
Ans. (d)
Explanation:
Shaik Nasiruddin Chirg-i-Dehlavi was a disciple of Shaik Nizarnuddin Auliya
and Qadiri order was first introduced in India by Shaik Niamtullah and
Makhdum Muhammad Jilani.
Who among the following was the first Bhakti saint to use Hindi for the
propagation of his message ?
(a) Dadu
(b) Kabir
(c) Ramananda
(d) Tulsidas
Ans. (c)
Explanation:
Ramananda was the first Bhakti saint to use Hindi for the propagation of his
message. He brought Bhaktism to North from South.
With reference to medieval Indian rulers, which one of the following statements
is correct?
(a) Alauddin Khalji first set up a separate ariz's department
(b) Balban introduced the branding system of horses in his military
(c) Muhammad bin Tughlaq was succeded by his uncle to the Delhi throne
(d) Firoz Tughlaq set up a separate department of slaves
Ans. (d)
Explanation:
Firoz Tughlaq set up a separate department of slaves known as "Diwan-iBandagan".
Separate ariz's department was set up by Balban. Allauddin Khalji introduced
the branding system of Horses in his military.
Muhammad Bin Tughlaq was succeeded by his cousin Firo?. Tughlaq.
The motive behind Shah Jahans Baikh campaign was to
(a) secure a friendly ruler in Balkh and Badakshan which bordered Kabul
(b) conquer Samarqand and Farghana, the Mughal homelands
(c) fix the Mughal frontier on the 'scientific line', the Amu Daria
(d) expand the Mughal Empire beyond the subcontinent
Ans. (a)
Explanation:
To secure a friendly ruler in Balkh and Badakshan which bordered Kabul, which
helps in controlling the rebel Afghan tribes ruling near Gazani and Khyber.
2003
Directions : The following items consist of two statements, one labelled as the
Assertion (A)' and the other as Reason (R). You are to examine these two
statements carefully and select the answers to these items using the code
given below :
(a) Both A and R are true and R is the correct explanation of A
(b) Both A and R are true but R is NOT the correct explanation of A
(c) A is true but R is false
(d) A is false but R is true
Assertion (A) : Emperor Akbar marched towards Afghanistan in 1581 with a
huge army.

24

Reason (R) : He was on his way to reclaim his ancestral country of Ferghana
in Central Asia.
Ans. (c)
Explanation:
Akbar marched towards Afghanistan in 1581 to create a safety valye to his
empire, but not to reclaim his ancestral country'of Ferghana in Central Asia.
54. Assertion (A) : Saluva Narasimha put an end to the old dynasty and assumed
the royal title.
Reason (R) : He wanted to save the kingdom from further degeneration and
disintegration.
Ans. (c)
Explanation:
Saluva Narasimha ended the Sangama dynasty and seized the throne for
himself and started the Saluva dynasty. He assumed the royal title due to his
greed for power.
55. Assertion (A) : Marathas emerged as the strongest native power in India after
the decline of Mughal empire.
Reason (R) : Marathas were the first to have a clear concept of a united Indian
nation.
Ans. (c)
Explanation:
Marathas rose on the spoils of Mughal decline. Aurangzeb's religious policy
coupled with growth of Maharashtradharma grew as a power enough to
dominate the political scene. But their feudal division was not a true approach
to United India concept.
56. Alam Khan, one of those who invited Babur to invade India was
(a) an uncle of Ibrahim Lodi and a pretender to the throne of Delhi
(b) a cousin of Ibrahim Lodi who was ill-treated and expelled from the country
(c) the father of Dilawar Khan to whom cruel treatment was meted out by
Ibrahim Lodi
(d) a high official in Punjab province who was very much discontented with
Ibrahim Lodis treatment to his tribe
Ans. (a)
Explanation:
Alam Khan was an uncle of Ibrahim Lodi and a pretender to the throne of Delhi.
57. The battle of Bharmat was fought between
(a) Muhammad Ghori and Jai Chand
(b) Babur and Afghans
(c) Aurangzeb and Dara Shikoh
(d) Ahmad Shah Durrani and the
Marathas
Ans. (c)
Explanation:
Dara Shikoh was a brother of Aurangzeb and both fought against each other for
Delhi's throne with the support of Raja Jaswant Singh and Prince Murad
respectively.
58. How did the MughaS Emperor Jahandarshahs reign come to an early end ?
(a) He was deposed by his Wazir
(b) He died due to a slip while climbing down steps
(c) He was defeated by his nephew in a battle
(d) He died of sickness due to too much consumption of wine
Ans. (c)

25

59.

60.

61.

62.

63.

Explanation:
Jahandar Shah was defeated and killed by his nephew Farrukhsiyar with the
help of Sayyid brothers.
Consider the following statements :
1. Kitab-i-Nauras, a collection of songs in praise of Hindu deities and Muslim
saints, was written by Ibrahim Adil Shah II.
2. Amir Khusrau was the originator in India of the early form of the musical
style known as Oawali.
Which of these statements is/are correct ?
(a) Only 1
(b) Only 2
(c) Both 1 and 2
(d) Neither 1
nor 2
Ans. (c)
Explanation:
Ibrahim Adil Shah II wrote Kitab-i-Nauras. Amir Khusrau was the originator in
India of the early form of musical style known as Qawali.
How did Sultan Qutb-ud-din Aibak die ?
(a) He was treacherously stabbed to death by one of his ambitious nobles
(b) He was killed in a battle with Taj-u-din Yildiz, the ruler of Ghazni who
entered into a contest with him over the capture of Punjab
(c) He sustained injuries while besieging the fortress of Kalinjar in Bundelkhand
and succumbed to them later
(d) He died after a fall from his horse while playing Chaugctn
Ans. (d)
Explanation:
Qutubuddin Aibak died after a fall from his horse while playing 'Chaugan'.
Consider the following statements:
1. Narasimha Saluva ended the Sangama dynasty and seized the throne for
himself & started the Saluva dynasty.
2. Vira Narasimha deposed the last Saluva ruler and seized the throne for
himself.
3. Vira Narasimha was succeeded by his younger brother, Krishnadeva Raya.
4. Krishnadeva Raya was succeeded by his half-brother, Achyuta Raya.
Which of the statements given above are correct?
(a) 1, 2 and 3
(b) 2, 3 and 4
(c) 1 and 4
(d) 1, 2, 3 and
4
Ans. (d)
Explanation:
Vir Narasimha started Tuluva dynasty by deposing the last Saluva ruler and
was succeeded by his younger brother, Krishna devaraya who was succeeded
by his half brother Achyuta Raya.
Who was the last ruler of the Tughluq dynasty of the Delhi Sultanate ?
(a) Firuz Shah Tughlaq
(b) Ghiyas-ud-din Tughlaq Shah II
(c) Nasir-ud-din Mahmud
(d) Nasrat Shah
Ans. (c)
Explanation:
Nasir-ud-din Mahmud was the last ruler. He was succeeded by Khizr Khan of
Saiyyad dynasty.
How did the dynasty of Nizam Shahis of Ahmadnagar come to an end ?
(a) Ahmadnagar was annexed into Mughal empire and Husain Shah was
consigned to life imprisonment

26

(b) Mughal troops destroyed Daulatabad fort and killed Nizam- ul Mulk of
Ahmadnagar
(c) Fateh Khan usurped the throne from Nizam-ul-Mulk
(d) Malik Ambar was defeated in a battle with Mughals in 1631 and the entire
royal family was killed by the Mughal troops
Ans. (a)
Explanation:
Ahmednagar was annexed into Mughal empire and Husain Shah was
consigned to life imprisonment.
64. Which one of the following sequences indicates the correct chronological order
?
(a) Shankaracharya - Ramanuja - Chaitanya
(b) Ramanuja - Shankaracharya - Chaitanya
(c) Ramanuja - Chaitanya - Shankaracharya
(d) Shankaracharya - Chaitanya - Ramanuja
Ans. (a)
Explanation:
Shankaracharya
- 8th Century A.D.
Ramanuja
- 12th Century A.D.
Chaitanya
- 15th Cenury A.D.
65. Consider the following statements about Sikh Gums :
1. Banda Bahadur was appointed as the military leader of the Sikhs by Guru
Tegh Bahadur.
2. Guru Arjun Dev became the Sikh Guru after Guru Ram Das.
3. Guru Arjun Dev gave to Sikhs their own script-Guru mukhi.
Which of the statements given above is/are correct ?
(a) 1 only
(b) 2 and 3
(c) 1 and 3
(d) 1 and 2
Ans. (d)
Explanation:
Banda Bahadur was appointed as the military leader of the Sikhs by Guru Tegh
Bahadur. Guru Angad (second guru) gave to sikhs their own script Guru Mukhi.
Guru Arjun Dev (5th guru) became Sikh Guru after Guru Ram Das (4th guru).
66. Consider the following statements:
1. In the Third Battle of Panipat, Ahmed Shah Abdali defeated Ibrahim Lodi.
2. Tipu Sultan was killed in the Third Anglo-Mysore War.
3. Mir Jafar entered in a conspiracy with the English for the defeat of Nawab
Siraj-ud-daulah in the Battle of Plassey.
Which of the statements given above is/are correct?
(a) 1, 2 and 3
(b) 3 only
(c) 2 and 3
(d) None
Ans. (b)
Explanation:
Ahmed Shah Abdali defeated the Marathas in the third battle of Panipat. Tipu
Sultan was killed in the Fourth Anglo-Mysore War.
Directions : The following items consist of two statements, one labelled as the
Assertion (A) and the other as Reason (R). You are to examine these two
statements carefully and select the answers to these items using the code
given below :
(a) Both A and R are true and R is the correct explanation of A
(b) Both A and R are true but R is NOT the correct explanation of A
(c) A is true but R is false
(d) A is false but R is true

27

61. Assertion (A) : Muhammad bin Tughluq issued a new gold coin which was
called Dinar by Ibn Batutah.
Reason (R) : Muhammad bin Tughluq wanted to issue token currency in gold
coins to promote trade with West Asian and North African countries.
Ans. (c)
Explanation:
Mohammad bin Tughlaq introduced token currency (1330), a bronze coin
having the same value as a silver tanka. But it proved unsuccessful and he had
to withdrew it.
68. Which one of the following is the correct chronological order of the Afghan
rulers to the throne of Delhi ?
(a) Sikandar Shah - Ibrahim Lodi - Bahlol Khan Lodi
(b) Sikandar Shah - Bahlol Khan Lodi - Ibrahim Lodi
(c) Bahlol Khan Lodi - Sikandar Shah - Ibrahim Lodi
(d) Bahlol Khan Lodi - Ibrahim Lodi - Sikandar Shah
Ans. (c)
Explanation:
Bahlol Khan Lodi
- 1451-1489
A.D.
Sikandar Shah (Lodi) - 1489-1517
A.D.
Ibrahim Lodi
- 1517-1526
A.D.
69. Bhakta Tukaram was & contemporary of which Mughal Emperor?
(a) Babar
(b) Akbar
(c) Jahangir
(d) Aurangzeb
Ans. (c)
Explanation:
Bhakta Tukaram (1590-1660) was a contemporary of Jahangir(1605-1627). He
is responsible for creating a background for Maratha nationalism. He was a
great devotee of Vithoba at Pandharpur. Shivaji drawn inspiration from his
works.
70. In India history, who was Abdul Hamid Lahori?
(a) An important military commander during Akbars reign
(b) An official historian of the reign of Shah Jahan
(c) An important rioble and confidant of Aurangzeb
(d) A chronicler and poet during the reign of Muhammad Shah
Ans. (b)
Explanation:
Abdul Hamid Lahori was an official historian during the reign of Shahjahan. He
wrote 'Padshanama' a book describing the events of Shahjahan's rule.
71. When Raja Wodeyar founded the Kingdom of Mysore, who was the ruler of the
Vijayanagar Empire ?
(a) Sadasiva
(b) Tirumala
(c) Ranga II
(d) Venkata II
Ans. (d)
Explanation:
Ranga II (1575-1586) was the ruler of the Vijayanagar empire when Raja
Wodeyar (1578-1617) founded the kingdom of Mysore. (Venkata-II ruled from
1586 to 1614)

3. MODERN INDIA
1.

1995
The radical wing of the Congress Party with Jawaharlal Nehru as one of its
main leaders, founded the independence for India League in opposition to
28

2.

3.

4.

5.

(a) the Gandhi-Irwin Pact


(b)
the Home Rule
Movement
(c) the Nehru report
(d) the Mont-ford reforms
Ans. (c)
Explanation:
Nehru report published in July 1928 favoured Dominion Status. But Jawaharlal
Nehru and Subhas Chandra Bose were not happy with the Dominion Status.
So, the radical wing of Congress under the leadership of J. Nehru founded
"Independence for India League" to demand Independence for the country.
Which one of the following was an emigree Communist journal of M.N. Roy ?
(a) Kisan Sabha
(b) The worker
(c) Vanguard
(d) Anushilan
Ans. (c)
Explanation:
"Radical Humanism" philosophy propounder M. N. Roy (Manavendra Nath Roy)
was the founder of emigree Communist journal "Vanguard".
Examine the map given below :
The places marked 1, 2, 3 and 4 were respectively the seats of powers of the
(a) Scindias, Holkars, Gaekwads & Bhonsies
(b) Holkars, Scindias, Gaekwads & Bhonsies
(c) Gaekwads, Bhonsies, Scindias & Holkars
(d) Scindias, Holkars, Bhonsies & Gaekwads
Ans. (a)
Explanation:
In the map 1 is Gwalior, which was under the dominance of Scindias, map 2 is
Indore, which was under the dominance of Holkars, map 3 is Baroda which was
under the dominance of Gaekwads, map 4 is Nagpur which was under the
dominance of Bhonsies.
Which one of the following. pairs is correctly matched ?
(a) Battle of Buxar
: Mir Jafar vs. Clive
(b) Battle of Wandiwash : French vs. East India Company
(c) Battle of ChiManwala : Dalhousie vs. Marathas
(d) Battle of Kharda : Nizam vs. East India Company
Ans. (b)
Explanation:
In 1761, the Battle of Wandiwash was fought between the French and East
India Company, in which the French was defeated and their power came to an
end in India.
What is the correct sequence of the following events ?
1. The Lucknow Pact
2.
The
Introduction of Dyarchy
3. The Rowlatt Act
4. The partition of Bengal
(a) 1, 3, 2, 4
(b) 4, 1, 3, 2
(c) 1, 2, 3, 4
(d) 4, 3, 2, 1
Ans. (b)
Explanation:
The Partition of Bengal was in 1905, Lucknow Pact was concluded in 1916
Lucknow session of Congress, Rowlatt Act was passed on 19 March 1919,
Introduction of Diarchy was a part of Montague-Chelmsford Reforms or
Government of India Act 1919, based on which elections were conducted in
1921.

29

6.

The word. Adivasi was used far the first tlrne to refer to the tribal people by :
(a) Mahatma Gandhi (b) Thakkar Bappa (c) Jyotiba Phule
(d)
B.
R.
Ambedkar
Ans. (b)
Explanation:
The term Adivasi, to mean primitive people of this country, was used by
Thakkar Bappa to refer to the tribal people.
7.
The Barrah dacoity was the first major venture of the revolutionary terrorists of
the freedom movement in
(a) Bombay-Kamataka
(b) Punjab
(c) East Bengal
(d) The Madras Presidency
Ans. (c)
Explanation :
Barrah dacoity took place at Barrah in E. Bengal in which State treasury was
looted. This was the first major venture of Revolutionary activtists in E. Bengal.
8.
Which of the following political parties is/are national political parties ?
1. Muslim League
2. Revolutionary Socialist Party
3. All India Forward Block
4. Peasants and Workers Party of
India
Codes :
(a) 1, 2 and 3
(b) 2 and 4
(c) 3 only
(d) None
Ans. (d)
9.
In 1930, Mahatma Gandhi started Civil Disobedience Movement from
(a) Sevagram
(b) Dandi
(c) Sabarmati
(d) Wardha
Arts. (b)
Explanation:
By breaking the Salt law in Dandi, Mahatma Gandhi started Civil Disobedience
Movement in 1930.
10. Which one of the following pairs are not correctly matched ?
(a) Jamnalai Bajaj
: Satyagraha Ashram at Wardha
(b) DadabhaiNaoroji : Bombay Association
(c) Lala Lajpat Rai : National School at Lahore
(d) Bal Gangadhar Tilak: Satya Shodhak Sabha
Ans. (d)
Explanation :
Satya Shodhak Sabha was founded by Mahatma Jyotiba Phule in 1873. But
was not by Bal Gangadhar Tilak.
11. Hughly was used as a base for piracy in the Bay of Bengal by
(a) the Portuguese (b) the French
(c) the Danish
(d) the British
Ans. (a)
Explanation:
Hughly was used as a base for piracy in the Bay of Bengal by Portuguese
pirators.
12. The Sarabands (no tax) campaign of 1922 was led by
(a) Bhagat Singh
(b) Chittaranjan Das (c) Rajaguru
(d) Vallabhbhai
Patel
Ans. (d)
Explanation:
In Gujarat, Sardar Vallabhbhai Patel led the Sarabandi (no tax) campaign.

30

13. Match List I with List II and select the correct answer using the codes given
below the lists:
List I
A. Governor General of Presidency of Fort William in Bengal (Under Regulating
Act, 1773)
B. Governor General of India (Under Charter Act, 1833)
C. Governor General and Viceroy of India (Under Indian Councils Act, 1858)
D. Governor General and Crown Representative (Under Government of India
Act, 1935)
List II
1. Archibald Percival Wavell, Viscount and Earl Wavell
2. James Andrew Broun-Ramsay, Earl and Marquess of Dalhousie
3. Charles Cornwallis 2nd Earl and First Marquess of Cornwallis
4. Gibert John EI!iot-Murray=Kynynmond, Earl of Minto
5. Louis Mountbatten, Earl Mountbatten of Burma
A B C D
A B C D
(a) 3 2 4 1
(b) 3 4 1 5
(c) 2 3 4 5
(d) 4 2 3 1
Ans. (a)
Explanation:
Governor General of Presidency of Fort William in Bengal was Lord Cornwallis
(1786-93)
Governor General of India was Lord Dalhousie (1848-56)
Governor General and Viceroy of India was Lord Minto (1905- 10)
Governor General and Crown's representative was Lord Wavell (1943-47).
14. Which one of the following first mooted the idea of a constituent assembly to
frame a constitution for India ?
(a) Swaraj Party in 1934
(b) Congress Party in 1936
(c) Muslim League in 1942
(d) All Parties Conference in 1946
Ans. (a)
Explanation:
The idea of a Constituent Assembly to form a Constitution for India was mooted
by Swaraj Party in 1934.
15. Consider the following statements :
Assertion (A) : The British sovereignty continued to exist in free India.
Reason (R) : The British sovereign appointed the last Governor General of
Free India.
In the contest of the above two statements, which one of the following is correct
?
(a) Both A and R are true and R is the correct explanation of A
(b) Both A and R are true but R is not a correct explanation of A
(c) A is true but R is false
(d)
A is raise but R
is true
Ans. (d)
Explanation:
On 15 August 1947, when India got the freedom, it became a sovereign counry.
So, British Sovereignty ceased to exist after that date. But on the request of
India, British sovereign appointed the last Governor General of Free India Lord
Mountbatten.

31

16. His principal forte was social and religious reform. He relied upon legislation to
do away with social ills and worked unceasingly for the eradication of child
marriage, the purdah system ........ To encourage consideration of social
problems on a national scale, he inaugurated the Indian National Social
Conference, which for many years met for its annual sessions alongside the
Indian National Congress.
The reference in this passage is to
(a) ishwar Chandra Vidyasagar
(b) Behramji Merwanji Malabari
(c) Mahadev Govind Ranade
(d) B. R. Ambedkar
Ans. (c)
Explanation:
In 1887, Indian National Social Conference was inaugurated by Mahadev
Govind Ranade.
17. Who among the following leaders did not believe in the drain theory of
Dadabhai Naoroji ?
(a) B. G. Tilak
(b) R. C. Dutt
(c) M. G. Ranade
(d)Sir Syed
Ahmed Khan
Ans. (d)
Explanation:
Sir Syed Ahmed Khan did not believe in the Drain theory. He was loyal to
British and was opposed to Congress ideas.
18. B. R. Ambedkar was elected to the Constituent Assembly from
(a) West Bengal
(b) the Bombay Presidency
(c) the then Madhya Bharat
(d) Punjab
Ans. (a)
Explanation:
Dr. Ambedkar was elected to the Constituent Assembly from Bengal. Though
he was a resident of Bombay Presidency he was not elected there.
19. The Anarchical and Revolutionary Crime Act (1919) was popularly known as
the
(a) Rowlatt Act
(b) Pitts India Act
(c) Indian Arms Act (d) Ilbert Bill
Ans. (a)
Explanation:
Rowlatt Act was also considered as "Black Act" by Gandhiji. It took away
Habeas Corpus which forms the basis of Civil liberties of Britain.
20: The following advertisement :
appeared in the Times of India dated 7th August
(a) 1931
(b) 1929
(c) 1921
(d) 1896
Ans. (d)
Explanation:
It was the first advertisement of Indian Cinema published in 7th August 1896 in
Times of India.
21. Who among the following was the first European to initiate the policy of taking
part in the quarrels of Indian princes with a view to acquire territories ?
(a) Clive
(b) Dupleix
(c) Albuquerque
(d) Warren
Hastings
Ans. (b)
Explanation:

32

22.

23.

24.

25.

The French Governor (Dupleix) was the first European to initiate the policy of
taking part in the quarrels of Indian princes with a view to acquire territories.
The meeting of Indian and British political leaders during 1930-32 in London
has often been referred to as the First, Second and Third Round TaDie
Conferences, it would be incorrect to refer to them as such because
(a) the Indian National Congress did not take part in two of them
(b) Indian parties other than the Indian National Congress participating in the
Conference represented sectional interests and not the whole of India
(c) the British Labour Party had withdrawn from the Conference thereby making
the proceeding of the Conference partisan
(d) it was an instance of a Conference held in three session and not that of
three separate conferences
Ans. (d)
Explanation:
It would be incorrect to refer to them as the first, second and third Round Table
Conference. Because, it was a Conference held in three sessions and not of
three separate Conferences. It was a Conference held to discuss Simon
Commission's Report, as the first and second sessions were failed and third
session was inevitable.
Who among the following was a prominent leader of the Congress Socialist
Party ?
(a) M. N. Roy
(b) Ganesh Shankar Vidyarthi
(c) Pattam Thanu Pillai
(d) Acharya Narendra Dev
Ans. (d)
Explanation:
Acharya Narendra Dev, Minu Masani and Jai Prakash Narayan were the
pioneers of Congress Socialist Party, which was formed as the Civil
Disobedience Movement could not bring any relief to the peasants and
congress did not provide a concrete social and economic programme. It was
formed in 1934.
Consider, the following landmarks in Indian education:
1. Hindu College, Calcutta
2. University of Calcutta
3. Adams Report
4. Woods Despatch
The correct chronological order of these landmarks is :
(a) 1, 3, 4, 2
(b) 1, 4, 3, 2
(c)3, 1, 4, 2
(d) 3, 2, 4, 1
Ans. (a)
Explanation:
Hindu College, Calcutta was established in 1817, three Adam's 'reports on
education in Bengal and Bihar in 1835, 1835, and in 1838. Wood's dispatch by
Charles Wood in 1854, Calcutta university was established in 1857.
Match List I with List I! and select the correct answer using the codes given
below the lists:
List I
List II
A. Surendranath Bannerjee
1. Hind Swaraj
B. M. K. Gandhi
2. The Indian Struggle
C. Subhash Chandra Bose
3. Autobiographical Writings
D. LajpatRai
4. A Nation in Making
A B C D
A B C D
(a) 4 1 3 2
(b) 1 4 3 2
(c) 4 1 2 3
(d) 1 4 2 3

33

26.

27.

28.

29.

Ans. (d)
Explanation:
Surendranath Banerjee wrote "Hind Swaraj". M.K. Gandhi wrote "A nation in
Making". Subhash Chandra Bose wrote 'The Indian Struggle". Lajpat Rai wrote
Auto Biographical Writings.
Consider the following statements about Jawaharlal Nehru .
1. He was the president of the Congress Party in 1947.
2. He presided over the Constituent Assembly.
3. He formed the first Congress Ministry in United Province before Indias
Independence
Of these statements :
(a) 1, 2 and 3 are correct
(b) 1 and 3 are correct
(c) 1 and 2 are correct
(d) none is correct
Ans. (d)
Explanation:
Upto 1946, Maulana Abul Kalam Azad was the President and in 1947 J. B,
Krlpalani was the President. Dr. Babu Rajendra Prasad presided over the
Constituent Assembly.
It was Govind Ballabh Panth who formed the first Congress Ministry in United
Province before Independence.
Which one of the following is not correct about the Cabinet Mission Plan ?
(a) Provincial grouping
(b) Interim Cabinet of Indians
(c) Acceptance of Pakistan
(d) Constitution framing right
Arts. (c)
Explanation:
Cabinet Mission Plan rejected the Pakistan demand and stood for United India.
Along with that, it proposed for the right of Constituent Assembly, provincial
groupings, interim government, etc.
Match List 1 with List II and select the correct answer using the codes given
below the lists
List I
List II
A. Abhinav Bharat Society
1. Sri Aurobindo Ghosh
B. Anushiian Samiti
2. Lala Hardayal
C. Ghadar Party
3. C. R. Das
D. Swaraj Parly
4. V. D. Savarkar
A B C D
A B C D
(a) 4 1 3 2
(b) 1 4 3 2
(c) 1 4 2 3
(d) 4 1 2 3
Ans. (d)
Explanation:
Abhinav Bharat Society founded by V.D. Savarkar. Sri Aurobindo Ghosh was
associated with Anushiian Samithi. Ghadar Party in San fransisco founded by
Lala Hardayal. Swaraj Party was founded by Motilal Nehru and C.R. Das.
Match List I with List II and select the correct answer using the codes given
below the lists:
List I (Period)
List II (Event)
A. 1883
1. Announcement of Communal Award from Whitehall
B. 1906
2. Formation, of the All India State People's Conference
C. 1927
3. Foundation of Muslim League at Dacca
D. 1932
4. First session of National Conference at Calcutta

34

30.

31.

32.

33.

A B C D
A B C D
(a) 4 3 1 2
(b) 3 4 1 2
(c) 4 3 2 1
(d) 3 4 2 1
Ans. (c)
Explanation:
First Session of National Conference at Calcutta in 1883. Foundation of Muslim
League at Dacca in 1906. Formation of All India State People's Conference in
1927. Announcement of Communal Award by Ramsay McDonald from
Whitehall in 1932.
Consider the following statements :
The Non-Cooperation Movement led to the
1. Congress becoming a mass movement for the first time.
2. growth of Hindu-Muslim unity.
3. removal of fear of the British might from the minds of the people.
4. British Governments willingness to grant political concessions to Indians.
Of these statements :
(a) 1, 2, 3 and 4 are correct
(b) 2 and 3 are correct
(c) 1 and 3 are correct (d)
3 and 4 are correct
Ans. (b)
Explanation:
The Non-Cooperation Movement launched in August, 1920 by Mahatma
Gandhi, saw for the first time, Hindu Muslim Unity in a big way and removed the
fear of the British "might" from the minds of the people.
Which one of the following is an important historical novel written during the
latter half of the nineteenth century ?
(a) Rast Goftar
(b) Durgesh Nandini (c) Mahratha
(d)
Nibandhamala
Ans. (b)
Explanation:
Durgesh Nandini, Banga Darshan and Anand Math were the great Historicai
novels written by Bankim Chandra Chatterjee.
Who among the following suggested the winding up of the Indian National
Congress after India attained independence?
(a) C. Rajagopaiachari
(b) Acharya Kripalani
(c) Mahatma Gandhi
(d) Jayaprakash Narayan
Ans. (c)
Explanation:
Mahatma Gandhi suggested the winding up of the INC as its purpose
achievement of Indian Independence, had been fulfilled. He proposed it to be a
voluntary oganisetion to serve the country after Independence.
Match List I with List I! and select the correct answer using the codes given
below the lists:
List I
List II
A. Butler Committee Report
1. Jallianwala Bagh massacre
B. Hartog Committee Report
2. Relationship between the Indian
States and
Paramount Power
C. Hunter Inquiry Committee Report
3. Working of Dyarchy as laid down
in the
Montague Chelmsford Reforms

35

D. Muddiman Committee Report


India &

4. The growth of education in British


potentialities

of

its

further

progress
A B C D
A B C D
(a) 3 2 1 4
(b) 1 4 2 3
(c) 2 1 3 4
(d) 2 4 1 3
Ans. (d)
Explanation:
Butler Committee was appointed in 1927 to give its report on relationship
between the Indian State and paramount power. Hartog Committee was
appointed in 1929 to give its report on the growth of education in British India
and potentialities of its further progress. Hunter Commission Report was on
Jallianwala Bagh Massacre in 1919 and Muddiman Committee Report on
working of Dyarchy as laid down in the Montague Chelmsford Reforms.
34. A graduate at 18, professor and associate editor of the Sudharak at 20,
Secretary of the Sarvajanik Sabha and of the Provincial Conference at 25,
Secretary of the National Congress at 29, leading witness before an important
Royal Commission at 31, Provincial legislator at 34, Imperial legislator at 36,
President of the Indian National Congress at 39 ....... a patriot whom Mahatma
Gandhi himself regarded as his master. This is how a biographer describes
(a) Pandit Madan Mohan Malaviya
(b) Mahadev Govind Ranade
(c) Gopal Krishna Gokhale
(d) Bal Gangadhar Tilak
Ans. (c)
Explanation:
This statement is about Gopal Krishna Gokhale. He was born in Kolhapur in
1866, graduated from Elphinston Coliege and appointed professor of
Fergueson College in Poona. Gandhiji himself regarded Gokhale as his Political
Master.
35. Who among the following was associated with supression of Thugs ?
(a) General Henry Prendergast
(b) Captain Sleeman
(c) Alexander Burnes
(d) Captain Robert Pemberton
Ans. (b)
Explanation:
Lord William Bentinck appointed Captain Sleeman for suppression of thugs. By
1837, thugs were suppressed by his effort.
36. Match List I with List II and select the correct answer using the codes given
below the lists:
List 1 (Events)
List 11 (Results)
A. Morley Minto Reforms
1. Country-wise agitation
B. Simon Commission
2. Withdrawl of a movement
C. The Chauri-Chaura incident
3. Communal Electorates
D. The Dandi March
4. Communal outbreaks
5. Illegal manufacture of salt
A B C D
A B C D
(a) 3 4 5 2
(b) 4 1 2 3
(c) 2 3 4 5
(d) 3 1 2 5
Ans. (d)
Explanation:

36

Minto Morley Reforms (1909) introduced separate electorates for Muslims, this
is called Communal electorate which ultimately led to the Division of India.
Simon Commission (1927) has no Indian member in it. So, it was rejected by
Indians and resulted in Country-wide agitation.
Chauri-Chaura incident (1922) resulted in withdrawal of Non-Cooperation
Movement.
In Dandi March (1930), by illegal manufacture of Salt, Gandhiji started Civil
Disobedience Movement.
37. The Poona Pact which was signed between the British Government and
Mahatma Gandhi in 1934 provided for
(a) the creation of dominion status for India (b) separate electorates for the
Muslims
(c) separate electorates for the Harijans (d) joint electorate with reservation of
Harijans
Ans. (d)
Explanation:
When Mahatma Gandhi protested against the Communal Award and went on a
fast unto death in the Yeravada Jail on 20 September 1932. Then Poona Pact
came into existence in 1932. Accordingly, in place of 71 seats as approved in
the Communal Award, 148 seats in different Provincial legislatures were
reserved for the depressed classes.
38. Match List I with List II and select the correct answer using the codes given
below the lists:
List I
List I!
A. Surat Split
1. 1929
B. Communal Award
2. 1928
C. All Party Convention
3. 1932
D. Pooma Swaraj Resolution
4. 1907
5. 1905
A B C D
A B C D
(a) 4 3 1 5
(b)
4 3 2 1
(c) 2 5 4 1
(d)
1 4 2 3
Ans. (b)
Explanation:
The famous Surat Split between Moderates and Extremists in the Congress
occurred in 1907. Communal Award was announced by Ramsay MacDonald
separate electorates for depressed classes in 1932. All party convention was
convened in which 29 organisations took part, to draw up a blue print for the
future Constitution of India in 1928. Poorna Swaraj was declared in Lahore
Session of Congress in December 1929.
39. Match List I with List II and select the correct answer using the codes given
below the lists:
List I
List II
A. Chittagong Armoury raid
I. Lala Hardayal
B. K. a icon Conspiracy
2. jatm uss
C. Lahore Conspiracy
3. Surya Sen
D. Ghadar Party
4. Ram Prasad Bismil
5. Vasudeo Phadke
A B C D
A B C D
(a) 3 4 1 5
(b) 4 3 2 5

37

40.

41.

42.

43.

(c) 3 4 2 1
(d) 2 4 3 1
Ans. (c)
Explanation:
Chittagong Armoury Raid took place in Bengal under the leadership of
Suryasen on the Government armouries in 1930. In 1925, at Kakori Railway
Station, a Shaharanpur-Lucknow train was looted under the leadership of
Ramprasad Bismil. Jatindas
became the first martyr on the 64th
day of his fast, who was tried in Lahore conspiracy case along with Bhagat
Singh, Rajguru and Sukhdev, in April 1929. Ghadar Party was established by
Lala Haradayal in 1913 at San-fransisco.
M. C. Setalwad, B. N. Rao and Alladi Krishnaswamy Iyer were distinguished
members of the
(a) Swaraj Party
(b) All
India
National
Liberal
Federation
(c) Madras Labour Union
(d)
Servants
of
India Society
Ans. (d)
Explanation:
Servants of India Society was founded by Gopal Krishna Gokhale in which M.C.
Setalwad, B.N. Rao, Alladi Krishna Swamv Iyer were distinguished members.
What is the correct chronological sequence of the following ?
1. Woods Education Despatch
2. Macaulays minute on education
3. The Sargent Education Report
4. Indian
Education
(Hunter
Commission)
(a) 2, 1,4, 3
(b) 2, 1,3,4
(c) 1, 2, 4, 3
(d) 4, 3, 1, 2
Ans, (a)
Explanation:
Macaulay's minute on education in 1835. Charles Wood's despatch in 1854.
Hunter Commission on Indian Education in 1882. Sargent's Plan on Education
in 1944.
Match List I with List II and select the correct answer using the codes given
below the lists:
List I
List II
A. Moplah revolt
1. Kerala
B. Pabna revolt
2. Bihar
C. Eka Movement
3. Bengal
D. Birsa Munda revolt
4. Awadh
A B C D
A B C D
(a) 1 3 4 2
(b) 2 3 4 1
(c) 1 2 3 4
(d) 3 4 1 2
Ans. (a)
Explanation:
Mopiah Revolt occurred in Malabar region of Kerala in 1921. Pabna revolt
occured in Bengal from 1831 to 1876. Ekka movement in 1921 occurred in
Awadh region. Munda rebellion under the leadership of Birsa Munda in Bihar
(now Jharkhand region) occurred in 1899.
1998
The educated middle class in India
(a) opposed the revolt of 1857
(b) supported the revolt of 1857
(c) remained neutral to the revolt of 1857 (d) fought against native rulers

38

44.

45.

45.

47.

48.

Ans. (c)
Explanation:
The educated middle class remained neutral to the revolt of 1857, as they were
English-Educated.
Lord Mountbatten came to India as Viceroy along with specific instruction to
(a) balkanize the Indian sub-continent
(b) keep India united if possible
(c) accept Jinnah's demand for Pakistan (d) persuade the Congress to accept
partition
Ans. (b)
Expiana tion:
Lord Mountbatten, the last Viceroy of India appointed after Independence with a
specific instruction to keep India united if possible. But he didn' find that
possibility.
Directions : The following items consist of two statements, one labelled as the
Assertion (A) and the other as Reason (R). You are to examine these two
statements carefully and select the answers to these items using the code
given below :
(a) Both A and R are true and R is the correct explanation of A
(b) Both A and R are true but R is NOT the correct explanation of A
(c) A is true but R is false
(d) A is false but R is true
Assertion A : The Khilafat movement did bring the urban Muslims into the fold
of the National Movement.
Reason R : There was a predominant elements of anti-imperialism in both the
National and Khilafat Movement.
Ans. (a)
Explanation:
The prominent element of anti-imperialism was the main reason to the urban
muslims to come into the fold of National movement.
Assertion A : Partition of Bengal in 1905 brought to an end the Moderates'
role in the Indian freedom movement.
Reason R : The Surat session of Indian National Congress separated the
Extremists from the Moderates.
Ans. (d)
Explanation:
Even after partition of Bengal in 1905, moderates continued to participate in the
freedom movement, later in 1916 at Lahore Session, both moderates and
extremists were reunited.
Assertion A : The first ever Bill to make primary education compulsory in India
was rejected in 1911.
Reason R : Discontent would have increased if every cultivator could read.
Ans. (a)
Explanation:
British Government rejected the first Bill to make primary education compulsory
in India, as it had feared that would spread the discontentment all over the
country.
Assertion A : The Congress rejected the Cripps proposals.
Reason R : The Cripps Mission consisted solely of whites.
Ans. (b)
Explanation:

39

49.

50.

51.

52.

53.

Cripps proposals promised Dominion Status for Inia. But it was the situation
when Congress was demanding the complete independence. So they rejected
the proposals. The Cripps Mission consisted solely of whites.
Assertion A : Gandhi stopped the Non-Co-operation Movement in 1922.
Reason R : Violence at Chauri Chaura led him to stop the movement.
Ans. (a)
Explanation:
On 5th February, 1922 an agitated crowd gheraoed the police station at ChauriChaura in the Gorakhpur district of Uttar Pradesh and, set fire to it, in which 22
policemen died. Seeing it turning violent Mahatma Gandhi announced the
suspension of the movement.
When the Indian Muslim League was inducted into the interim government in
1946, Liyaqat AM Khan was assigned the portfolio of:
(a) Foreign affairs
(b) Home
(c) Finance
(d) Defence
Ans. (c)
Explanation:
In October 1946,. upon repeated persuasion, Muslim League joined the Interim
Government, in which Liyaqat Ali Khan was assigned to the port folio of
Finance.
The Indian National Congress agreed in 1947 to the partition of the country
mainly because
(a) the principle of two-Nation theory was then acceptable to them
(b) it was imposed by the British Government and the Congress was helpless in
this regard
(c) they wanted to avoid large-scalec ommunal riots
(d) india would have otherwise lost the opportunity to attain freedom
Ans. (c)
Explanation:
Indian National Congress agreed in 1947 to the partition of the country because
they wanted to avoid large-scale communal riots, which took lives of lakhs of
people, for the past several years. Because of the rise of Communication, the
communal bonafire was spreading very fast. To check this our national leaders
were forced to agree the partition.
At the time of India's Independence, Mahatama Gandhi was :
(a) a member of Congress Working Committee
(b) not a member of the Congress
(c) the President of the Congress
(d) the General Secretary of the
Congress
Ans. (b)
Explanation:
At the time of Indias Independence, Mahatma Gandhi was not a member of
Congress.
What is the correct sequence of the following events ?
1. Tilak's Home Rule League.
2. Komagatamaru Incident.
3. Mahatama Gandhi's arrival in India.
Select the correct answer using the codes given below the lists:
(a) 1, 2, 3
(b) 3, 2, 1
(e) 2, 1, 3
(d) 2, 3, 1
Ans. (d)
Explanation:

40

54.

55.

56.

57.

58.

In 1913, the ship Komagatamaru's incident occurred, Mahatma Gandhi arrived


India from South Africa in 1915, Bal Gangadhar Tilak started Home Rule
League in Poona on 23rd April 1916.
Simon Commission of 1927 was boycotted because
(a) there was no Indian Member in the Commission
(b) it supported the Muslim League
(c) Congress felt that the people of India are entitled to Swaraj
(d) there were differences among the members
Ans. (a)
Explanation:
On the fateful day of 3 February 1928 when Simon Commissioin (1927)
reached Bombay, it was greeted with biack flags and cries of "Simon Go back"
all over the counry. As there was no Indian member in it, almost ail the political
parties opposed the Commission.
The Indian Muslims, in general, were not attracted to the Extremist movement
because of the
(a) influence of the Sir Sayyid Ahmed Khan (b)
anti-Muslim
attitude of Extremist leaders
(c) indifference shown to Muslim aspirations (d)
Extremists'
policy of harping on Hindu past
Ans. (d)
Explanation:
In general, the Indian Muslims were not attracted to the Extremist movement
because of the extremists policy of harping on Hindu past like Shivaji Jayanti
and Ganesh-festivais organised by Tilak, Holy Dip in River Ganga during
Swadeshi movement etc.
Which one of the following events, was characterised by Montague as
'Preventive Murder'?
(a) Killing of INA activists
(b)
Massacre
of
Jallianwalia Bagh
(c) Shooting of the Mahatma
(d) Shooting of Curzon-Wythe
Ans. (b)
Explanation:
Jallianwala Bagh Massacre of April 13, 1919 by General Dyer was called as
"Preventive Murder" by Montague.
What is the correct sequence of the following events ?
1. The August offer.
2. The I.N.A. trial.
3. The Quit India Movement.
4. The Royal Indian Naval Ratings
Revolt.
Select the correct answer using the codes given below:
(a) 1, 3, 2, 4
(b) 3, 1, 2, 4
(c) 1, 3, 4, 2
(d) 3, 1, 4, 2
Ans. (a)
Explanation:
The 'August Offer' which promised dominion status came on 8th August, 1940.
Quit India Movement was launched in the CWC meeting at Wardha on 14 July,
1942. The I.N.A. trial began in 1945. The Royal Indian Naval Rating's Revolt in
February, 1946.
Which one of the following defines extremist ideology during the early phase of
Indian freedom movement?

41

59.

60.

61.

62.

(a) Stimulating the production of indigenous articles by giving them preference


over imported commodities
(b) Obtaining self-government by aggressive means in place of petitions and
constitutional ways
(c) Providing national education according to the requirements of the countiy
(d) Organising coups against the British empire through military revolt
Ans. (b)
Explanation:
The extremist groups of the early phase of Indian Freedom movement had
believed in obtaining self-government by aggressive means in place of petitions
and constitutional ways.
Which of the following pairs are correctly matched ?
1. Theodore Beck : Mohammadan Anglo Oriental College, Aligarh
2. llbert Bill : Ripon
3. Pherozeshah Mehta : Indian National Congress.
4. Badruddin Tyabji : Muslim League
Select the correct answer using the codes given below :
(a) 1, 2, 3 and 4
(b) 2 and 4
(c) I, 3 and 4
(d) 1, 2 and 3
Ans. (d)
Explanation:
Theodore Beck was the first principal of Mohammedan Anglo-Oriental College,
Aligarh, which was founded in 1875 by Sir Sayyid Ahmad Khan. During Ripon
period, in 1883, Ilbert bill was passed. Pherozeshah Mehta was one of the
founder members of Indian National Congress, established in 1885. Sir
Badruddin Tyabji was not associated with Muslim League. He was the first
Muslim President of Indian National Congress in 1887.
"The Congress is tottering to its fail and one of my great ambitions while in
India, is to assist It to a peaceful demise." This statement is attributed to
(a) Lord Dufferin
(b) Lord Curzon
(c) Lord Lytton
(d) None of the
above
Ans. (b)
Explanation:
When there was inner fight going on between moderates and extremists in the
Congress, Lord Curzon made that statement.
Who was the leader of the Ghadar Party ?
(a) Bhagat Singh
(b) Lala Hardayai
(c) Bal Gangadhar Tilak
(d)V.D.
Savarkar
Ans. (b)
Explanation:
In 1913, the Ghadar Party was established and set up a headquarter called
"Yugantar Ashram" in San Fransisco by Lala Hardayai. It started a weekly
newspaper "The Ghadar" for free circulation.
1999
The first venture of Gandhi in all-India politics was the :
(a) Non-Cooperation Movement
(b) Rowlatt Satyagraha
(c) Champaran Movement
(d) Dandi March
Ans. (b)
Explanation:
Though Gandhiji began his experiments with Satyagraha in Champaran-Bihar
(in 1917), Kheda - Gujarat (in 1918), Ahmedabad (in 1918), they were localised

42

63.

64.

65.

66.

67.

and confined to that regions only. But, the first all-India level movement
launched by him was the Satyagraha against the Rowlatt Act (March 1919).
Later Non-Cooperation Movement in 1920-22 and Dandi Satyagraha in 193034 were organised by him.
The Congress policy of pray and petition ultimately came to an end under the
guidance of
(a) Aurobindo Ghosh
(b) Bal Gangadhar Tilak
(c) Lala Lajpat Rai
(d) Mahatma Gandhi
Ans. (b)
Explanation:
Under the guidance of Bal Gangadhar Tilak, the extremist leader, came to an
end the Congress policy of pray and petition.
Match List ! with List II and select the correct answer using the codes given
below the lists:
List I (Persons)
List II (Journals)
A. Shyamji Krishna Varma
1. Bande Mataram
B. Madam Bhikaji Cama
2. Indian Sociologist
C. Annie Besant
3. The Talwar
D. Aurobindo Ghosh
4. Commonwealth
A B C D
A B C D
(a) 2 3 4 1
(b) 3 2 1 4
(c) 2 3 1 4
(d) 3 2 4 1
Ans. (a)
Explanation:
Shyamji Krishna Varma who founded 'India House' in London, also associated
with the journal 'Indian Sociologist1, Madam Cama - The Talwar, Annie Besant
- Commonwealth, and Aurobindo was associated with Bande Mataram.
There was no independent development of industries in India during British rule
because of the
(a) absence of heavy industries
(b) scarcity of foreign capital
(c) scarcity of natural resources
(d) preference of the rich to invest in
land
Ans. (a)
Explanation:
During British rule, due to the absence of heavy industries, independent
development of industries was not taken place. That is why, our leaders
focussed on heavy industries after the Independence.
'Abhinava Bharat a secret society of revolutionaries was organised by
(a) Khudiram Bosf
(b) V.D. Savarkar
(c) Prafulla Chaki
(d) Bhagat
Singh
Ans. (b)
Explanation:
"Abhinava Bharat" was a secret society of revolutionaries organised by Vinayak
Damodar Savarkar. It was founded by him in 1904.
The most short-lived of all of Britain's constitutional experiments in India was
the
(a) Indian Councils Act of 1861
(b) Indian Councils Act of 1892
(c) Indian Councils Act of 1909
(d) Government of India Act of 1919
Ans. (c)
Explanation:

43

68.

69.

70.

71.

72.

(In the given optionals) Indian Councils Act of 1909 or famously Minto-Morley
Reforms was the most short lived as it lasted, only for 10 years, upto the 1919
act or Montague-Chelmsford Reforms.
"It made its proposals in May. It still wanted a united India. There was to be a
Federal Union composed of British provinces."
The above quotation is related to
(a) Simon Commission
(b) Gandhi-lrwin Pact
(c) Cripps Mission
(d) Cabinet Mission
Ans. (d)
Explanation :
The Cabinet Mission Proposals came in May 1946, which opposed the demand
of Pakistan and supported United India composed of British provinces.
The Governor-General who followed a spirited "Forward" policy towards
Afghanistan was
(a) Minto
(b) Dufferin
(c) Elgin
(d) Lytton
Ans. (d)
Explanation :
It was Lord Ellenborough who followed a 'forward' policy towards Afghanistan to
regain the lost prestige of the English forces that had been defeated in first
Anglo-Afghan War (1839-1842). But in the given options, It was Lytton who did
so.
At a time when empires in Europe were crumbling before the might of Napoleon
which one of the following Governors-General kept the British flag flying high in
India ?
(a) Warren Hastings (b) Lord Cornwallis (c) Lord Wellesley
(d)
Lord
Hastings
Ans. (d)
Explanation:
It was Lord Hastings, through Anglo-Nepalese War (1814-16), suppression of
the Pindaris and the end of the Maratha Confederacy, kept the British flag flying
high in India, when empires in Europe were crumbling before the might of
Napolean.
Which Indian nationalist leader looked upon a war between Germany and
Britain as a godsent opportunity which would enable Indians to exploit the
situation to their advantage?
(a)G. Rajagopalachari
(b) M.A. Jinnah
(c) Subhash Chandra Bose
(d) Jawaharlal Nehru
Ans. (c)
Explanation:
Subhash Chandra Bose was eager to take advantage of the situation of
hovering war clouds in Europe, for the country's independence. He went to
Germany and 3apan to organise Indian National Army (INA) to overthrow the
British rule in India.
Which one of the following leaders of the Congress was totally In favour of
Cabinet Mission plan ?
(a) Mahatma Gandhi
(b) jawaharlal Nehru
(c) Sardar Patel
(d) Maulana Abul Kalam Azad
Ans. (c)
Explanation:
Sardar Patel of the Congress was totally in faovur of CMP.

44

73. Which one of the following Indian leaders was dismissed by the British from the
Indian Civil Service ?
(a) Satyendranath Tagore
(b) Surendranath Banerji
(c) R.C. Dutt
(d) S. C. Bose
Ans. (b)
Explanation:
Surendranath Banerjee was dismissed by the British from Indian Civil Services
on the false charge as they can't withstand an Indian at equal or high post.
74. Consider the following events :
1. Indigo Revolt
2. Santhal Rebellion
3. Deccan Riot
4. Mutiny of the Sepoys
The correct chronological sequence of these events is
(a) 4, 2, 1,3
(b) 4, 2, 3, 1
(c)2,4,3,l
(d) 2, 4, 1, 3
Ans. (a)
Explanation:
Indigo Revolt against British planters occurred in Bengal in 1860, Santal
rebellion in Raj Mahal hills, under leadership of Sidho and Kanho occurred in
1855-56, and Deccan riot in 1864 at Baliore, Sepoy mutiny or 1857 revolt in
1857.
75. Match List I with List II and select the correct answer using the codes given
below the lists:
List I (Years)
List II (Events)
A. 1775
1. First Angio-Burmese War
B. 1780
2. First Anglo-Afghan War
C. 1824
3. First Anglo-Maratha War
D. 1838
4. Second Anglo-Mysore War
A B C D
A B C D
(a) 4 3 2 1
(b) 4 3 1 2
(c) 3 4 1 2
(d) 3 4 2 1
Ans. (c)
Explanation:
First Anglo-Burmese war was fought in 1824.
First Anglo-Afghan war was fought in 1838-42.
First Anglo-Maratha war was fought in 1775-82.
Second Anglo-Mysore war was fought in 1780-84.
76. The term "imperial preference" was applied to the
(a) special privileges on British imports in India
(b) racial discrimination by the Britishers
(c) subordination of Indian interest to that of the British
(d) preference given to British political agents over Indian Princes
Ans. (d)
Explanation:
The term imperial preference' was applied to the preference given to British
political agents over Indian princes.
Directions : The following items consist of two statements, one labelled as the
Assertion (A) and the other as Reason (R). You are to examine these two
statements carefully and select the answers to these items using the code
given below :
(a) Both A and R are true and R is the correct explanation of A
(b) Both A and R are true but R is NOT the correct explanation of A

45

77.

78.

79.

80.

81.

82.

(c) A is true but R is false


(d) A is false but R is true
Assertion (A) : Lord Linlithgow described the August Movement of 1942 as
the most serious rebellion since Sepoy Mutiny.
Reason (R) : There was massive upsurge of the peasantry in certain areas.
Ans. (a)
Explanation:
The Quit India Movement which was started on 8 August, 1942 had,
unprecedent involvement of peasant community which made the struggle more
fierce, i hus Lord Linlithgow described this movement as the most serious
rebellion since Sepoy Mutiny.
2000
"In this instance we could not pfay off the Mohammedans against the Hindus" .
To which one of the following events did this remark of Aitchison relate ?
(a) Revolt of 1857
(b) Champaran Satyagraha (1917)
(c) Khilafat & Non-Cooperation Movement (1919 -22) (d) August Movement of
1942
Ana. (c)
Explanation:
During the Khilafat and Non-Cooperation Movement period, Muslims joined
with Hindus to struggle against the common enemy British. This period had
seen unprecedented unity between these two religions. This forced Aitchison to
make the above statement.
The Indian National Army (I.N.A.) came into existence in 1943 in
(a) Japan
(b) then Burma
(c) Singapore
(d) then Malaya
Ans. (c)
Explanation:
Though INA was formed by Captain Mohan Singh, on 1 September 1942, it
was failed and came into existence again on 2 July 1943, when Subhash
Chandra Bose reached Singapore and gave the rousing war cry of "Dilli Chalo".
The last major extension of British Indian territory took place during the time of
(a) Dufferin
(b) Dalhousie
(c) Lytton
(d) Curzon
Ans. (b)
Explanation:
Dalhousie (1849-56) introduced the 'Doctrine of Lapse' which proved fatal for
the princely states. Based on this, he annexed a lot of States and expanded the
British Indian territory to a large extent.
As an alternative to the partition of India, Gandhiji suggested to Mountbatten
that he
(a) postpone granting of independence
(b) invite Jinnah to form the
government
(c) invite Nehru and Jinnah to form the government together
(d) invite the army to take over for some time
Ans. (b)
Explanation :
Gandhiji, as an alternative to the partition of India, suggested to Mountbatten to
invite Jinnah to form the Government. But it was opposed by Congress as well
as Muslim League.
The native state 'of Tripura became involved ^ in the Freedom movement early
in the 20th century because
(a) the kings of Tripura were always anti-British

46

83.

84.

85.

86.

(b) the Bengal revolutionaries took shelter in Tripura


(c) the tribes of the state were fiercely freedom loving
(d) there were already some groups fighting against the kingship and its
protector, the British
Ans. (d)
Explanation:
Tripura became involved in the freedom struggle early in the 20th century
because there was already some groups (Bengal revolutionary armies) who
were fighting against the Kingship and its protector, the British.
That the per capital income in India was Rs. 20/- in 1867-68, was ascertained
for the first time by
(a) M. G. Ranade
(b) Sir W. Hunter
(c) R. C. Dutta
(d) Dadabhai
Naoroji
Ans. (d)
Explanation:
For the first time in 1867-68, Dada Bhai Naoroji had ascertained the per capita
income, in his Book "poverty and un-British rule in India", as Rs. 20/-.
After returning from South Africa, Gandhiji launched his first successful
Satyagraha in
(a) Chauri-Chaura
(b) Dandi
(c) Champaran
(d) Bardoli
Ans. (c)
Explanation:
In 1915, Gandhiji returned from South Africa, and in 1917, at Champaran
(Bihar) he experimented the Satyagraha to alleviate the problems of indigo
cultivators.
Match List I with List II and select the correct answer using the codes given
below the lists:
List I
List II
A. Chittagong Armoury Raid
1. Kalpana Dutt
B. Abhinav Bharat
2. Guru Ram Singh
C. Anushiian Samiti
3. Vinayak Damodar Savarkar
D. Kuka Movement
4. Aurobindo Ghosh
A B C D
A B C D
(a) 1 3 4 2
(b) 1 3 2 4
(c) 3 1 2 4
(d) 3 1 4 2
Ans. (a)
Explanation:
Kalpana Dutt and Preetl Lata Waddedar were associated with Chittagong
Armoury Raid in 1930, under the leadership of Surya Sen. Abhinav Bharat was
a revolutionary organisation founded by V.D. Savarkar in 1904, Aurobindo
Ghosh was associated with Anushiian Samiti, which was founded by Sisir
Kumar Ghosh in
Bengal. Kuka Rebellious movement was organised by
Ram Singh in Punjab in 1863.
Match List I with List II and select the correct answer using the codes given
below the lists:
List I
List II
A. Land allotted to big feudal landlords
1. Jagirdari System
B. Land allotted to revenue farmers or
2. Ryotwari System
rent collectors
C. Land allotted to each peasant with
3. Mahalwari

47

the right to sublet, mortgage, gift or sell


D. Revenue settlements made
4. Zamindari System
at village level
A B C D
A B C D
(a) 1 3 2 4
(b) 1 4 2 3
(c) 3 4 1 2
(d) 2 1 3 4
Ans. (b)
Explanation:
The system in which Land alloted to a big landlord was called Jagirdari System,
(jagirdar = landlord). The system in which land alloted to revenue farmers of
rent collectors (Zamindars) was Zamindari System. The land allotted to each
peasant with the right to sublet, mortgage transfer, gift or sell was called as
Ryotwari System, as individual farmer had the owning rights. (Mahal = Village)
If revenue settlement was made at village level, then it was called as Mahalwari
System.
Directions : The following items consist of two statements, one labelled as the
Assertion (A) and the other as Reason (R). You are to examine these two
statements carefully and select the answers to these items using the code
given below :
(a) Both A and R are true and R is the correct explanation of A
(b) Both A and K are true but K is NOT the correct explanation of A
(c) A is true but R is false
(d) A is false but R is true
87. Assertion (A) : Lord Linlithgow described the August Movement of 1942 as
the most serious revolt after the Sepoy mutiny.
Reason (R) : Peasants joined the movement in large number in some places.
Ans. (a)
Explanation:
The Quit India Movement which was started on 8 August, 1942 had,
unprecedent involvement of peasant community which made the struggle more
fierce. Thus Lord Linlithgow described this movement as the most serious
rebellion since Sepoy Mutiny.
88. Assertion (A) : The basic weakness of the early nationalist movement lay in
its narrow social base.
Reason (R) : It fought for the narrow interests of the social groups which
joined it.
Ans. (c)
Explanation:
Early nationalists fought for the rights of the common man also. The reason for
narrow social base of the early Nationalist movement was, the organization in
those starting days was weak and at any point the British could crush it. So,
leaders confined to mere prayers and petitions.
89. While delivering the presidential address, the Congress President who
advocated the introduction of Roman script for Hindi language was
(a) Mahatma Gandhi
(b) Jawaharlal Nehru
(c) Abul Kalam Azad
(d) Subhash Chandra Bose
Ans. (a)
Explanation:
It was Subhash Chandra Bose during his Presidential address in 1938,
advocated the introduction of Roman Script for Hindi Language.

48

90. At the time of partition of India, which one of the following provinces of British
India came forward with a plan for a united and independent existence ?
(a) Punjab
(b) Assam
(c) Bengal
(d) Bihar
Ans. (a)
Explanation:
During the Partition of India, Punjab province came forward with a plan for a
United and Independent existence. But due to Sardar Vallabh Bhai Patel's effot,
Patiala and East Punjab States Union (PEPSU) became a part of Indian Union.
91. The Balkan Plan for fragmentation of India was the brain-child of
(a) W. Churchill
(b) M. A. Jinnah
(c) Lord Mountbatten (d)
V.
P.
Menon
Ans. (c)
Explanation:
British Government authorised Lord Mountbatten to resolve the situation as
early as possible. It is he who felt that the situation had deteriorated so much,
that the creation of Pakistan appeared inevitable.
92. Consider the following statements about the Indian National Congress
1. Sarojini Naidu was the first woman to be the President of the Congress
2. C. R. Das was in prison when he functioned as the President of the
Congress
3. The first Britisher to become the President of the Congress was Alan
Octavian Hume
4. Alfred Webb was the President of the Congress in 1894
Which of these statements are correct ?
(a) 1 and 3
(b) 2 and 4
(c) 2, 3 and 4
(d) 1, 2, 3 and 4
Ans. (b)
Explanation:
Sarojini Naidu was second woman to preside over the Congress in 1925, first
woman was Annie Besant in 1917, C. R. Das presided over Congress from
Prison in 1921. A. 0. Hume was the founder of Indian National Congress. But
he never presided over the Congress. Sir Alfred Webb was the President of
Madras Session of Congress in 1894.
93. Which one of the following is NOT a feature . of the Government of India Act of
1935 ?
(a) Diarchy at the Centre as well as in the provinces
(b) A bicameral legislature
(c) Provincial autonomy
(d) An All-India Federation
Ans. (a)
Explanation:
Diarchy was introduced at the centre but not in the provinces. Diarchy was
introduced in the provinces through GOI act, 1919 but it was abolished by the
GOI act, 1935.
2001
94. Who among the following Indian rulers established embassies in foreign
countries on modern lines ?
(a) Haider Ali
(b) Mir Qasim
(c) Shah Alam II
(d) Tipu Sultan
Ans. (d)
Explanation:
Tipu Sultan established embassies in foreign countries like Egypt, France and
Turkey on modern lines.

49

95. The Hunter Commission was appointed after the


(b) Black-hole incident
(b) Jalianwallabagh massacre
(c) Uprising of 1857
(d) Partition of Bengal
Ans. (b)
Explanation:
To investigate into the events of Jallianwalabagh tragedy, occurred in April 13,
1919, the Hunter Commission was appointed.
96. Under the Permanent Settlement, 1793, the Zamindars were required to issue
pattas to the farmers which were not issued by many of the Zamindars. The
reason was
(a) The Zamindars were trusted by the farmers
(b) There was no official check upon the Zamindars
(c) It was the responsibility of the British government
(d) The farmers were not interested in getting pattasAns. (b)
Explanation:
There was no official check upon the Zamindars under the permanent
settlement, 1793. They were required to give 10/llth of revenue collections to
the British authorities.
97. Consider the following statements :
1. Arya Samaj was founded in 1835.
2. Lala Lajpat Rai opposed the appeal of Arya Samaj to the authority of Vedas
in support of its social reform programmes.
3. Under Keshab Chandra Sen, the Brahmo Samaj campaigned for women's
education.
4. Vinoba Bhave founded the Sarvodaya Samaj to work among refugees.
Which of these statements are correct ?
(a) 1 and 2
(b) 2 and 3
(c) 2 and 4
(d) 3 and 4
Ans. (d)
Explanation:
Arya Samaj was founded in the year 1875 by Swami Dayananda Saraswathi.
Laia Lajpat Rai was one of the members of Arya Samaj and supported the
appeal of Arya Samaj to the authority of Vedas in support of the social reform
programmes.
98. Who among the following leaders proposed to adopt Complete Independence
as the goal of the Congress in the Ahmedabad session of 1920?
(a) Abdul Kalam Azad
(b) Hasrat Mohani
(c) Jawahar Lai Nehru
(d)
Mohandas
Karamchand Gandhi
Ans. (b)
Explanation:
Hasrat Mohani proposed to adopt complete independence as the goal of the
Congress in the Ahmedabad session of 1920. It was opposed by Gandhiji
which led to the rejection of resolution for complete independence.
99. Who among the following organized the famous Chittagong armoury raid ?
(a) Laxmi Sehgal
(b) Surya Sen
(c) Batukeshwar Datta
(d) J. M. Sengupta
Ans. (b)
Explanation:
Surya Sen led the famous Chittagong Armoury raid on 18th April 1930.

50

100. A London branch of the All India Muslim League was established in 1908 under
the presidency of
(a) Aga Khan
(b) Ameer Ali
(c) Liaquat Ali Khan (d)
M.
A.
Jinnah
Ans. (b)
Explanation:
Aga Khan was the President of Muslim League in India. Liaquat Ali and Jinnah
were in Congress at that time. Ameer Ali, a judge from Calcutta High Court
(1890-1904), later settled in England and founded Muslim League there.
101. Who among the following, was the President of the All-India States Peoples*s
Conference in 1939 ?
(a) Jaya Prakash Narayan
(b) Jawahar Lai Nehru
(c) Sheikh Abdullah
(d) Sardar Vallabhbhai Patel
Ans. (b)
Explanation:
Jawaharlal Nehru was the President of the All-India States People's
Conference in 1939 which aims at securing peoples sovereignty in the princely
states.
102. Who amongst the following Englishmen, first translated Bhagvad-Gita into
English ?
(a) William Jones
(b) Charles Wilkins (c) Alexander Cunningham (d) John
Marshall
Ans. (b)
Explanation:
Charles Wilkins was the first Englishman to translate Bhagavad Gita into
English. He was a member of Asiatic Society founded by William Jones.
2002
103. During the Indian freedom struggle, the Khudai Khidmatgars. also known as
Red Shirts, called for
(a) the Union of Pashtun tribal areas in north-west with the Afghanistan
(b) the adoption of terrorist tactics and methods for terrorising and finally ousting
the colonial rulers
(c) the adoption of communist revolutionary ideology for political and social
reform
(d) the Pathan regional nationalist unity and a struggle against colonialism
Ans. (d)
Explanation:
Khan Abdul Gaffar Khan or Frontier Gandhi led the Khudai Khidmatgars. It calls
for the pathan regional nationalist unity and a struggle against colonialism.
Directions : The following items consist of two statements, one labelled as the
Assertion (A) and the other as Reason (R). You are to examine these two
statements carefully and select the answers to these items using the code
given below :
(a) Both A and R are true and R is the correct explanation of A
(b) Both A and R are true but R is NOT the correct explanation of A
(c) A is true but R is false
(d) A is false but R is true
104. Assertion (A) : The effect of labour participation in the Indian nationalist
upsurge of the early 1930s was weak.
Reason (R) : The labour leaders considered the ideology of Indian National
Congress as bourgeois and reactionary.

51

105.

106.

107.

108.

109.

Ans. (a)
Explanation:
The labour leaders with communist ideology considered the ideology of Indian
National Congress as burgeosis and reactionary. Hence the participation of
labour was very meagre in the Indian Nationalist Upsurge in 1930's.
The last opportunity to avoid the partition of India was lost with the rejection of
(a) Cripps Mission
(b) Rajagopalachari Formula
(c) Cabinet Mission (d) Wavell Plan
Ans. (c)
Explanation:
Based on the recommendations of Cabinet Mission Plan, elections were held to
form Constituent Assembly for drafting a new Constitution. The Muslim League
was reluctant to join the interim Government formed under the leadership of
Jawaharial Nehru. Thus with the rejection of Cabinet Plan by Muslim League,
the last opportunity to avoid partition was lost.
The members of the Constituent Assembly which drafted the Constitution of
India were
(a) nominated by the British Parliament
(b) nominated by the Governor
General
(c) elected by the Legislative Assemblies of var
ious provinces
(d) elected by the Indian National Congress and Muslim League
Ans. (c)
Explanation:
As per the Cabinet Mission Plan, the members of the Constituent Assembly
which drafted the Constitution of India, were elected by the legislative
assemblies of various provinces.
With which one of the following mountain tribes did the British first come into
contact with after the grant of Diwani in the year 1765 ?
(a) Garos
(b) Khasis
(c) Kukis
(d) Tipperahs
Ans. (b)
Explanation:
Khasis were the first mountain tribes to come into contact with the British after
the grant of Diwani in the year 1765.
With reference to the period of extremist nationalist movement in India with its
spirit of Swadeshi, which one of the following statements is not correct ?
(a) Liakat Hussain led the Muslim peasants of Barisal in their agitations
(b) In 1898, the scheme of national education was formulated by Satish
Chandra Mukherjee
(c) The Bengal National College was founded in 1906 with Aurobindo as the
Principal
(d) Tagore preached the cult of Atmasakti, the main plank of which was social
and economic regeneration of the villages
Ans. (a)
Explanation:
Barisal Movement was for breaking salt law during Civil Disobedience
Movement. Liakat Hussain did not lead the Barisal movement of peasants.
With reference to the Indian freedom struggle, which one of the following
statements is not correct ?
(a) Hakim Ajmal Khan was one of the leaders to start a nationalist and militant
Ahrar movement

52

(b) When the Indian National Congress was formed, Sayyid Ahmad Khan
opposed it
(c) The All-India Muslim League which was formed in 1906 vehemently
opposed the partition of Bengal and separate electorates
(d) Maulana Barkatullah and Maulana Obeiduliah Sindhi were among those who
formed a Provisional Government of India in Kabul
Ans. (c)
Explanation:
All India Muslim League which was formed in 1906 strongly supported the
Partition of Bengal which led to the carving out of East Bengal with Muslim
majority. Under the leadership of Aga Khan, they demanded separate
electorate for the Muslims.
110. Which one of the following submitted in 1875 a petition to the House of
Commons demanding India's direct representation in the British Parliament ?
(a) The Deccan Association
(b) The Indian Association
(c) The Madras Mahajan Sabha
(d) The Poona Sarvajanik Sabha
Ans. (d)
Explanation:
The Poona Sarvajanik Sabha submitted a petition in 1875 to the House of
Commons demanding Indias direct representation to the British Parliament.
Here except Poona Sarvajanik Sabha all others were founded later than 1875.
111. The real intention of the British to include the princely states in the Federal
Union proposed by the India Act of 1935 was to :
(a) exercise more and direct political and administrative control over the princely
states
(b) involve the princes actively in the administration of the colony
(c) finally effect the complete political and administrative take-over of all the
princely states by the British
(d) use the princes to counter-balance the antiimperialist doctrines of the
nationalist leaders
Ans. (d)
Explanation:
The British real intention was to use the princes to counter balance the antiimperialist doctrines of the nationalist leaders. The princes were loyal to the
British authorities.
112. Mateh List I with List II and select the correct answer using the codes given
below the lists:
List I (Acts of Colonial
List
II
(Provisions) Government of India)
A. Charter Act, 1813
1. Set up a Board of Control in
Britain to fully
regulate
the
East
India
Company's affairs
in India
B. Regulating Act
2. Company's trade monopoly in
India was
ended
C. Act of 1858
3. The power to govern was
transferred from

53

the East India Company to the


British
Crown
4. The Company's directors were

D. Pitt's India Act


asked to

present to the British government


all
correspondence and documents
pertaining to the administration of
the

113.

114.

115.

116.

company
A B C D
A B C D
(a) 2 4 3 1
(b) 1 3 4 2
(c) 2 3 4 1
(d) 1 4 3 2
Ans. (a)
Explanation:
A-2, B-4, C-3, D-l
Which one of the following Acts of British India strengthened the Viceroy's
authority over his executive council by substituting "portfolio or departmental
system for corporate functioning?
(a) Indian Councils Act, 1861
(b) Government of India Act, 1858
(c) Indian Councils Act, 1892
(d) Indian Councils Act, 1909
Ans. (a)
Explanation:
The Indian Councils Act, 1861 passed during Lord Cannings Viceroyality has
strengthened the Viceroy's authority over his executive Council by substituting
'portfolio' or departmental system for corporate functioning.
The President of Indian National Congress at the time of partition of India was
(a) C. Rajagopalachari
(b) J. B. Kripalani
(c) Jawaharlal Nehru
(d) Maulana Abul Kalam Azad
Ans. (b)
Explanation:
J.B. Kripalani was the President of INC at the time of Partition of India.
With reference to colonial period of Indian history, Match List I with List II and
select the correct answer using the codes given below the lists :
List I (Person)
List II (Event)
A. Macdonald
1. Doctrine of Lapse
B. Linlithgow
2. Communal Award
C. Dalhousie
3. August Offer
D. Chelmsford
4. Dyarchy
A B C D
A B C D
(a) 3 2 1 4
(b) 3 2 4 1
(c) 2 3 1 4
(d) 2 3 4 1
Ans. (c)
Explanation:
Ramsay Macdonald - Communal Award, Lord Linlithgow -August Offer, Lord
Dalhousie - Doctrine of Lapse, Chelmsford - Dyarchy.
2003
The leader of the Bardoii Satyagraha (1928) was
(a) Sardar Vallabhbhai Patel
(b) Mahatma Gandhi

54

117.

118.

119.

120.

(c) Vithalbhai J. Patel


(d) Mahadev Desai
Ans. (a)
Explanation:
Bardoii Satyagraha by the Kunbi-patidar land owning peasants and
untouchables, supported by Vallabhbhai Patei, Mehta brothers (1928, Surat,
Gujarat) against oppression by upper castes and hike in revenue by 22 percent
by the Bombay Government. The title 'Sardar' was given by Bardoii women to
Vallabhbhai Patel.
During the colonial period in India, what was the purpose of the. Witley
Commission ?
(a) To review the fitness of India for further political reforms
(b) To report on existing conditions of labour and to make recommendations
(c) To draw up a plan for financial reforms for India
(d) To develop a comprehensive scheme for Civil Services in India
Ans. (b)
Explanation:
To report on existing conditions of labour and to make recommendations.
Directions : The following items consist of two statements, one labelled as the
Assertion (A) and the other as Reason (R). You are to examine these two
statements carefully and select the answers to these items using the code
given below :
(a) Both A and R are true and R is the correct explanation of A
(b) Both A and R are true but R is NOT the correct explanation of A
(c) A is true but R is false
(d) A is false but R is true
Assertion (A) : In 1916, Maulana Mohammad Ali and Abul Kalam Azad
resigned from the Legislative Council.
Reason (R) : The Rowlatt Act was passed by the Government in spite of being
opposed by all Indian members of the Legislative Council.
Ans. (d)
Explanation:
During World War-I, Mohammad Ali was interned and Azad's publication Al hilal
was suppressed. They were not members of the Legislative Council.
Assertion (A) : Shah Alam II spent the initial years as an Emperor far away
from his capital.
Reason (R) : There was always a lurking danger of foreign invasion from the
north-west frontier.
Ans. (c)
Explanation:
Shah Alam II spent the initial years as an Emperor away from his capita!
because of the fear of his wazir.
With reference to the entry of European powers into India, which one of the
following statements is NOT correct ?
(a) The Portuguese captured Goa in 1499
(b) The English opened their first factory in South India at Masulipatam
(c) In Eastern India, the English Company opened its first factory in Orissa in
1633
(d) Under the leadership of Dupleix, the French occupied Madras in 1746
Ans. (a)
Explanation:
The Portuguese captured Goa in 1510.

55

121. With reference to Indian freedom struggle, which one of the following
statements is NOT correct ?
(a) The Rowlatt Act aroused a wave of popular indignation and led to the
Jailianwala Bagh massacre
(b) Subhas Chandra Bose formed the Forward Bloc
(c) Bhagat Singh was one of the founders of Hindustan Republican Socialist
Association
(d) in 1931, the Congress Session at Karachi opposed Gandhi-Irwin Pact
Ans. (d)
Explanation:
Gandhi prevailed upon Karachi Session to approve this agreement of 1931,
122. In India, among the following locations, the Dutch established their earliest
factory at
(a) Surat
(b) Pulicat
(c) Cochin
(d)
Cassimbazar
Ans. (b)
Explanation:
The Dutch established their earliest factory at Pulicat.
123. Which one of the following statements is NOT correct?
(a) The National Song Vande Mataram was composed by Bankimchandra
Chatterji originally in Bengali
(b) The National Calendar of India based on Saka era has its 1st Chaitra on
22nd March normally and 21st March in a leap year
(c) The design of the National Flag of India was adopted by the Constituent
Assembly on 22nd July, 1947
(d) The song Jana-gana-rnana, composed originally in Bengali by
Rabindranath Tagore was adopted in its Hindi version by the Constituent
Assembly on 24th January, 1950 as the National Anthem of India
Ans. (a)
Explanation:
The National Song Vande Mataram was composed by Bankim Chandra
Chatterji originally in Sanskrit.
124. The aim of education as stated by the Woods dispatch of 1854 was
(a) the creation of employment opportunities for native Indians
(b) the spread of western culture in India
(c) the promotion of literacy among the people using English medium of
language
(d) the introduction of scientific research and rationalism in the traditional Indian
education
Ans. (c)
Explanation:
Wood's dispatch of 1854 states that the aim of education is the promotion of
literacy among the people using English as medium of language.
125. Which one of the following statements is NOT correct ?
(a) Ali Mardan Khan introduced the system of revenue farming in Bengal
(b) Maharaja Ranjit Singh set up modern foundries to manufacture cannons at
Lahore
(c) Sawai Jai Singh of Amber had Euclids Elements of Geometry translated
into Sanskrit

56

(d) Sultan Tipu of Mysore gave money for the construction of the idol of
Goddess Sarda in the Shringeri temple
Ans. (a)
Explanation:
Murshid Qu!i Khan introduced the system of revenue farming in Bengal.
126. Which one of the following provisions was NOT made in the Charter Act of
1833 ?
(a) The trading activities of the East India Company were to be abolished
(b) The designation of the supreme authority was to be changed as the
Governor-General of India in Council
(c) All law-making powers to be conferred on Governor-General in Council
(d) Am Indian was to be appointed as a Law Member in Governor-Generals
Council
Ans. (d)
Explanation:
A law member was to be appointed but not an Indian. First law member
appointed under this Charter Act, 1833 was Lord Macaulay.
127. With reference to colonial rule in India, what was sought by the libert Bill in
1883 ?
(a) To bring Indians and Europeans on par as far as the criminal jurisdiction of
courts was concerned
(b) To impose severe restrictions on the freedom of the native press as it was
perceived to be hostile to colonial rulers
(c) To encourage the native Indians to appear for civil service examinations by
conducting them in India
(d) To allow native Indians to possess arms by amending the Arms Act
Ans. (a)
Explanation:
To bring Indians and Europeans on par as far as the criminal jurisdiction of
courts was concerned. The controversy of libert Bill was that, it proposed to
give the Indian Judges the right to trial Europeans also. It was introduced
during Lord Rippon period and was repealed later.
128. An important aspect of the Cripps Mission of 1942 was
(a) that all Indian States should join the Indian Union as a condition to consider
any degree of autonomy for India
(b) the creation of an Indian Union with Dominion status very soon after the
Second World War
(c) the active participation and cooperation of the Indian people, communities
and political parties in the British war effort as a condition for granting
independence with full sovereign status to India after war
(d) the framing of a constitution for the entire Indian Union, with no separate
constitution for any province,and a Union Constitution to be accepted by all
provinces
Ans. (b)
Explanation:
The Cripps Mission was sent during World War-II after the August offer was
rejected, to get the full support of Indians for the British in their war efforts. The
important aspect was the creation of an Indian Union with dominion status very
soon after the Second World War.

57

129. When Congress leaders condemned the Montague-Chelmsford Report, many


moderates left the party to form the
(a) Swarajya Party
(b) Indian Freedom Party
(c) Independence Federation of India
(d) Indian Liberal Federation
Ans. (d)
Explanation:
Indian Liberal Federation' was formed by the moderates who left the Congress
Party after Montford report. Among them prominent member was Surendranath
Banerjee.
2004
130. Consider the following statements:
Some of the main features of the Government of India Act, 1935 were the
1. abolition of diarchy in the Governors provinces
2. power of the Governors to veto legislative action and to legislate on their
own
3. abolition of the principle of communal representation
Which of the statements given above is/are correct?
(a) 1 only
(b) 1 and 2
(c) 2 and 3
(d) 1, 2 and 3
Ans. (b)
Explanation:
Separate electorates provided for Muslims, Sikhs, Christians, Anglo-Indians
and Europeans. Introduction of provincial autonomy and abolition of dyarchy in
the provinces. Governors retained the veto power.
131. Consider the following statements:
1. The First Session of the Indian National Congress was held in Calcutta.
2. The Second Session of the Indian National Congress was held under the
presidentship of Dadabhai Naoroji.
3. Both Indian National Congress and Muslim League held their sessions at
Lucknow in 1916 and concluded the Lucknow Pact.
Which of the statements given above is/are correct?
(a) 1 and 2
(b) 2 only
(c) 2 and 3
(d) 3 only
Ans. (c)
Explanation:
The first meeting of INC began on 28 December 1885 in the hall of Gokuldas
Tejpal Sanskrit College, Bombay under the leadership of W.C. Banerjee.
Second session of INC was held at Calcutta under the Presidentship of
Dadabhai Naoroji
132. Which one of the following statements is correct ?
(a) The Constituent Assembly of India was elected by the Provincial Assemblies
in the year 1946
(b) Jawaharlal Nehru, M. A. Jinnah and Sardar Vallabhbhai Patel were
members of the Constituent Assembly of India
(c) The First Session of the Constituent Assembly of India was held in January,
1947
(d) The Constitution of India was adopted on 26 th Januaiy, 1950
Ans. (a)
Explanation:
M.A. Jinnah of Muslim League was not a member of Constituent Assembly.
The First Session of Constituent Assembly was held in December, 1946. The
Constitution of India was adopted on 26th November, 1949.

58

133. The Montague-Chelmsford Report formed the basis of


(a) the Indian Councils Act, 1909
(b) the Government of India Act,
1919
(c) the Government of India Act, 1935
(d) the Indian Independence Act,
1947
Ans. (b)
Explanation:
In line with the Government policy contained in Montague's statement (August,
1917), the Government announced further constitutional reforms in July 1918,
known as Montague-Chelmsford or Mont ford reforms, which formed the
basis for the Government of India Act, 1919.
134. During the Indian Freedom Struggle, who among the following proposed that
Swaraj should be defined as complete independence free from all foreign
control ?
(a) Mazharul Haque
(b) Maulana Hasrat Mohani
(c) Hakim Ajmal Khan
(d) Abul Kalam Azad
Ans. (b)
Explanation:
In 1921, Republican Muslim leader Hasrat Mohani wanted to move a resolution
defining Swaraj as complete Independence, free from all foreign control which
was opposed by Gandhiji and led to its rejection.
135. Consider the following princely States of the British rule in India :
1. Jhansi
2. Sambalpur
3. Satara
The correct chronological order in which they were annexed by the British is
(a) 1, 2, 3
(b) 1, 3, 2
(c)3, 2, 1
(d) 3, 1, 2
Ans. (c)
Explanation:
These states were annexed by Dalhousie in the sequence of
Satara - 1844 A.D., Sambalpur - 1849 A.D., Jhansi - 1853 A.D.
136. The name of the famous person of India who returned the Knighthood
conferred on him by the British Government as a token of protest against the
atrocities in Punjab in 1919 was
(a) Tej Bahadur Sapru
(b) Ashutosh Mukherjee
(c) Rabindra Nath Tagore
(d) Syed Ahmad Kban
Ans. (c)
Explanation:
Rabindranath Tagore returned the 'Knighthood' as a token of protest against
Jallianwala Bagh tragedy (April 13, 1919) in Punjab.
137. Which of the following pairs are correctly matched ?
List I (Period)
List II (Wars)
1. AD 1767-69
: First Anglo-Maratha War
2. AD 1790-92
: Third Mysore War
3. AD 1824-26
: First Anglo-Burmese War
4. AD 1845-46
: Second Sikh War
Select the correct answer using the codes given below:
(a) 2 and 4
(b) 3 and 4
(c) 1 and 2
(d) 2 and 3
Ans. (d)
Explanation:
First Angio-Maratha War - 1775-82
Second Angio-Sikh War - 1848-49

59

138. Which one of the following pairs is not correctly matched ?


(a) Pitts India Act
:
Warren Hastings
(b) Doctrine of Lapse
:
Dalhousie
(c) Vernacular Press Act :
Curzon
(d) Ilbert Bill
:
Ripon
Ans. (c)
Explanation:
Vernacular Press Act was enacted in 1878 during the Viceroyship of Lord
Lytton.
139. Consider the following Viceroys of India during the British rule :
1. Lord Curzon
2. Lord Chelmsford 3. Lord Hardinge
4. Lord Irwin
Which one of the following is the correct chronological order of their tenure ?
(a) 1 -3-2-4
(b) 2-4-1 -3
(c) 1 -4-2-3
(d) 2-3-1-4
Ans. (a)
Explanation:
Lord Curzon : 1899-1905, Lord Hardinge 1910-1916, Lord Chelmsford : 19161921, Lord Irwin : 1926-31.
140. Consider the following events during Indias freedom struggle :
1. Chauri - Chaura Outrage
2. Minto - Morley Reforms
3. Dandi March
4. Montague - Chelmsford Reforms
Which one of the following is the correct chronological order of the events
above ?
(a) 1 -3 -2-4
(b) 2-4-1 -3
(c) 1 -4-2-3
(d) 2-3 -1 -4
Ans. (b)
Explanation:
Morley Minto Reforms : 1909 Montague-Chelmsford Reforms : 1919 ChauriChaura Outrage : 1922 Dandi March : 1930
2005
141. Which party was founded by Subhash Chandra Bose in the year 1939 after he
broke away from the Congress ?
(a) Indian Freedom Party
(b) Azad Hind Fauz
(c) Revolutionary Front
(d) Forward Block
Ans. (d)
Explanation:
Forward Block was founded in the year 1939 by Subhash Chandra Bose.
142. Consider the following statements :
The Government of India Act, 1935 provided for
1. the provincial autonomy.
2. the establishment of Federal
Court.
3. All India Federation at the Centre.
Which of the statements given above are correct ?
(a) 1 and 2
(b) 2 and 3
(c) 1 and 3
(d) 1, 2 and 3
Ans, (d)
Explanation:
Government of India Act, 1935 provides for provincial autonomy, Federal court
and All
India Federation at the Centre.
143. Consider the following statements :
1. Warren Hastings was the first Governor General who established a regular
police force in India on the British pattern.
2. A Supreme Court was established at Calcutta by the Regulating Act, 1773.

60

3. The Indian Penal Code came into effect in the year 1860.
Which of the statements given above are correct ?
(a) 1 and 2
(b) 2 and 3
(c) 1 and 3
(d) 1, 2 and 3
Ans. (b)
Explanation:
Cornwallis was the first Governor General who established a regular force on
the British pattern in India.
144.Which one of the following is the correct chronological order of the battle fought
in India in the 18th Century?
(a) Battle of Wandiwash - Battle of Buxar - Battle of Ambur - Battle of Plassey
(b) Battle of Ambur - Battle of Plassey - Battle of Wandiwash - Battle of Buxar
(c) Battle of Wandiwash - Battle of Plassey - Battle of Ambur - Battle of Buxar
(d) Battle of Ambur - Battle of Buxar - Battle of Wandiwash - Battle of Plassey
Ans. (b)
Explanation:
Battle of Ambur - Early Seventees
Battle of Plassey - 1757
Battle of Wandiwash - 1760
Battle of Buxar - 1764
145. Which one of the following is the correct statement?
(a) The modem Kochi was a Dutch colony till Indias Independence
(b) The Dutch defeated the Portuguese and built Fort Williams in the modern
Kochi
(c) The modern Kochi was first a Dutch colony before the Portuguese took over
from them
(d) Ihe modern Kochi never became a part ot toe British colony
Ans. (d)
Explanation:
Dutch lost all their settlements in India to British in 1795. Fort Williams was built
by the British in Kolkata. Portuguese were the first to take Kochi as their
Colony. Kochi never became a part of British Colony.
146. Consider the following statements :
1. Lord Mountbatten was the Viceroy when Simla Conference took place.
2. Indian Navy Revolt, 1946 took place when the Indian sailors in the Royal
Indian Navy at Bombay and Karachi rose against the Government.
Which of the statements given above is/are correct?
(a) 1 only
(b) 2 only
(c) Both 1 and 2
(d) Neither 1
nor 2
Ans. (b)
Explanation:
Simla Conference (1945) took place'when Lord Wavell was the Viceroy (194347). On February 18, 1946 Naval ratings of HMIS Talwar at Bombay and later
the Naval ratings of HMIS Hindustan at Karachi went on strike. On 28th
February 1946, with the efforts of Vallabhbhai Patel, Mohammad Ali jinnah and
some of the communist leaders, the ratings were agreed to surrender.
147. Which one of the following territories was not affected by the Revolt of 1857 ?
(a) Jhansi
(b) Chittor
(c) Jagdishpur
(d) Lucknow
Ans. (b)
Explanation:

61

148.

149.

150.

151.

152.

Most of the Punjab, Rajputana (Chittor) and Kashmir remained peaceful Nizam,
of Hyderabad, Gulab Singh of Kashmir, Sikh rulers of Patiala, Nabha and Jind,
Holkar of Indore, Scindia of Gwalior, Nawab of Bhopal, Rulers of Tehri and
Tikargarh in order to retain their position and state, helped the British to
suppress the revolt of 1857.
Which one of the following places did Kunwar Singh, a prominent leader of the
Revolt of 1857 beiong to ?
(a) Bihar
(b) Madhya Pradesh (c) Rajasthan
(d)
Uttar
Pradesh
Ans. (a)
Explanation:
Kunwar Singh, the Zamindar of Jagdishpur led the revolt, in Bihar. He joined
the sepoys when they reached Arrah from Dinapore.
Which of the following uairs Hi*is co rrectly matched ?
Movement/Satyagraha
Person Actively Associated With
1. Champaran
:
Rajendra Prasad
2. Ahmedabad Mill Workers :
Morarji Desai
3. Kheda
:
Vallabhbhai PAtei
Select the correct answer using the codes given below :
(a) 1 and 2
(b) 2 and 3
(c) 1 and 3
(d) 1, 2 and 3
Ans, (d)
Explanation:
Morarji Desai born in 1896 participated in the Ahmedabad Miil worker's
Satyagraha.
Who among the following was not associated with the formation of U.P. Kisan
Sabha in February 1918 ?
(a) Indra Narain Dwivedi
(b) Gauri Shankar Misra
(c) Jawaharlal Nehru.
(d) Madan Mohan Malviya
Ans. (c)
Explanation:
The UP Kisan Sabha was set up in February 1918 by Gauri Shankar Misra and
Indra Narayan Dwivedi while Madan Mohan Malviya supported their efforts.
Jawaharlal Nehru was not associated with the formation of U.P. Kisan Sabha
but later in 1920, he visited the villages in U.P. to look at farmer's conditions at
the behest of Baba Ramchandra,
Who among the following drafted the resolution on fundamental rights for the
Karachi Session of Congress in 1931 ?
(a) Dr. B. R. Ambedkar
(b) Pandit Jawaharlal Nehru
(c) Dr. Rajendra Prasad
(d) Sardar Vallabhbhai Patel
Ans. (b)
Explanation:
Karachi Session of Conress in 1931 was presided over by Sardar Vallabhbhai
Patel but the draft on Fundamental Rights was done by Jawaharlal Nehru.
In October 19205 who of the following headed a group of Indians gathered at
Tashkent to set up a Communist Party of India ?
(a) H. K. Sarkar
(b) P. C. Joshi
(c) M. C. Chagla
(d) M. N. Roy
Ans. (d)
Explanation:
M.N. Roy headed the group of Indians along with Ayani Mukherjee, Mohammad
Ali and Mohammad Shafi at Tashkent in October, 1920.

62

153. At which Congress Session was the 'Working Committee authorised to launch
a programme of Civil Disobedience ?
(a) Bombay
(b) Lahore
(c) Lucknow
(d) Tripuri
Ans. (b)
Explanation:
A resolution for Purna Swaraj was adopted at the Lahore Session (December,
1929) under the Presidentship of Jawaharlal Nehru. In the same session
congress working committee (CWC) authorized to launch a programme of civil
disobedience including non-payment of taxes and all members of legislature
asked to resign their seats.
154. In which one of the following provinces was a Congress ministry not formed
under the Act
of 1935?
(a) Bihar
(b) Madras
(c) Orissa
(d) Punjab
Ans. (d)
Explanation:
In Punjab, Sir Sikandar Hayat Khan of National Unionist Party formed the
Government. The Conress got a majority in all provinces except in Bengal,
Assam, Punjab, Sindh and NWFP, and emerged as the largest party in Bengal,
Assam and NWFP.
155. Consider the following statements : On the eve of the launch of Quit India
Movement, Mahatma Gandhi
1. asked the government servants to resign.
2. asked the soldiers to leave their posts.
3. asked the Princes of the Princely States to accept the sovereignty of their
own people.
Which of the statements given above is/are correct?
(a) 1 and 2
(b) 2 and 3
(c) 3 only
(d) 1,2 and 3
Ans. (c)
Explanation :
Mahatma Gandhi asked the Government servants not to resign but declare
allegiance to the Congress. The Soldiers were asked not to leave the army but
do not fire on Compatriots.
156. Consider the following statements :
1. Ishwar Chandra Vidyasagar founded the Bethune School at Calcutta with
the main aim of encouraging education for women.
2. Bankim Chandra Chattopadhyay was the first graduate of the Calcutta
University.
3. Keshav Chandra Sens campaign against Safi led to the enactment of a law
to ban Sati by the then Governor -General.
Which of the statements given above is/are correct?
(a) 1 only
(b) 1 and 2
(c) 2 and 3
(d) 1, 2 and 3
Ans, (b)
Explanation:
Joh Elliot Drink water Bethune a law member in Governor Generals Council
was the founding father of Bethune College which blossoms into Bethune
School to promote female education. Bankim Chandra Chatterjee was one of
the first graduate of Calcutta University. His first Bengali Novel was 'Durgesh
Nandini'.
Raja Ram Mohan Roy's campaign against Sati led to the enactment of a Law to
ban Sati (1829).

63

157. Consider the following statements :


1. In the First Round Table Conference Dr. Ambedkar demanded separate
electorates for the depressed classes.
2. In the Poona Act, special provisions for representation of the depressed
people in the local bodies and civil sen/ices were made.
3.
The Indian National Congress did not take part in the Third Round Table
Conference.
Which of the statements given above is/are correct?
(a) 1 and 2
(b) 2 and 3
(c) 1 and 3
(d) 1, 2 and 3
Ans. (c)
Explanation:
First Round Table Conference (1930) presided by Ramsay MacDonald (head of
labour party) where Ambedkar demanded separate electorate for depressed
classes.
Second Round Table Conference (1931) : Congress participated.
Third Round Table Conference (1932) : Congress didn't participate . Poona
pact had no provisions for reservation in Civil Services and local bodies.
158. Who among the following was a proponent of Fabianism as a movement ?
(a) Annie Besant
(b) Michael Madhusudan Dutt
(c) A.O.Hume
(d) R. Palme Dutt
Ans. (a)
Explanation:
Annie Besant (1897-1"33), as a member of the National Secular Society, she
had preached free thought and as a member of the Fabian Society, Socialism.
159. What was Komagata Maru ?
(a) A political party based in Taiwan
(b) Peasant communist leader of
China.
(c) A naval ship on voyage to Canada.
(d) A Chinese village where Mao Tre Lung begar his long march.
Ans. (c)
Explanation:
Baba Gurdip Singh chartered a Japanese ship called Komagata Maru, early in
1914 to carry large number of Punjabis to Canada.
160. Where were the Ghadar revolutionaries, who became active during the
outbreak of the World War I based ?
(a) Central America (b) North America
(c) West America
(d) South
America
Ans. (b)
Explanation:
Ghadar revolutionaries had headquarters at San Francisco in North America.
2006
161. Who was the Governor-General of India during the Sepoy Mutiny ?
(a) Lord Canning
(b) Lord Dalhousie (c) Lord Hardinge
(d) Lord Lytton
Ans. (a)
Explanation:
Lord Canning was the Governor-General of India during the Sepoy Mutiny. He
was the last and first Governor General and Viceroy of British India
respectively.
162. Consider the following statements about Madam Bhikaji Cama :

64

163.

164.

165.

166.

1. Madam Cama unfurled the National Flag at the International Socialist


Conference in Paris in the year 1907.
2. Madam Cama served as private secretary to Dadabhai Naoroji,
3. Madam Cama was bom to Parsi parents.
Which of the statements given above is/are correct?
(a) 1, 2 and 3
(b) 2 and 3, only
(c) 1 and 2, only (d) 3 only
Ans. (b)
Explanation:
Madam Cama held aloft the flag of Indian Freedom by taking part in
International Conference at Stuttegaurd in Germany on 18 August 1907.
Consider the following statements :
1. The Charter Act 1853 abolished East India Companys monopoly of Indian
trade.
2. Under the Government of India Act, 1858, the British Parliament abolished
the East India Company altogether and undertook the responsibility of ruling
India directly.
Which of the statements given above is/are correct?
(a) 1 only
(b) 2 only
(c) Both I and 2
(d) Neither 1
nor 2
Ans. (b)
Explanation:
The Charter Act of 1813 abolished East India Company's monopoly of Indian
trade except for trade in tea and trade with China. Charter Act 1833 abolished
company's monopoly of trade in tea and trade with China.
Which one of the following revolts was made famous by Bankim Chandra
Chatierjee in his noval A.mind Math ?
(a) Bhil uprising
(b) Rangpur and Dinapur uprising
(c) Bishnupur and Birbhum rebellion
(d) Sanyasi rebellion
Arts. (d)
Explanation:
Sanyasi Rebellion (1770) : restrictions imposed on visits to holy places
estranged the Sanyasis. The Sanyasis retaliated by organising raids on the
company's factories and state treasuries under the leadership of Kena Sarkar'
and 'Dirji Narayan' in Bengal and Bihar.
In the year 1613, where was the English East India Company given permission
to set up a factor (trading post) ?
(a) Bangalore
(b) Madras
(c) Masulipattam
(d) Surat
Ans. (a)
Explanation:
The Company was granted permission to open Factory at Surat in 1608. In
1615, Sir Thomas Roe received royal farman from Jahangir to establish
factories at Agra, Ahmedabad, Baroda and Broach (on West Coast) :
Masulipatnam and Armagaon near Pulicat (South-East Coast).
With reference to the revolt of the year !8575 who of the following was betrayed
by a -friend;
captured and put to death by the British ?
(a) Nana Sahib
(b) Kunwar Singh
(c) Khan Bahadur Khan (d) Tantia
Tope
Ans. (d)
Explanation:

65

167.

168.

169.

170.

171.

Tantia Tope was betrayed by his friend Man Singh. He was captured and put to
death by the British in April 1859.
Under whose presidency was the Lahore Session of the Indian National
Congress held in the year 1929 wherein a resolution was adopted to gain
complete independence from the British ?
(a) Bal Gangadhar Tilak
(b) Gopal Krishna Gokhale
(c) Jawaharlai Nehru
(d) Motilal Nehru
Ans. (c)
Explanation :
Jawaharlai Nehru was the President of the Lahore Session of INC in the year
1929, where a resolution to gain complete independence (purna swaraj) from
the British was adopted.
Leetures from Colombo to Almora is based on the experiences of which one
of the following ?
(a) Veer Savarkar
(b) Annie Besant
(c) Ramkrishna Paramhansa
(d) Swami Vivekanand
Ans. (d)
Explanation:
"Lectures from Colombo to Almora" is a collection of 29 speeches of 'Swami
Vivekananda' right from his first public address given in Colombo on Jan 15 th,
1897, after coming back from his triumphant visit to America.
2007
The song Amar Sonar Bangfa written during the Swadeshi Movement of India
inspired the liberation struggle of Bangladesh and was adopted as the National
Anthem of Bangladesh. Who wrote this song ?
(a) Rajni Kanta Sen
(b) Dwijendralal Ray
(c) Mukunda Das
(d) Rabindranath Tagore
Ans. (d)
Explanation:
Rabindranath Tagore wrote and composed the song "Amar Sonar Bangla' (My
Golden Bengal). The song was written in 1906 when Bengal ws divided in two
halves by the British Govt, on religion. The first 10 lines of the song constitute
the national anthem of Bangladesh.
The First Factory Act restricting the working hours of women and children, and
authorizing local governments to make necessary rules was adopted during
whose time ?
(a) Lord Lytton
(b) Lord Bentinck
(c) Lord Ripon
(d)
Lord
Canning
Ans. (c)
Explanation:
The first Factory Act was passed (1881) during Lord Ripon's Viceroyship (188084) to improve labour conditions.
Viceroy
Tenure
Lord Lytton
1876-80
Lord Bentinck
1828-35
Lord Canning (1st Viceroy)
1856-58, 1858-62
Who among the following wro.te the book Babuvivah ?
(a) Raja Rammohan Roy
(b)
Ishwar
Chandra Vidyasagar
(c) Pandita Rambai
(d) Rabindranath Tagore

66

172.

173.

174.

175.

176.

177.

Ans. (b)
Who wrote the book - The Story of the Integration of the Indian States ?
(a) B. N. Rao
(b) C. Rajagopalachari (c) Krishna Menon (d) V. P.
Menon
Ans. (d)
Explanation:
V. P. Menon was an Indian Civil Servant played an important role in the
partition of India and the integration of Independent India during the period
1945-50.
Which one of the following aroused a wave of popular indignation that led to the
massacre by tfce British at Jallianwala Bagh ?
(a) The Arms Act
(b)The Public Safety Act
(c) The Rowlatt Act
(d)The Vernacular Press Act
Ans. (c)
Explanation:
Jallianwala Bagh Massacre took place on 13th April, 1919, when a huge crowd
had assembled in Amritsar to protest peacefully against the arrest of Saifuddin,
Kitchlew and Satyapal who were arrested on 10 April, 1919 for starting a
Rowlatt Satyagraha against Roylatt Act.
At which one of the following places did Mahatma Gandhi first start his
Satyagraha in India?
(a) Ahmedabad
(b) Bardoli
(c) Champaran
(d) Kheda
Ans. (c)
Explanation:
Champaran Stayagraha - 1917 : Gandhi's first great experiment in Satyagraha
came in 1917 in Champaran, a district in Bihar. He took up the cause of the
poor peasants against the excesses of the indigo planters in Champaran. He
defied the government order asking him to leave the area and was willing to
face trial and imprisonment. The Government was forced to inquire into the
injustices committed by the indigo planters and to put an end to them.
Ahmedabad Mills Strike - 1918 Kheda Satyagraha - 1918
Bardoli (Surat, Gujarat) - Satyagraha was led by Sardar Vallabhbhai Patel in
1928.
Who among the following started the newspaper Shome Prakash ?
(a) Dayanand Saraswati
(b) Ishwar Chandra Vidyasagar
(c) Raja Rammohan Roy
(d)
Surendranath
Banerjee
Ans. (b)
Explanation:
A Bangali newspaper Somprakash' was started in 1898 by IshWar Chandra
Vidyasagar.
The ruler of which one of the following States was removed from power by the
British on the pretext of misgovernance?
(a) Awadh
(b) Jhansi
(c) Nagpur
(d) Satara
Ans. (a)
Explanation:
During the administration of Nawab 'Wazid AH', Awadh was annexed under the
pretext of misrule.
Who among the following Europeans were the last to come to preindependence
India as traders ?

67

178.

179.

180.

181.

(a) Dutch
(b) English
(c) French
(d) Portuguese
Ans. (c)
Explanation :
Entry of European powers in India :
Portuguese
- 1498
English
- 1600
Dutch
- 1602
Danish
- 1616
French
- 1664
Consider the following statements :
1. Robert Clive was the first Governor-General of Bengal.
2. William Bentinck was the first Governor-General of India.
Which of the statements given above is/are correct ?
(a) 1 only
(b) 2 only
(c) Both 1 and 2
(d) Neither
1
nor 2
Ans. (b)
Explanation:
Warren Hastings was the first Governor-General of Bengal as per the
Regulating Act of 1773.
Source : India at a glance - 2007 - Page No. 170
Which one of the following was the first fort constructed by the British in India ?
(a) Fort William
(b) Fort St. George (c) Fort St. David
(d) Fort
St.
Angelo
Ans. (b)
Explanation:
Fort St. George - 1644 (Madras) - British
Fort William - 1700 (Calcutta) - British
Fort St. David - Fort sold by Marathas to British in 1690.
Fort Angelo - Built by Portuguese in 1505 at Kannur (Kerala).
Which one of the following places was associated with Acharya Vinoba Bhaves
Bhoodan Movement at the beginning of the movement?
(a) Udaygiri
(b) Rapur
(c) Pochampalli
(d) Venkatagiri
Ans. (c)
Explanation:
Eminent Gandhian constructive worker Acharya Vinoba Bhave, drew upon
Gandhian techniques and ideas such as constructive work and trusteeship, to
launch the Bhoodan Movement in early fifties (1950's). Vinoba Bhave received
the first donation of land on 18 April 1951 in the village of Pachampalli in the
Telangana district of Andhra Pradesh.
Assertion (A) : According to the Wavell Plan, the number of Hindu and Muslim
members in the Executive Council were to be equal.
Reason (R) : Wavell thought that this arrangement would have avoided the
partition of India.
Arts. (c)
Explanation:
Wavell Plan proposed a council with equal numbers of Hindus and Muslims,
and one representative each of Sikhs and the Depressed classes, besides the
Commander in chief and the main purpose was to delay the process of partition
but not to avoid partition.

68

4. INDIAN CULTURE
1.

2.

3.

4.

5.

6.

7.

1996
Which one of the following temples figured in the news regarding the institution
of the Devadasis ?
(a) Jagannath Temple, Puri
(b) Pasupatinath temple, Kathmandu
(c) Kandariyamahadev temple, Khajuraho (d) Chaunsathyogini
temple,
Bheraghat
Ans. (a)
Explanation:
Devadasis an institution at Jagannath Temple, Puri.
In the context of ancient Indian society, which one of the following terms does
not belong to the category of the other three ?
(a) Kula
(b) Vamsa
(c) Kosa
(d) Gotra
Ans. (c)
Explanation:
Kula, Vamsa, Gotra belong to same category which represents kinship groups
of ancient Indian society. The term 'Kosa' was used for 'treasury'.
Which one of the following sculptures invariably used green schist as the
medium ?
(a) Maurya sculptures
(b) Mathura sculptures
(c) Bharhut sculptures
(d) Gandhara sculptures
Ans. (c)
Explanation:
Bharhut sculptures invariably used green schist as the medium.
The term apabhramsa was used in medieval Sanskrit texts to denote
(a) outcastes among the Rajputs
(b) deviations from Vedic rituals
(c) early forms of some of the modern Indian languages
(d) non-Sanskrit verse metres
Ans. (c)
Explanation:
Apabhramsa (falling away) denotes the early forms of some of the modern
Indian languages.
Which one of the following was a Saiva sect in ancient India ?
(a) Ajivika
(b) Mattamayura
(c)Mayamata
(d)Isanasivagurudevapaddhati
Ans. (a)
Explanation:
Ajivika was Saiva sect in Ancient India. Other Saiva sects are Pasupata,
Virashaiva or Lingayats (earliest Saiva sect), etc.
Which one of the following texts of ancient India allows divorce to a wife
deserted by her husband ?
(a) Kamasutra
(b) Manavadharmashastra
(c) Sukra Nitisara
(d) Arthashastra
Ans. (d)
Explanation :
Arthashastra written by Kautilya (Chanakya) allows divorce to a wife deserted
by her husband.
Which one of the following pairs is correctly matched ?
(a) Naqqual
: Bihar
(b) Tamasha
: Orissa
69

(c)AnkiaNat
: Assam
(d) Baha
: Punjab
Ans. (a)
Explanation :
Naqqual is a kind of folk theatre in Bihar. Tamasha is a folk drama form
flourishing in Maharashtra. Ankita Nat is a one-act play with its roots in rural
Assam.
8.
Which one of the following pairs is correctly matched ?
(a) Guru Amar Das : Miri and Piri
(b) Guru Arjun Dev : Adi Granth
(c) Guru Ram Das : Dal KJialsa
(d) Guru Gobind Singh: Manji
Ans. (b)
Explanation:
Guru Arjun Dev compiled Adi Granth and Sukhmani. Guru Amardas collected
hymns and Guru Ramdas founded Amritsar. Guru Gobind ingh founded the
Khalsa panth and compiled supplementary Granth called Daswen Padshah Ka
Granth.
1997
9.
Which one of the following pairs of composers in different languages and their
works on the Mahabharata theme is correctly matched ?
(a) Sarladasa
: Bengali
(b) Kasirama
: Oriya
(c) Tikkana
: Marathi
(d) Pampa
: Kannada
Ans. (d)
Explanation:
Pampa : Father of Kannada poetry wrote Adi purana, and Vikramarjunaya
Vijaya.
Tikkana : Telugu version of Mahabharata in 13th Century along with Nannaya
(11th century) and Yerrapragada (14th century).
Sarladasa : Oriya version of Mahabharata in the 14th Century.
Kasirama : Bengali version of Mahabharata.
10. Which one of the following dances involves solo performance ?
(a) Bharatanatyam (b) Kuchipudi
(c) Mohiniattam
(d) Odissi
Ans. (a)
Explanation:
Bharatnatyam and Mohiniattam involves solo performance. Mohiniattam is
Kerala's solo female dance.
11. Match List I with List II and select the correct answer using the'codes given
below the lists:
List I (Author)
List II (Text)
A. Varahamihira
1. Prabandha Chintamani
B. Visakhadatta
2. Mrichchha-Katikam
C. Sudraka
3. Brihat-Samhita
D. Bilhana
4. Devi-Chandraguptam
5. Vikramankadeva-Charita
A B C D
A B C D
(a) 3 4 5 2
(b) 3 4 2 5
(c) 5 3 4 1
(d) 1 3 5 2

70

12.

13.

14.

15.

16.

Arts. (b)
Explanation:
Varahamihira - Brihat Samhita,
Visakhadatta - Devi-Chandraguptam,
Mudra Rakshasam Sudraka - Mrichchha-katikam Bilhana - Vikramankadevacharita
1998
Which of the following pairs are correctly matched ?
1. Mrichchakatikam - Shudraka
2. Buddhacharita
- Vasuvandhu
3. Mudrarakshasha - Vishakhadatta
4. Harshacharita
- Banabhatta
Select the correct answer using the codes given below :
(a) 1, 2, 3 and 4
(b) 1, 3 and 4
(c) 1 and 4
(d) 2 and 3
Ans. (b)
Explanation :
Buddhacharita was written by Ashvaghosh.
1999
Which one of the following pairs of folk dance forms and states is not correctly
matched ?
(a) Korku
: Maharashtra
(b) Jhummar
: Haryana
(c) Thali
: Himachal Pradesh
(d) Mukna
: Manipur
Ans. (a)
Which one of the following statements is not correct ?
(a) 'Neel Darpan' was a play based on the exploitation of the indigo farmers
(b) The author of the play 'Ghashiram Kotwal' is Vijay Tendulkar
(c) The play 'Navanna' by Navin Chandra Das was based on the famine of
Bengal
(d) Urdu theatre used to depend heavily on Parsi meatre
Ans. (b)
Match List I with List II and select the correct answer using the codes given
below the lists:
List I (Books)
List II (Authors)
A. The First Indian War of Independence 1. Rabindranath Tagore
B. Anand Math
2. Sri Aurobindo
C. Life Divine
3. Bankim Chandra Chatterji
D. Sadhana
4. Vinayak Damodar Savarkar
A B C D
A B C D
(a) 4 3 2 1
(b) 3 4 1 2
(c) 4 3 1 2
(d) 3 4 2 1
Ans. (a)
Explanation:
First Indian War of Independence - Vinayak Damodar Savarkar.
Anand Math - Bankim Chandra Chatterji
Life Divine - Sri Aurobindo Sadhana - Rabindranath Tagore.
The paintings of Abanindranath Tagore are classified as :
(a) realistic
(b) socialistic
(c) revivalistic
(d)
impressionistic

71

17.

18.

19.

20.

Ans. (c)
Explanation :
Abanindranath Tagore's paintings are classified as revivalistic. He is
considered as the father of modern Indian painting.
Match List I with List II and select the correct answer using the codes given
below the lists:
List I
List II
A. Pandit Vishnu Digambar Paluskar
1. Introduced the scheme of Raga
classification in Indian music
B. Venkatamahi
2. Proponent of the Carnatic music
C. Shyama Shastri
3. Proponent of the Khayal form of
Hindustani
music
D. Amir Khusru
4. Wrote the music for the song
'Vande Mataram'
A B C D
A B C D
(a) 4 1 3 2
(b) 4 1 2 3
(c) 1 4 3 2
(d) 1 4 2 3
Ans. (b)
Explanation:
A-4, B-l, C-2, D-3
The first feature film (talkie) to be produced in India was :
(a) Hatimtai
(b) Alam Ara
(c) Pundalik
(d)Raja
Harishchandra
Ans. (b)
Explanation:
Alam Ara - First Feature Film (talkie) released on May 14, 1931 by A. M. Irani.
Raja Harishchandra - First Feature Film (Silent) released on May 3, 1913 by
Dada Saheb Phalke.
2000
Directions: The following items consist of two statements, one labelled as
'Assertion A' and the other labelled as 'Reason R'. You are to examine these
two statements carefully and decide if the Assertion A and the Reason R are
individually true and if so, whether the Reason is a correct explanation of the
Assertion. Select your answers to these items using the codes given below and
mark your answer sheet accordingly.
(a) Both A and R are true and R is the correct explanation of A
(b) Both A and R are true but R is NOT the correct explanation of A
(c) A is true but R is false
(d) A is false but R is true
Assertion (A) :
The Aham and Puram poems of the Padinen Kilukanakku
group formed a continuation of the Sangam composition.
Reason (R) : They were included under the Post-Sangam works as against the
Sangam works proper.
Ans. (c)
Explanation :
Kilukanakku forms the didactic group of Sangam literature.
Match List-I with List-II and select the correct answer using the codes given
below the lists:
List-I (Artist)
List-II (Medium of music delivery)
A. Balamurali Krishna
1. Hindustani vocal

72

B. Mita Pandit
C. Kanyakumari
D. Nikhil Bannerjee

21.

22.

23.

24.

2.
3.
4.
5.

Ghatam
Sitar
Violin
Carnatic vocal
A B C D
(b) 4 3 1 5
(d) 5 4 1 3

A B C D
(a) 5 1 2 3
(c) 3 1 5 2
Ans. (a)
Explanation:
A. Balamurali Krishna - Carnatic vocal
B. Mita Pandit - Hindustani vocal
C. Kanyakumari - Ghatam
D. Nikhil Bannerjee - Sitar
Which one of the following pairs is not correctly matched ?
(a) Bakti Itihas
: Badal Sarkar
(b) Sita Swayamvar : Vishnu Das Bhave
(c) Yayati
: Girish Karnad
(d) Giddha
: Jabbar Patel
Ans. (d)
Explanation:
'Giddha' is written by 'Vijay Tendulkar'.
The Raga which is sung early in the morning is
(a) Todi
(b) Darbari
(c) Bhopali
(d) Bhimpalasi
Ans. (a)
Explanation :
Togo is the Raga which is sung early in the morning.
2001
Which of the following pairs is NOT correctly matched ?
(a) India's first technicolour film
: Jhansi Ki Rani
(b) India's first 3-Dfilm
:
My
Dear
Kuttichethan
(c) India's first insured film
: Taal
(d) India's first actress to win the Bharat Ratna:
Meena Kumari
Ans. (d)
Explanation:
No actress was awarded Bharat Ratna. Satyajit Ray (Director) and Lata
Mangeshkar (Singer) were awarded Rharat Ratna in 1992 and 2001
respectively.
Match List I with List II and select the correct answer using the codes given
below the lists:
List I (Dancer)
List II (Dance)
A. Kalamandalam Kshemavaty
1. Kathakali
B. Kottakkal Sivaraman
2. Manipuri
C. Lakshmi Viswanathan
3.
Mohiniattam
D. N. Madhabi Devi
4. Bharatanatyam
A B C D
A B C D
(a) 1 3 2 4
(b) 3 1 4 2
(c) 1 3 4 2
(d) 3 1 2 4
Ans. (b)
Explanation:

73

25.

26.

27.

28.

29.

Kalamandalam Kshemavaty - Mohiniattam


Kottakkal Sivaraman - Kathakali
Lakshmi Viswanathan - Bharatanatyam
N. Madhabi Devi - Manipuri
In which one of the following cities is the Lingaraja Temple located ?
(a) Bhubaneswar
(b) Bijapur
(c) Kolkata
(d)
Shravanabelagola
Ans. (a)
Explanation :
Lingaraja temple located at Bhubaneswar. It was dedicated to Shiva
(Mukteswara temple) built by 'Chodaganga'.
2002
Which one of the following statements is not correct ?
(a) The statue of Gomateshwara at Sravanabelagola represents the last
Tirthankara of Jains
(b) India's largest Buddhist monastery is in Arunachal Pradesh
(c) The Khajuraho temples were built under Chandela Kings
(d) The Hoysalesvara temple is dedicated to Siva
Ans. (a)
Explanation:
The Statue of Gomateswara at Sravanbelagola represent '3rd' Jain Tirthankar.
The last and 24th Jain Tirthankara is 'Vardhamana Mahavira'. The 23rd Jain
Tithankara is 'parsvanath', and the 1st Tirthankara is Rishabhnath.
Match List I with List II and select the correct answer using the codes given
below the lists:
List I (Artist)
List II (Art)
A. Hiren Bhattacharya
1. Bharatanatyam dance
B. Malini Rajurkar
2. Hindustani vocal music
C. Pratibha Prahlad
3. Kuchipudi dance
D. Vempati Chinna
4. Puppetry Satyam
A B C D
A B C D
(a) 4 2 1 3
(b) 3 1 2 4
(c) 4 1 2 3
(d) 3 2 1 4
Ans. (c)
Explanation:
Hiren Bhattacharya - Puppetry
Malini Rajurkar - Bharatanatyam dance
Pratibha Prahlad - Hindustani vocal music
Vempati Chinna Satyam - Kuchipudi dance.
Chapachur Kut is a festival celebrated in the state of
(a) Arunachal Pradesh (b) Assam
(c) Mizoram
(d) Sikkim
Ans. (c)
Explanation:
Chapachur kut is a festival celebrated in the state of Mizoram.
2004
Who among the following is well known as an exponent of flute ?
(a) Debu Choudhary (b) Madhup Mudgal (c) Ronu Mazumdar (d)
Sahafaat
Ahmad
Ans. (c)
Explanation:

74

30.

31.

32.

33.

Ronu Mazumdar - Flute


Debu Choudhary - Sitar
Sahafaat Ahmed - Tabla
Madhup Mudgal - Vocal
The Archaeological Survey of India is an attached office of the
Department/Ministry of
(a) Culture
(b) Tourism
(c) Science and Technology
(d) Human Resource Development
Ans. (a)
Explanation:
The Archaeological Survey of India is an attached office of the Department of
Culture, Ministry of Culture and Tourism.
Consider the following statements :
1. The Islamic Calendar is twelve days shorter than the Gregorian Calendar.
2. The Islamic Calendar began in AD 632
3. The Gregorian Calendar is a solar calendar Which of the statements given
above is/are correct?
(a) 1 only
(b) 1 and 2
(c) 1 and 3
(d) 3 only
Ans. (d)
Explanation:
The Julian Year was 11 min. and 14 sec. longer than the Solar year. To make
the Vernal equinox occur on March 21, as it had in AD 325, the year of the First
Council of Nicaea, 'Pope Gregory XIII' issued a decree dropping 10 days from
the Calendar. To prevent further displacement he instituted a Calendar, known
as the Gregorian Calendar.
Islamic Calendar a lunar one used in most Muslim countries was started from
AD 622, the day after the Hegira, or emigration of Mohammad from Mecca to
Medina.
2006
Match List-1 with List-II and select the correct answer using the code given
below the lists:
List - I (Place of Archaeo)
List - II (State) logical Monument)
A. Sisupalgarh
1. Assam
B. Piprahwa
2. Manipur
C. Goalpara
3. Orissa
D. Bishnupur
4. Uttar Pradesh
A B C D
A B C D
(a) 2 4 1 3
(b) 2 1 4 3
(c) 3 4 1 2
(d) 3 1 4 2
Ans. (c)
Explanation:
Sisupalgarh - Orissa Piprahwa - Uttar Pradesh Goalpara - Assam Bishnupur Manipur
Which one of the following pairs is correctly matched ?
(a) Vikramsila Monastery
: Uttar Pradesh
(b) Hemkund Gurudwara : Himachal Pradesh
(c) Udayagiri Caves
: Maharashtra
(d) Amaravati Buddhist Stupa: Andhra Pradesh
Ans. (d)
Explanation:

75

34.

35.

36.

37.

Vikramsila Monastery - Bihar (50 km from Bhagalpur, on the foot of Hemkund


Lake).
Hemkund Gurudwara - Uttaranchal
Udaygiri Caves - Orissa (12 km from Bhubaneshwar)
Amaravati Buddhist Stupa - Andhra Pradesh
Who among the following is a Hindustani classical singer ?
(a) Geeta Chandran (b) Leela Samson
(c) Gangubai Hangal (d)
Swapnasundari
Ans. (c)
Explanation:
Geeta Chandran and Leela Samsonand are the exponents of Bharatanatyam
dance while Swapna Sundari is an exponent of Kuchipudi.
Which one of the following is not correctly matched ?
Prominent Indian Writer
Language
(a) RajaRao
: Telugu
(b) GobindTrimbukDeshpande
: Marathi
(c) Subramaniyam Bharati
: Tamil
(d) Tara Shankar Joshi
: Gujarati
Ans. (a)
Explanation:
Raja Rao is a prominent Indian Writer in English language. His literary works
are Kanthapura, The Serpent and the Rope, The Cat and the Shakespeare,
Comrade Kirillov.
Which one of the following pairs is not correctly matched ?
Monastery
State
(a) Dhankar Monastery :
Himachal Pradesh
(b) Rumtek Monastery
: Sikkim
(c) Tabo Monastery
:
Himachal Pradesh
(d) Kye Monastery
:
Arunachal Pradesh
Ans. (d)
Explanation:
'Kye Monastery' is situated in the Lahaul and Spiti district of Himachal Pradesh.
2007
Where is the famous Vijaya Vittala temple having its 56 carved pillars emitting
musical notes located ?
(a) Belur
(b) Bhadrachalam
(c) Hampi
(d) Srirangam
Ans. (c)
Explanation:
Vijaya VittaL Temple is one of the most exquisitely carved temples in Hampi.
The magnificently sculptured pillars are the marvel in the history of temple
architecture. Each of the pillars are different, which gives different musical
notes when hit with fingers.

5. INDIAN GEOGRAPHY
1.

1995
The word Hindu as reference to the people of Hind (India) was first used by
(a) the Greeks
(b) the Romans
(c) the Chinese
(d) the Arabs
Ans. (d)
Explanation:

76

2.

3.

4.

5.

6.

The term 'Hindu' is derived from "Sindhu". The Arabs and Persians pronounce
'S' as 'H' and thus called Sindhu as Hindu. So, the Arabs used the term 'Hindu',
for the first time to refer to the people of India.
The alpine vegetation in the western Himalayas is found only up to a height of
3000 meters while in the eastern Himalayas it is found up to a height of 4000
meters. The reason for this variation in the same mountain range is that
(a) eastern Himalayas are higher than western Himalayas
(b) eastern Himalayas are nearer to the equator and sea coast than western
Himalayas
(c) eastern Himalayas get more monosoon rainfall than the western Himalayas
(d) eastern Himalayas rocks are more fertile than the westerm Himalayas rocks
Ans. (c)
Explanation :
The Alpine vegetation in the eastern Himalayan region is found upto a height of
4000 meters. Because this region gets more monsoon rainfall than the western
Himalayas.
According to the 1991 census, the highest percentage of population in India is
to be found in the age group of
(a) 60 years and above
(b) 35 to 55 years
(c) 25 to 34 years
(d) 5 to 14 years
Arts. (d)
Explanation:
As per 1991 Census, the proportion of 6-14 age group population is 21-1%.
High proportion of young population is due to high birth rate and rapidly
declining infant and child mortalities. It forms part of dependent population.
Consider the map given below : The division along Indias coastal region
indicate
(a) coastal pollution zones
(b) salinity density isopleths
(c) the areas upto which sovereignty extends (d) under water relief contours
Ans. (d)
Explanation:
In the given map, the division along India's Coastal region indicates under
water relief contours.
In the map given four areas are differently shaded, three of which indicate
cereal production areas and one indicates non-cereal production area. As per
the index given, the non cereal crop production area is
(a) 1
(b) 2
(c) 3
(d) 4
Ans. (d)
Explanation :
It is the Rann of Kutch and Gujarat region and some parts of Maharashtra, noncereal producing area. It is Cotton and Jute producing area.
1. Wheat production area,
2. Rice producing area,
3. Cereais producing area.
Monoculture of commercially viable trees is destroying the unique natural
profile of .............
Thoughtless exploitation of timber, deforesting vast traets
for palm cultivation, destruction of mangroves illegal logging by tribals and
poaching only compound the problem. Fresh water pockets are fast drying up

77

7.

8.

9.

due to deforestation and destruction of mangroves. The place referred to this


quotation is
(a) Sunderbans
(b) Kerala coast
(c) Orissa coast
(d) Andaman & Nicobar Island
Ans. (a)
Explanation:
The place referred to this quotation is Sundarbans of West Bengal region. This
region is facing the above mentioned problems.
The palaeomagnetic results obtained from India indicated that In the past, the
Indian Sand mass has moved
(a) northward
(b) southward
(c) eastward
(d) westward
Ans. (a)
Explanation:
The paieomagnetic results obtained from India indicated that in the past, the
Indian landmass has moved northwards.
In which one of the following states of India it is legal for a Hindu male and
illegal for a Muslim male to have more than one living wife?
(a) Nagaland
(b) Mizoram
(c) Goa
(d) Arunachal
Pradesh
Ans. (c)
Explanation:
It is in Goa region, legal to have more than one living wife for a Hindu male,
whereas illegal for a Muslim male.
Of the four places marked 1, 2, 3, and 4 of the map, the one indicating a tidal
port is

(a) 1
(b) 2
(c) 3
(d) 4
Ans. (d)
Explanation:
It is Kandla, a tidal port with higher tide range located in Gujarat. (Hughli is
considered as
a riverine port).
10. In which one of the following crops international trade is low in the contest of
total produce?
(a) Rice
(b) Coffee
(c) Rubber
(d) Wheat
Ans. (a)
Explanation:
Indian International trade is low in the context of Rice, though the production is
high. It is because of the high level of consumption within the country, and also
not qualitative enough to be in International Competition.
11. The Indian subcontinent was originally part of a huge mass called
(a) Jurassic land mass
(b) Aryavarta
(c) Indiana
(d) Gondwana continent
Ans. (d)
Explanation:
Indian Subcontinent was originally part of Gondwana continent. It was
separated into South America, Africa, India, Australia and Antarctica. Indian
plate later merged with Eurasian plate in the north.
12. Which one of the following does not belong to biosphere reserves set-up so far
?

78

13.

14.

15.

16.

(a) Great Nicobar


(b) Sunderbans
(c) Nanda Devi
(d) Gulf
of
Kachchh
Ans. (d)
Explanation:
So far, 14 Biosphere reserves have been set up. They are-Nilgiri, Nanda Devi,
Nokrek, Great Nicobar, Gulf of Mannar, Manas, Sunderbans, Similipal, Dibru
Daikhowa, Dehong Debang, Panchmarhi, Kanchanjanga, Agasthyamalai and
Acnanakmar Amarkantak.
Which one of the following mountain ranges is spread over only one State in
India ?
(a) Aravalli
(b) Satpura
(c) Ajanta
(d) Sahyadri
Ans. (c)
Explanation :
In the given optionals, Ajanta ranges are smallest and spread over in
Maharashtra state only.
Aravalli to Gujarat and Rajasthan.
Satpura range to Maharashtra and Madhya Pradesh.
Sahyadri range (Western Ghats) to Maharashtra and Karnataka.
Arakan Yoma Is the extension of the Himalayas located in
(a) Baluchistan
(b) Myanmar
(c) Nepal
(d) Kashmir
Ans. (b)
Explanation :
South eastern ending of Himalayas is called Arakanyoma in Mayanmar.
Between Peguyoma and Arakan yoma, 'Irrawaddy', the mighty river of
Mayanmar flows.
The national highway from Amritsar to Calcutta via Delhi is numbered
(a) 1
(b) 2
(c) 4
(d) 8
Ans. (b)
Explanation:
From Delhi to Kolkata, National Highway 2 has a 1465 km long stretch.
Examine the map of Jammu and Kashmir given below:

The mountain ranges marked 1, 2, 3 and 4 are respectively :


(a) Ladakh, Zaskar, Karakoram and Pir Panjal
(b) Karakoram, Ladakh, Zaskar and Pir Panjal
(c) Karakoram,_Zaskar, Pir Panjal and Ladakh
(d) Ladakh, Pir Panjai, Karakoram and Zaskar
Ans. (b)
Explanation:
1 is Karakoram range
2 is Ladakh rasge.
3 is Zaskar range
4 is Pir Panjai range.
17. In the map given below, three out of the four places marked indicates places
where a thermal power station is located:
The place where there is no thermal power station is marked
(a) 1
(b) 2
(c) 3
(d) 4
Ans. (c)
Explanation:
Nagarjuna Sagar (3) is a hyctel power project in Andhra Pradesh.
1996

79

18. Which one of the following rivers thrice forks into two streams and reunites a
few miles farther on, thus forming the islands of Srirangapattanam,
Sivasamudram and Srirangam ?
(a) Cauvery
(b) Tungabnadra
(c) Krishna
(d) Godavari
Ans. (a)
Explanation:
The river Cauvery thrice forks into two streams and reunites a few miles farther
on, thus forming the islands of Srirangapattanam, Sivasamudram and
Srirangam in the eastern part of Tamil Nadu.
19. As per the 1991 Census, which one of the following states has a lower
population density than the other three
?
(a) Sikkim
(b) Nagaland
(c) Meghalaya
(d) Manipur
Ans. (a)
Explanation:
As per 1991 Census, Sikkim has a lower population density of 57. (in the given
optionals). Whereas Nagaland has 75, Meghalaya 79 and Manipur has 82.
20. In the Indian context, the term .De-notified tribes: refers to :
(a) tribes which are aboriginals
(b) nomadic tribes
(c) tribes practising shifting cultivation
(d) tribes which were earlier classified as criminal tribes
Ans. (d)
Explanation:
In the Indian context, de-notified tribes refers to tribes which were earlier
classified as criminal tribes,
21. Which one of the following regions of India is now regarded as an ecological
hot spot ?
(a) Western Himalayas
(b) Eastern Himalayas
(c) Western Ghats
(d) Eastern Ghats
Ans. (c)
Explanation:
Western Ghats
22. High temperature and low pressure over the Indian Sub-Continent during the
summer season draws air from the Indian Ocean leading to the In-blowing of
the
(a) Southeast Monsoon
(b) Southwest Monsoon
(c) Trade winds
(d) Westerlies
Ans. (b)
Explanation:
As a result of rapid rise of temperature and low pressure over the Indian Subcontinent during May (Summer season) sucks in air from the Arabian Sea and
Bay of Bengal. These are the South-West Monsoon, which are the major
source of rains in India.
23. Saddle peak the highest peak of Andaman and Nicobar Islands is located in
(a) Great "Nicobar
(b) Middle Andaman (c) Little Andaman (d)
North
Andaman
Ans. (d)
Explanation:
Highest point in Andaman Group : Saddle Peak (732 m) is present in North
Andaman Island.

80

24. Given below are two statements; one labelled as Assertion (A) and the other
labelled as Reason (R) :
Assertion (A) : Mangroves are very specialised forest eco-systems of tropical
and sub-tropical regions bordering certain sea coasts.
Reason (R) : They stabilise the shoreline and act as bulwark against
encroachments by sea.
In the context of the above two statements, which one of the following is correct
?
(a) Both A and R are true and R is the correct explanation of A
(b) Both A and R are true but R is not a correct explanation of A
(c) A is true but R is false
(d)
A is false but R
is true
Ans. (b)
Explanation:
Both these statements are correct but R is not the reason for A.
Mangroves are specialised forest eco-systems, which stabilise the shoreline
and act as bulwark against encroachments by Sea. They are in the tropical and
sub tropical regions bordering certain sea coasts.
25. Match List I (Ores) with List II (States where they are' mined) and select the
eorrest answer using the codes given below the lists :
List I
List II
A. Manganese
1. Madhya Pradesh
B. Nickel
2. Orissa
C. Lead-zinc
3. Rajasthan
D. Asbestos
4. Andhra Pradesh
A B C D
A B C D
(a) 1 3 2 4
(b) 4 3 2 1
(c) 1 2 3 4
(d) 4 2 3 1
Ans. (c)
Explanation:
Large producers of Manganese ore are Orissa (33-3%), Madhya Pradesh (211%), Maharashtra and Karnataka. Nickel is produced more in Orissa and
Jharkhand.
Lead and Zinc are produced more in Rajasthan. Next to it, Gujarat,
Maharashtra, West Bengal. Almost all Zinc is produced in Rajasthan.
Asbestos is produced in Southern parts of Andhra Pradesh.
26. Which of the following pairs are correctly matched ?
1. Idukki
: Thermal power station
2. Sabarigiri
: Hydro-electric project
3. Ghatprabha
: Irrigation project
4. Ramganga
: Multipurpose project
(a) 2, 3 and 4
(b) 1, 2, 3 and 4
(c)3 and 4
(d) 1 and 2
Ans. (a)
Explanation:
Sabarigiri is a hydro-electric project located in Kerala on Pamba Kaki river.
Idukki project on Periyar river is a Hydel power project located in Kerala.
Ramganga project is a multi-purpose project located in Uttar Pradesh on Ram
Ganga River.
Ghatprabha Valley Scheme on River Ghatprabha Is a major irrigation project
located in Karnataka,

81

27.

28.

29.

30.

31.

So, in the given optionais, 2, 3, 4 are correct and i is wrong.


Physico-chemical characteristics of water in water sources undergo changes
due to:
(a) aquatic macrophytes
(b) aquatic fungi
(c) effluents
(d) evapotranspiration
Ans. (c)
Explanation:
Effluents from various industrial units and sewage are the major reasons which
change the physico-chemical characteristics of water.
(a) and (b) are natural organisms, (d) is a natural process through which plants
loose water for thermoregulation.
Which one of the following is an important crop of the Barak Valley ?
(a) Jute
(b) Tea
(c) Sugarcane
(d) Cotton
Ans.. (a)
Explanation:
Barak valley in Assam has Jute as its important crop.
Consider the following programmes :
1. Afforestation and development of wastelands
2. Reforestation and replantation in existing forests
3. Encouraging the wood substitutes and supplying other types of fuel
4. Promotion of wide use of insecticides and pesticides to restrict the loss of
forest area from degradation caused by pests and insects.
The National Forest Policy of 1988 includes
(a) 1, 2, 3 and 4
(b) 2 and 4
(c) 1, 3 and 4
(d) 1, 2 and 3
Ans. (d)
Explanation:
India's forest policy of 1894 revised in 1952 and again in 1988 includes :
(i) Maintenance of environmental stability through ecological balance.
(ii) Check on soil erosion and extension of sand dunes,
(iil) Afforestation and social forestry.
(iv)
Reforestration
(v) Encouraging wood substitutes.
So, 1, 2, 3 are correct but not 4.
Consider the following statements:
At the present level of technology available in India, solar energy can be
conveniently used to
1. supply hot water to residential buildings
2. supply water for minor irrigation projects
3. provide street lighting
4. electrify a cluster of villages and small towns
(a) 1, 2, 3 and 4 are correct
(b) 2 and 4 are correct
(c) 1 and 3 are correct
(d) 1, 2 and 3 are correct
Ans. (a)
Explanation:
At the present level of technology available in India, Solar energy can be used
to supply hot water to residential buildings, Supply water for minor irrigation
projects, provide street lighting, and electricity to a cluster of villages and small
towns, through solar photo yoltaic systems.
Local supply of coal is not available to
(a) TISCO, Jamshedpur
(b) VSL, Bhadravati

82

32.

33.

34.

35.

36.

(c) HSL, Durgapur


(d) HSL, Bhilai
Ans.(b)
Explanation:
VSL Bhadravati is the only Iron and Steel plant situated outside the coal
producing area. So, the energy requirement can be met by Hydro-electric
power from Mahatma Gandhi H.E.P.
Which one of the following areas of India produces largest amount of cotton ?
(a) North western India and Gangetic West Bengal
(b) North western and Western India
(c) Western and Southern India
(d) Plains of Northern India
Ans.
(c)
Explanation:
Largest producers of cotton are Gujarat and Maharashtra of Western part of
India and some southern parts of India (remaining parts in the given optionals
are rich producers of Cereals).
You might see a few curious Danes around, but that is because........used to
be Danish outpost.
This quaint town with its fort and a beautiful church, the New Jerusalem, empty
streets and deserted beachfront is a quaint gem. The place referred to in this
quotation lies on the
(a) Tamil Nadu coast (b) Kerala coast
(c) Karnataka coast (d) Goa coast
Ans. (b)
Explanation :
Kerala coast.
The Palk Bay lies between
(a) Gulf of Kachehh and Gulf of Khambhat (b) Gulf of Mannar and Bay of
Bengal
(c) Lakshadweep and Maldive Islands
(d) Andaman and Nicobar Islands
Ans. (b)
Explanation:
Palk Bay lies between Gulf of Mannar and Bay of Bengal. It is situated in the
Southern most region of India, between Sri Lanka and India.
Consider the following rivers :
1. Kishenganga
2. Ganga
3. Wainganga
4. Penganga
The correct sequence f these rivers when arranged in the north-south
direction is
(a) 1, 2, 3, 4
(b) 2, 1, 3, 4
(c)2, 1, 4, 3
(d) 1, 2, 4, 3
Ans. (b)
Explanation:
Ganga is the northern most river flows mainly through Uttar Pradesh and Bihar.
Kishenganga lies south to Ganga.
Wainganga and Penganga are the tributaries of Godavari located in
Maharashtra.
Which of the following pairs of states and their important crops are correctly
matched?
1. Kerala
: Tapioca
2. Maharashtra
: Cotton
3. West Bengal
: Jute
4. Gujarat
: Groundnut
(a) 1, 2 and 3
(b) 1, 2 and 4
(c) 1, 3 and 4
(d) 2, 3 and 4

83

Ans. (d)
Explanation:
Maharashtra, West Bengal, Gujarat are important crop growing regions of
Cotton, Jute, Groundnut respectively. So 2, 3 and 4 are correct.
37. As per the 1991 Census, the average size of households in terms of number of
persons per household in respect of the given states follows the sequence
(highest first, lowest fast) :
(a) Haryana, Uttar Pradesh, Kerala, Tamil Nadu
(b) Uttar Pradesh, West Bengal, Gujarat, Kerala
(c) Gujarat, Haryana, Uttar Pradesh, West Bengal
(d) Uttar Pradesh, West Bengal, TamilNadu, Kerala
Ans. (d)
Explanation:
As per the 1991 Census, the ayerage size of household is highest in Uttar
Pradesh > West Bengal > Tamil Nadu > Kerala.
1997
38. Match List I with List II and select the correct answer using the codes given
below the lists:
List I (Climatic conditions)
List II (Reasons)
A. Madras is warmer than Calcutta
1. North-East Monsoon
B. Snowfall in Himalayas
2.
Altitude
C. Rainfall decreases from West Bengal 3. Western depressions
to Punjab
D. Sutlej-Ganga Plain gets some rain in 4. Distance from sea
winter
5. Latitude
A B C D
A B C D
(a) 1 2 4 5
(b) 4 5 1 3
(c) 5 2 4 3
(d) 5 1 3 4
Ans. (c)
Explanation:
Madras is warmer than Calcutta, because it is located towards the equator.
Snowfall in Himalaya is due to altitude because increase in the altitude causes
decrease in the temperature.
Rainfall decreases from Coastal regions to interior land regions, as the
moisture laden winds looses most of their moisture in the regions nearer to the
Sea.
Winter rainfall in North India is due to Western
depressions.
39. Match List I with List II and select the correct answer using the codes given
below the lists:
List I
List II
A. Deccan Traps
1. Late cenozoic
B. Western Ghats
2. Pre-cambrian
C. Aravalli
3. Cretaceous Eocene
D. Narmada-Tapi alluvial deposits
4. Cambrian
5. Pleistocene
A B C D
A B C D
(a) 3 5 1 4
(b) 3 1 2 5
(c) 2 1 3 4
(d) 1 4 2 5
Ans. (b)
Explanation :

84

40.

41.

42.

43.

44.

Deccan traps were formed in Cretaceous Eocene age.


Western Ghats were formed In Late Cenozolc age. Aravaliis were formed in
Pre-Cambrian age.
Narmada-Tapi were formed in Pleistocene age.
The January isotherm taken as a basis for dividing India into tropical and
subtropical zones is
(a)21C
(b) 1BC
(c)12C
(d) 15C
Ans. (c)
Explanation:
The January isotherm of 12C is taken as a basis for dividing India into tropical
and sub-tropical zones.
What is the correct sequence of the descending order of the following States in
respect of female literacy rates as per the 1991 census ?
1. Mizoram
2. Kerala
3. Goa
4. Nagaland
Choose the correct answer using the codes given below :
(a) 2, 3, 1, 4
(b) 2, 1, 3, 4
(c) 2, 3, 4, 1
(d) 1, 2, 4, 3
Ans. (d)
Explanation:
According to 2001 Census, female literacy rate in Kerala is 87-7%, in Mizoram :
86-7%, in Goa : 75-3% and in Nagaland : 61-46%.
So order is : Kerala > Mizoram > Goa > Nagaland.
Match List I with List II and select the correct answer using the codes given
below the lists:
List I (Minerals)
List
II
(Topical areas
of
occurrence)
A. Coal
1. Bhandara
B. Gold
2. Karanpura
C. Mica
3. Hirtti
D. Manganese
4. Neliore
A B C D
A B C D
(a) 1 3 2 4
(b) 2 3 4 1
(c) 3 4 2 1
(d) 2 1 4 2
Ans. (b)
Explanation:
Coal mines are rich in Jharia, Giridih and Karnanpura of Jharkhand.
Gold mines are rich in Kolar and Hutti regions of Karnataka. Mica is available in
Vishakhapatnam, Nellore and Krishna Districts of Andhra Pradesh.
Manganese is available in Bhandara and Ratnagiri districts of Maharashtra.
According to the latest census figures sex ratio in India is
(a) declining
(b) stable
(c) increasing
(d) fluctuating
Ans. (b)
Explanation:
According to the 1991 Census, Sex ratio is declining. According to the 2001
Census, Sex ratio is increasing. Because Sex ratio in 1981 is 934.
Sex ratio in 1991 is 926.
Sex ratio in 2001 is 983.
The Aiamatti is on the river
(a) Godavari
(b) Cauvery
(c) Krishna
(d) Mahanadi
Ans. (c)

85

Explanation:
Alamatti dam is on the river Krishna.
45. Consider the map given below :

46.

47.

48.

49.

The predominant languages spoken in the areas marked A, B, C and D are


respectively
(a) Brajbhasha, Bhojpuri, Maithili and Awadhi
(b) Awadhi, Maithili, Bhojpuri and Brajbhasha
(c) Brajbhasha, Awadhi, Bhojpuri and Maithili
(d) Pvlaithili, Bhojpuri, Brajbhasha and Av/adhi
Ans. (c)
Explanation:
The area marked by 'A' is Mathura region and there the Braj Bhasha is spoken.
B is Lucknow region, and Awadhi is spoken there.
'C' and 'D' region are in North Bihar, there Bhojpuri and Maithili are spoken
respectively.
The canal-carrying capacity of Farrakka is
(a) 75,000 Cusecs (b) 70,000 Cusecs (c) 40,000 Cusecs (d)
35,000
Cusecs
Ans. (a)
Explanation:
75,000 cusecs
Which one of the following pairs is correctly matched ?
(a) Teak
: Jammu and Kashmir
(b) Deodar
: Madhya Pradesh
(c) Sandalwood
: Kerala
(d) Sundari
: West Bengal
Ans. (d)
Explanation:
Sundari trees are abundant in Sunderban delta of West Bengal.
Which one of the following is cultivated by transplanting seedlings ?
(a) Maize
(b) Sorghum
(c) Onion
(d) Soyabean
Ans. (c)
Explanation :
Onion is cultivated by transplanting seedlings, whereas Maize, Sorghum and
Soyabean are cultivated through direct seeding.
Consider the map given below :
The places marked A, B, C and D in the map are respectively
(a) Rift valley region, Chattisgarh plain, Rain shadow region and Chotanagpur
plateau
(b) Chattisgarh plain, Chotanagpur plateau, Rift valley region and Rain shadow
region
(c) Rift valley region, Chattisgarh plain, Chotanagpur plateau and Rain shadow
region
(d) Chattisgarh plain, Rain shadow region, Chota nagpur plateau and Rift valley
region
Ans. (c)
Explanation:
The place marked as 'A' is rift valley region of Narmada and Tapti rivers.

86

B' is Chattisgarh plain formed by Mahanadi river.


'C' is Chota Nagpur Plateau of Jharkhand.
D' is Rainshadow region of Western Ghats.
50. Consider the map given below :

51.

52.

53.

54.

The places marked A, B, C and D in the map are respectively known for the
cultivation of
(a) groundnut, ragi, tobacco and sugarcane
(b) groundnut, sugarcane, ragi and tobacco
(c) ragi, sugarcane, groundnut and tobacco
(d) ragi, groundnut, sugarcane and tobacco
Ans. (b)
Explanation:
'A' is Gujarat region known for groundnut cultivation.
B' is known for Sugarcane cultivation in Western Maharashtra.
'C' is Orissa region - Ragi cultivation.
'D' is Andhra Pradesh region - Tobacco cultivation.
Which of the following places are known for paper manufacturing industry ?
1. Yamunanagar
2. Guwahati
3. Shahabad
4. Ballarpur
Choose the correct answer using the codes given below :
(a) 1, 2 and 3
(b) 1, 2 and 4
(c) 1, 3 and 4
(d) 2, 3 and 4
Ans. (b)
Explanation:
Yamunanagar, Guwahati and Ballarpur.
The tribal population in Andaman and Nicobar Islands belongs to the
(a) Australoid race
(b) Caucasoid race (c) Mongoloid race (d) Negroid
race
Ans. (d)
Explanation:
The tribes Onges and Jarwas of Andaman and Nicobar Islands belong to the
Negroid race.
The following are the major oilseeds produced in India.
1. Sesamum
2. Mustard
3. Groundnut
4. Soyabeen
Which one of the following is the correct sequence of the descending order of
the quantity of their production ?
(a) 1, 2, 3, 4
(b) 3, 2, 4, 1
(c) 2, 4, 3, 1
(d) 3, 4, 2, 1
Ans. (c)
Explanation:
Mustard (8 mt) > Soyabean (7 rnt) > Groundnut (6-5 mt) > Sesamum.
Match List ! (Hazardous industries using child labour leading to the filing of a
public interest petition in the Supreme Court) with List II (Located at) and select
the correct answer using the codes given below the lists :
List I
List II
A. Glass Industry
1. Moradabad
B. Brassware Industry
2. Marakpur
C. Slate Industry
3. Ferozabad
D. Handmade Carpet Industry
4. Mirzapur
A B C D
A B C D
(a) 3 1 2 4
(b) 1 3 4 2
(c) 3 1 4 2
(d) 1 3 2 4

87

Ans. (a)
Explanation:
A. Glass Industry - Ferozabad
B. Brassware Industry - Moradabad
C. Slate Industry - Mirzapur
D. Handmade Carpet Industry Marakpur
55. In the map shown in the given figure, river labelled as 1, 2, 3 and 4 respectively
(a) Kosi, Gomati, Ghaghara and Gandak (b) Kosi,
Ganga,
Gomati
and
Ghaghara
(c) Gandak, Ganga, Gomati and Ghaghara (d) Teesta, Gomati, Ghaghara and
Kosi
Ans. (a)
Explanation:
The rivers marked in the map are
1. Kosi
2. Gomati
3. Ghaghara
4. Gandak
56. The water pollution in river is measured by the dissolved amount of
(a) Chlorine
(b) Ozone
(c) Nitrogen
(d) Oxygen
Ans. (d)
Explanation:
The water pollution in rivers is measured by the dissolved amount of oxygen.
Polluted water has high BOD, i.e., less amount of oxygen is dissolving in it.
57. What is the correct' sequence of the following Indian states in descending order
of their length of surface roads per 100 km2 of their area ?
1. Haiyana
2. Maharashtra
3. Punjab
4. Tamil Nadu
(a) 4, 3,2,1
(b) 4, 3, 1, 2
(c)3,4, 1,2
(d) 3, 4, 2, 1
Ans. (d)
Explanation :
Punjab - Tamil Nadu - Maharashtra - Haryana.
58. The four railway junctions shown- by numerals 1, 2, 3, 4 on the rough outline
map of Gujarat are respectively
(a) Patanpur, Mahasena, Ahmedabad and Vadodara
(b) Mahasena, Surendranagar, Rajkot and Junagarh
(c) Palanpur, Kandla, Bhuj and Okla
(d) Ahmedabad, Vadodara, Bhavnagar and Broach
Ans, (b)
Explanation :
The Railway junctions marked as 1, 2, 3, 4 are Mahasena,. Surendranagar,
Rajkot and Junagarh.
59. Which one of the following, is the correct sequence of the states (labelled 1, 2,
3 and 4) of India shown on the map in descending order In terms of their
available ground water-resources for irrigation ?
(a) 3, 4, 1, 2
(b) 3, 4, 2, I
(c) 4, 3, 1, 2
(d) 4, 3, 2, 1
Ans. (d)
Explanation:
In terms of available ground water resources for irrigation, the decreasing order
is : '4' Uttar Pradesh (7.25 m ha.m)> 3' Maharashtra (2.54 m. ha.m) > '2 West
Bengal (1.96 m. ha.rn) > '1' Assam (1.91 m. ha.m)

88

60. Among the Indian states shown labelled I, 2, 3 and 4 in the rough outline map
given, the correct sequence of descending order of percent of scheduled tribe
population to their total population is :

61.

62.

63.

64.

(a) 1, 3, 2, 4
(b) 3, 1, 2, 4
(c) 3, 1, 4, 2
(d) 1, 3, 4, 2
Ans. (b)
Explanation:
The descending order of given states, according to their percent of S.T.'s
population to their total population is -Mizoram (94.75%) > T Arunachal
Pradesh (63.66%) > 2' Manipur (34.41%) > Orissa (22.2%).
Which one of the following sets orstates stands to benefit the most from the
Koiikan Railway?
(a) Goa, Karnataka, Maharashtra, Kerala (b) M.P., Maharashtra, Tamil "Nadu,
Kerala
(c) Tamil Nadu, Kerala, Goa, Maharashtra (d) Gujarat, Maharashtra, Goa, Tamil
Nadu
Ans. (a)
Explanation:
The States of Goa, Karnataka, Maharashtra, and Kerala would benefit most
from the Konkan railway, as it passes through these states.
Which one of the following languages belongs to the Austria group ?
(a) Marathi
(b) Ladakhi
(c) Khasi
(d) Tamil
Ans. (c)
Explanation:
Khasi and Nicobsrese languages belongs to Monkhrrver branch, of Austric
languages.
Ladakhi belongs to Sino-Tibetan family.
Marathi belongs to Indo-Aryan language family.
Tamil belongs to Dravidian language family.
Which one of the following east flowing rivers of India has rift valley due to
down warping ?
(a) Damodar
(b) Mahanadi
(c) Son
(d) Yamuna
Ans. (b)
Explanation :
Mahanadi, the east flowing river of India has rift valley due to down warping.
Forest areas have been labelled as 1, 2, 3 and 4 in the rough outline map given
:

Among these, those which were threatened in 1997 by a serious epidemic


include
(a) teak forests of 3 and 4
(b) oaks forests of 1 and sal forests
of 2
(c) sal forests of 3
(d) sandalwood forests of 4
Ans. (c)
Explanation:
The place marked '3' indicates sal trees of Madhya Pradesh. They were
affected in 1997 by the insects.
65. Commercial production of mineral oil has started recently in which one of the
areas of India, labelled 1, 2, 3 and 4 in the rough map given below :

89

66.

67.

68.

69.

(a) 1
(b) 2
(c) 3
(d) 4
Ans. (c)
Match List-i with List-II and select the' correct answer using the codes given
below the lists:
List I (Minerals)
List II (Mining area)
A. Graphite
1. Bellaiy
B. Lead
2. Didwana
C. Salt
3. Rampa
D. Silver
4. Zawar
A B C D
A B C D
(a) 3 4 1 2
(b) 1 4 2 3
(c) 3 1 4 2
(d) 2 3 1 4
Ans. (b)
Explanation:
A. Graphite - Beliary B. Lead - Zawar
C. Salt - Didwana
D.
Silver
Rampa
Lead ore - Zawar in Udaipur district in Rajasthan Silver ore - Beliary district in
Karnataka, Baramula in J & K, Almora in U.P., Cuddapah, Guntur Kurnool in
Andhra Pradesh.
Graphite - Rampa Chodavaram, E. Godavari District in Andhra Pradesh.
Salt - Salt lakes of Sambhar in Jaipur, Dindwana, Phalodi and Pachbhadra in
Jodhpur of Rajasthan.
The discovery of Oak flora in 1966 added a new chapter to the history of Indian
Sericulture.
Winch one oi the following states is the leading producer of Oak tasar silk ?
(a) Assam
(b) Bihar
(c) Manipur
(d) Orissa
Ans. (b)
Explanation:
Bihar is the leading producer of Oak tasar silk.
Match List-I with List-II and select the correct answer using the codes given
below the lists:
List I (Places)
List II (Industries)
A. Jamnagar
1. Aluminium
B. Hospet
2. Woollen Textile
C. Korba
3. Fertilizers
D. Haldia
4. Cement
5. Iron and Steel
A B C D
A B C D
(a) 4 3 1 2
(b) 2 5 1 3
(c) 4 5 2 1
(d) 2 1 4 3
Ans. (b)
Explanation:
Kanpur, Vadodara, Jamnagar etc. are important centres of Woollen industry.
Korba - Aluminium industry (BALCO).
Hospet (in Bellary district) - The Vijainagar Steel plant. Haldia - Fertilizer
Corporation of India Ltd.
In the vicinity of Mumbai, a number of specialised towns have been developed.
Match the lists of specialisation with towns and select the correct answer using
the codes given below the lists :
List I (Towns)
List II (Specialisation)

90

A. Alibag
1. Fishing Centre
B. Balapur
2. Holiday resort
C. Mhava Sheva
3. Petro-Chemical complex
D. Ratnagiri
4. Port
A B C D
A B C D
(a) 1 3 2 4
(b) 2 3 4 1
(c) 3 4 2 1
(d) 2 1 4 3
Ans. (d)
Explanation:
A. Alibag - Holiday Resort
B. Balapur - Fishing Centre
C.j Nhava Sheva - Port
D. Ratnagiri
Petro-Chemical
complex
70. Match List-I with List-II and select the correct answer using the codes given
below the lists:
List I (Agricultural products)
List II (Foremost producer)
A. Cotton
1. Madhya Pradesh
B. Gram
2. Gujarat
C. Black pepper
3. West Bengal
D. Pineapple
4. Kerala
A B C D
A B C D
(a) 2 1 4 3
(b) 2 1 3 4
(c) 1 2 4 3
(d) 1 2 3 4
Ans. (a)
Explanation:
Gram - Madhya Pradesh
Cotton - Gujarat and Maharashtra
Black Pepper (Spices) - Kerala, Karnataka and Tamil Nadu
Pineapple - West Bengai
71. The rough outline map given shows centres if cement industry labelled 1, 2, 3
and 4. Match these centres with the following sets of names
Names:
A. Katni
B. Tirunelveli
C. Sikka
D. Churk
A B C D
A B C D
a) 3 4 2 1
b) 2 4 1 3
c) 1 2 4 3
d) 2 3 1 4
Ans. (b)
Explanation:
Katni in Madhya Pradesh.
Tirunelveli in Kerala Sikka in Gujarat.
Churkin Uttar Pradesh are the important centres of cement industry.
72. Which one of the following ports shown on the rough outline map of India is a
riverine port
(a) 1
(b)2
(c) 3
(d)4
Arts. (d)
Explanation:
Hugli in West Bengal is a riverine port.
73. Match the following research institutes A, B, C and D with their respective
location labelled as 1 to 6 in the given rough outline map :

91

74.

75.

76.

77.

78.

A. Central Drug Research Institute


B. National Atlas and Thematic Mapping Organisation
C. National Institute of Ocean Technology
D. Temperate Forest Research Centre
Select the correct answer using the codes given below..
A B C D
A B C D
(a) 4 3 2 6
(b) 4 5 1 6
(c) 2 3 4 5
(d) 1 6 2 3
Ans. (a)
Explanation:
A.-Lucknow, B.-Kolkata, C.-Chennai, D.-Shimla
Some people in Manipur live in houses built on floating islands of weeds and
decaying vegetation held together by suspended silt. These islands are called
(a) Tipis
(b) Burkhans
(c) Phoomdis
(d) izba
Ans. (c)
1999
The Rawa offshore block, with great potential for oil, is located in
(a) Krishna-Godavari basin
(b) Cauvery basin
(c) Mahanadi basin
(d) Polar-Pennar basin
Ans. (a)
Explanation:
The Rawa offshore block, with great potential for oil, is located in KrishnaGodavari basin of Andhra Pradesh.
Among which one of the following sets
of social/religious groups is the
extent of poverty the highest, as per Government statistics for the nineties ?
(a) Muslims in Kerala, Gujarat and A.P.
(b) Tribals in Bihar, Orissa, M.P. and Maharashtra
(c) Scheduled Castes in Punjab, Western UP, northern Rajasthan and Tamil
Nadu
(d) Christians in Gujarat, Maharashtra and Assam
Ans. (b)
Explanation :
Tribals in Bihar, Orissa, Madhya Pradesh and Maharashtra.
The minimum land area recommended for forest cover to maintain proper
ecological balance in India is
(a) 25%
(b) 33%
(c) 43 %
(d) 53 %
Ans. (b)
Explanation:
The minimum land area recommended for forest cover to maintain proper
ecological balance in India is one-third of the area of the country (i.e. 33%).
With this objective NFAP (National Forestry Action Programme) has been
formulated.
Which one of the following statements is not true of the Konkan Railway ?
(a) The total length is about 760 km
(b) It runs through the states of Karnataka, Goa, Maharashtra and Kerala
(c) It is the only rail route that cuts across the Western Ghats
(d) The Konkan Railway Construction Company which came into being raised
money through Public Issues
Ans. (b)

92

Explanation:
It runs through Maharashtra, Goa and Karnataka, but not in Kerala.
79. The given figure shows a portion of Southern India. The proposed site
(Koodankulam) for the construction of two 1000 MW nuclear power plants has
been labelled in the map as
(a) A
(b) B
(c) C
(d) D
Ans. (b)
Explanation:
India had entered into an agreement with the Russian Federation for setting up
an atomic power station comprising jtwo pressurised water reactors of 1,000
MWa capacity each, at Kudankulam in Tamil Nadu.
80. Match List I with List II and select the correct answer using the codes given
below the lists:
List I (Resorts)
List II (States)
A. Chakrota
1. Assam
B. Haflong
2. West Bengal
C. Kalimpong
3. Uttar Pradesh
D. Kufri
4. Himachal Pradesh
Codes :
A B C D
A B C D
(a) 1 3 2 4
(b) 3 1 4 2
(c) 3 1 2 4
(d) 1 3 4 2
Ans. (c)
Explanation:
Chakrota is located in Uttar Pradesh.
Haflong is located in Assam Kalimpong is located in West Bengal Kufri is
located in Himachal Pradesh
81. In the given figure, the site of the Tehri dam has been labelled as
(a) A
(b) B
(c) C
(d) D
Ans. (c)
Explanation:
Tehri dam is on Bhagirathi riyer, in Uttaranchal state. (The river is called
Bhagirathi above Devprayag town, and below it the Ganga. The Alaknanda an
important affluent joins it at Devprayag town. So, Tehri is located above
Devprayag, and on Bhagirathi river).
82. In which one of the following areas in the given snap was there a recent
discovery of cop
per deposits by the Atomic Minerals Division of
Department of Atomic Energy ?
(a) 1
(b) 2
(c)3
(d) 4
Aas. (c) ____________________________________________________
83. Match List I with List II and select the correct answer using the codes given
below the lists
List I (Industries)
List II (Industrial Centres)
A. Pearl fishing
1. Pune
B. Automobiles
2. Tuticorin
C. Shipbuilding
3. Pinjore
D. Engineering goods
4. Marmagao

93

A B C D
A B C D
(a) 2 1 4 3
(b) 2 1 3 4
(c) 1 2 4 3
(d) 1 2 3 4
Ans. (a)
Explanation:
A. Pearl fishing - Tuticorin
B. Automobiles - Pune
C. Ship building - Marmagao
D. Engineering goods - Pinjore
84. The first marine sanctuary in India, having within its bounds coral reefs,
mollusca, dolphins, tortoises and various kinds of sea birds, has been
established in
(a) Sunderbans
(b) Chilka Lake
(c) Gulf of Kachchh (d)
Lakshadweep
Ans. (b) _____________________________________________________
Explanation:
Chilka lake is the first marine sanctuary in India having within its bounds coral
reefs, mollusca, dolphins, tortoises, 3nd various kinds of sea birds. It is located
in Orissa.
85. The population growth rate in Kerala is the lowest among major Indian states.
Which one of the following is the most widely accepted reason for this ?
(a) Kerala has made the highest investment. in family planning
(b) Kerala has the highest literacy rate in India
(c) Kerala has invested heavily in promoting literacy and public health and
placed high priority on social policies
(d) The population pyramid in Kerala has relatively fewer women in the
reproductive age group
Ans. (c)
Explanation:
It is due to high literacy rate, more urban population and more developed
economy. The birth rate is competitively lower.
86. Match the areas shown as A, B, C and D on the given map showing with the
largest religious minorities. Select the correct answer using the codes given
below the list of minorities :
Largest Religious Minorities
1. Buddhists
2. Christians
3. Jains
4. Muslims
5. Sikhs
A B C D
A B C D
(a) 5 1 3 2
(b) 4 3 2 1
(c) 5 3 1 2
(d) 4 2 1 3
Ans. (c)
Explanation:
A. Punjab and neighbouring district - Sikhs.
B. Rajasthan - Jains.
C. Maharashtra - Buddhists.
D. Andhra Pradesh - Christians.
87. Which one of the following pairs of states and tribes is not correctly matched ?
(a) Assam
: Miri
(b) Nagaland
: Konyak
(c) Arunacha! Pradesh :
Apatani
(d) Madhya Pradesh : Lambada
Ans. (*)

94

Explanation:
Assam - Mikir, Bodos, Abhor, Angame Nagas Nagaland - Nagas Arunachal
Pradesh - Apatamis Madhya Pradesh - Baiga, Kol, Muriyas.
88. As per 1991 Census, which one of the following groups of Union Territories had
the highest literacy rate ?
(a) Chandigarh and Dadra & Nagar Haveli
(b) Delhi and Andaman & Nicobar Islands
(c) Andaman & Nicobar Islands & Pondicherry
(d) Pondicherry and Delhi
Ans. (b)
Explanation:
Delhi and Andaman and Nicobar Islands had the highest literacy rates
according to 2001 Census, with 81.67, 81.30 respectively.
89. In the rough outline map of a part of Jammu and Kashmir shown in the figure,
places marked A, B, C and D represent respectively
(a) Anantnag, Baramula, Srinagar and Kargil
(b) Baramula, Srinagar, Kargil and Anantnag
(c) Baramula, Srinagar, Anantnag and Kargil
(d) Srinagar, Baramula, Kargil and Anantnag
Ans. (c)
Explanation:
Places marked A,B, C and D are Baramula, Srinagar, Anantnag and Kargil
respectively.
90. Match List I with List II and select the correct answer using the codes given
below the lists:
List I (Rivers)
List II (Dams)
A. Cauvery
1. Alamatti
B. Krishna
2. Mettur
C. Narmada
3. Gandhi Sagar
D. Chambal
4. Sardar Sarovar
A B C D
A B C D
(a) 1 4 2 3
(b) 2 1 4 3
(c) 2 1 3 4
(d) 1 3 4 2
Ans. (b)
Explanation:
A. Mettur project on the river Cauveri in TamilNadu.
B. Aiamatti dam on the river Krishna.
C. Sardar Sarovar Project on Narmada River.
D. Gandhi Sagar Project on Chambal river in Madhya Pradesh.
91. Consider the following statements :
Assertion (A) : According to statistics, more female children are bom each
year than male children in India.
Reason (R) : In India, the death rate of a male child is higher than that of the
female child.
(a) Both A and R are true and R is the correct explanation of A
(b) Both A and R are true but R is NOT the correct explanation of A
(c) A is true but R is false
(d) A is false but R is true
Ans. (*)
Explanation:

95

92.

93.

94.

95,

Both Statements are wrong.


Causes of decling sex ratio.
1. More males are born than females.
2. More females dies at infancy and during the reproductive period (Death rate
of a female child is higher)
In a business concern, there are four functions, namely, Production (PR),
Finance (FN), Personnel (PS) and Marketing (MK), The Customer (C) may also
play a role in the prosperity of the concern. Match List I with List II and select
the correct answer using the codes given below the lists :
List I (Pie Diagram)
List II (Statement)
A.
1. Customer is the controlling factor
B.
2. Marketing is an integrating factor
while
customer is the controlling factor
C.
3. Marketing is the controlling factor
D.
4. Marketing is more important than
others
5. Marketing is as equal a function
A B C D
A B C D
(a) 4 3 1 2
(b) 5 4 1 2
(c) 4 3 2 5
(d) 5 4 2 1
Ans. (b)
2000
Which one of the following statements is not true ?
(a) Ghaggar's water is utilised in the Indira Gandhi Canal
(b) Narmada rises from Amarkantak region
(c) Nizam Sagar is situated on the Manjira river
(d) Penganga is a tributary of the Godavari
Ans. (a)
Explanation:
Indira Gandhi Canal starts from the confluence of Sutlej and Beas river in
Punjab, and flows through Rajasthan. It receives water from Sutlej and Beas
rivers.
Consider the following statements about the megacities of India :
1. Population of each megacity is more than 5 million.
2. All the megacities are importance sea ports.
3. Megacities are either national or state capitals. Which of these statements
are correct?
(a) 1, 2 and 3
(b) 1 and 2
(c) 2 and 3
(d) 1 and 3
Ans. (d)
Explanation:
Statements 1 and 3 are correct.
Urban agglomerations, with over five million population are called Mega cities.
These are six - Greater Mumbai, Kolkata, Delhi, Chennai, Bangalore and
Hyderabad. Delhi is N.C.T. and remaining cities are state capitals.
The correct sequence in decreasing order of the four sugarcane producing
States in India is :
(a) Maharashtra, U.P., Tamil Nadu, Andhra Pradesh
(b) U.P., Maharashtra, Tamil Nadu, Andhra Pradesh
(c) Maharashtra, U.P., Andhra Pradesh, Tamil Nadu

96

(d) U.R, Maharashtra, Andhra Pradesh, Tamil Nadu


Ans. (b)
Explanation:
The correct sequence in decreasing order of the Sugarcane producing states is
U.P. > Maharashtra > Tamil Nadu > Andhra Pradesh. U.P, is the largest
producing state of Sugarcane in India.
96. Match the cities labelled as 1, 2, 3, 4 and 5 in the given map with the names of
the institutes located in these cities and select the correct answer using the
codes given below the names of the Institutes :
Name of institutes
A. Central Marine Fisheries Research Institute
B. Central Sheep Breeding Farm
C. National Dairy Research Institute
D. National Institute of Agricultural Extension Management
A B C D
A B C D
(a) 5 1 3 2
(b) 5 2 1 4
(c) 4 2 1 3
(d) 1 2 3 4
Ans. (b)
Explanation:
A - Cochin (5)
B - Hissar (2)
C - Karnal (1)
D - Hyderabad
(4)
97. Consider the following statements :
1. Maharashtra has the highest acreage under jawar in India.
2. Gujarat is the largest producer of groundnut in India.
3. Rajasthan has the largest area of cultivable wasteland in India.
4. Andhra Pradesh has the highest per hectare yield of maize in India.
Which of these statements are correct ?
(a) 1 and 4
(b) 2 and 3
(c) 1 and 3
(d) 2 and 4
Ans. (c)
Explanation :
1, 3 statements are correct. Because Andhra Pradesh is the largest producer of
Groundnut in India. Karnataka has the highest productivity (per hectare yield) of
maize in India.
98. Match the locations of ports labelled as A, B, C and D in the given map with the
names of those ports and select the correct answer using the codes given
below the names of the ports:
Names of Ports
1. Kakinada
2. Karwar
5. Veraval
A B C D
(a) 4 2 3 5
(c) 1 3 4 2
Ans. (b)
Explanation :
'A' is Veraval port of Gujarat.
'C is Tuticorin port of Tamil Nadu.
Pradesh.

3. Mangalore

4. Tuticorin

A B C D
(b) 5 2 4 1
(d) 5 3 2 1
B is Karwar port of Karnataka.
'D' is Kakinada port of Andhra

97

99. Which one of the following ports of India handles the highest tonnage of import
cargo ?
(a) Calcutta
(b) Kandla
(c) Mumbai
(d)
Visakhapatnam
Ans. (b)
Explanation:
Kandla handles the highest tonnage of import cargo.
100. Match the different ports of Union Territory of Pondicherry labelled as A, B, C
and D in the given map with their respeeitve names and select the correct
answer using the codes given below the lisi of ports:
List (Ports of Pondicherry)
1. Karaikal
2. Mahe
3. Pondicherry
4. Yanam
A B C D
A B C D
(a) 2 1 3 4
(b) 1 2 3 4
(c) 2 1 4 3
(d) 1 2 4 3
Ans. (a)
Explanation:
'A' is Mahe.
'B' is Karaikal.
'C is Pondicherry.
'D' is Yanam.
101. Which one of the following pairs of primitive tribes and places of their
inhabitation is NOT correctly matched ?
(a) Buksa
: Pauri-Garhwal
(b) Kol
: Jabalpur
(c) Munda
: Chhotanagpur
(d) Korba
: Kodagu
Ans. (a)
Explanation:
Primitive Tribal Groups (PTGs) are Scheduled Tribes known for their declining
or stagnant population, low levels of literacy, pre-agricultural technology,
primarily belonging to the hunting and gathering stage, and extreme
backwardness, mere are 7b Pibs spread over 15 states and UT's,
102. Which one of the following statements is true according to 1991 census data ?
(a) U.P. has the highest density of population in India
(b) Himachal Pradesh has the highest female to male sex ratio in India
(c) West Bengal has the highest growth rate of population in India
(d) Bihar has the lowest literacy rate in India
Ans. (d)
Explanation:
According to 2Q01 Census, Highest population density is in West Bengal (904).
Sex ratio is highe in Kerala (1058). Highest growth rate of population is in
Nagaland (64-41%). Bihar has the lowest literacy rate (47-43%).
Hence, 'd' is correct.
Directions: The following items consist of two statements, one labelled as
'Assertion A and the other labelled as 'Reason R'. You are to examine these
two statements carefully and decide if the Assertion A and the Reason R are
individually true and if so, whether the Reason is a correct explanation of the
Assertion. Select your answers to these items using the codes given below and
mark your answer sheet accordingly.
(a) Both A and R are true and R is the correct explanation of A
(b) Both A and R are true but R is MOT the correct explanation of A

98

103.

104.

105.

106.

107.

108.

(c) A is true but R is false


(d) A is false but R is true
Assertion (A) : The frequency of floods in North Indian plains has increased
during the last couple of decades.
Reason (R) : There has been reduction in the depth of river valleys due to
deposition of silt.
Ans. (a)
Explanation:
Frequency of floods in North Indian plains increased due to reduction in depth
of valleys by silt deposition.
Assertion (A) : Ganga Plain is the most densely populated part of India.
Reason (R) : Ganga is the most harnessed river of India.
Ans. (c)
Explanation:
Cauvery is the most harnessed river of India.
Consider the following statements :
1. Tides are of great heip in navigation and fishing.
2. High tide enables big ships to enter or leave the harbour safely.
3. Tide prevents siltation in the harbours.
4. Kandla & Diamond Harbour are tidal ports. Which of these statements are
correct ?
(a) 1 and 4
(b) 2, 3 and 4
(c) 1, 2 and 3
(d) 1, 2, 3 and 4
Ans. (d)
Explanation:
Ail the four statements are correct.
Kandla in Gujarat, Diamond Harbour in Calcutta are tidal ports.
Along which one of the following meridians did India experience the first light of
the sunrise of the new millennium?
(a) 2 30' W
(b) 82 30' E
(c) 92 30' W
(d) 92 30' E
Ans. (d)
Explanation:
In the given optionals 9230' E is the easternmost point in India. So it
experienced the first light of the sunrise of the new millenium.
Consider the following statements regarding environmental issues of India :
1. Gulf of Mannar is one of the biosphere reserves.
2. The Ganga Action Plan, phase II has been merged with the National River
Conservation Plan.
3. The National Museum of Natural History at New Delhi imparts non-formal
education in envionment and conservation.
4. Environmental Information System (ENVIS) acts as a decentralised
information network for environmental information.
Which of these statements are correct ?
(a) 1, 2 and 4
(b) 1, 2, 3 and 4
(c) 2 and 3
(d) 1, 3 and 4
Ans. (b)
Explanation:
All the statements are correct. 'Gulf of Mannar' is one of the 14 biosphere
reserves of India. Along with Nilgiri and .Sunderbans, it was recognised on
world Network of Biosphere Reserve by UNESCO.
The approximate age of the Aravaliis range is
(a) 370 million years (b) 470 million years (c) 570 million years (d) 670 million
years

99

Ans. (d)
Explanation:
The Aravaili system is said to date back to 3,500 million years. Some later
sediments seem to have been incorporated around 1,000 million years ago. So,
in the given answers, maximum age is 670 million years.
109. In the above map, the black maps show the distribution of :
(a) Asbestos
(b) Gypsum
(c) Limestone
(d) Mica
Ans. (c)
Explanation:
The given map shows the distribution of limestone. Limestone's areal
distribution is Madhya Pradesh, Tamil Nadu, .Orissa, Andhra Pradesh,
Karnataka, Assam, Bihar, Rajasthan, and Gujarat.
110. In the shaded area of the above map, the mean temperature for the month of
July varies between
(a) 22.5 C - 25.0 C (b) 25.0 C - 27.5 C (c) 27.5 C - 30.0 C (d) 30.0 C 32.5 C
Ans. (b)
Explanation:
During the month of July,
the temperature continues to be at
25-27C in Central and South-Western parts of India, whereas in South-Eastern
and North-Western parts, it is 30C. The July temperature in North-Eastern
parts also is at 25C.
111. Consider the decada! Census data given below: Decadal population (in
millions)
Year
Population
1961
10.7
1971
14.3
1981
16.2
1991
18.9
The above data refer to which one of the 'Population by Religion Groups ?
(a) Sikhs
(b) Jains
(c) Christians
(d) Buddhists
Ans. (c)
Explanation:
The data mentioned here refers to the Christian population. The population of
Christians was 10-7 million in 1961 (2-4%), 19-6 million in 1991 (2-34%).
112. The above map is the Union Territory of
(a) Chandigarh
(b) Daman and Diu (c) Dadra and Nagar Haveli (d)
Pondicherry
Ans. (c)
The given map is the Union ierritory of Dadra and Nagar Haveli.
113. Consider the following statements about the minorities in India :
1. The Government of India has notified five communities, namely, Muslims,
Sikhs, Christians, Buddhists and Zoroastrians as Minorities.
2. The National Commission for Minorities was given statutory status in 1993.
3. The smallest religious minority in India are the Zoroastrians.
4. The Constitution of India recognizes and protects religious and linguistic
minorities.

100

114.

115.

116.

117.

Which of these statements are correct ?


(a) 2 and 3
(b) 1 and 4
(c) 2, 3 and 4
(d) 1, 2 and 4
Ans. .(*)
Explanation:
All the statements are correct. Five religious communities1, i.e., Muslims,
Christians, Sikhs, Buddhists and Zoroastrians have been notified as minorities
as per the provisions under the "National Commission for Minorities", which
was set up as a Statutory body in 1993. The smallest religious minority in India
are the Zoroastrians concentrated in Maharashtra.
The largest number of Buddhists is found in
(a) Bihar
(b) Karnataka
(c) Maharashtra
(d) Uttar
Pradesh
Ans. (c)
Explanation:
The largest number of Buddhists is found in Maharashtra. These are NeoBuddhists who embraced this religion after large scale conversion under the
influence of the movement launched by Baba Saheb Ambedkar.
Consider the following statements regarding power sector in India :
1. The installed capacity of power generation is around 95000 MW.
2. Nuclear plants contribute nearly 15% of total power generation.
3. Hydroeiectricity plants contribute nearly 40% of total power generation.
4. Thermal plants at present account for nearly 80% of total power generation.
Which of these statements is/are correct ?
(a) 1 only
(b) 2 and 3
(c) 3 and 4
(d) 1 and 4
Ans. (a)
Explanation:
Power Generation in India :
1. Thermal
- 80,902-45 MW
2. Hydro
- 30,935-63 MW
3. Wind
- 3,811-01 MW
4. Nuclear
- 2,770-00 MW
Total
- 1,18,419-09 MW
2002
The sea coast of which one of the following states has become famous as a
nesting place for the giant Olive Ridley turtles from South America ?
(a) Goa
(b) Gujarat
(c) Orissa
(d) Tamil Nadu
Ans. (c)
Explanation:
Orissa coast is famous as a nesting place for the giant olive ridley turtles from
South America.
With reference to India, which one of the following statements is not correct ?
(a) About one-third of the area of the country records more than 750 millimetres
of annual rainfall
(b) The dominant source of irrigation in the country is wells
(c) Alluvial soil is the predominant type of soil in the northern plains of the
country
(d) The mountain areas account for about thirty percent of the surface area of
the country
Ans. (a)
Explanation:

101

118.

119.

120.

121.

122.

The average rainfall in India is about 125 cm. So more than one-third of the
area of the country records more than 750 millimetres of annual rainfall.
Remaining three statements are correct.
Open stunted forests with bushes and small trees having long roots and sharp
thorns or spines are commonly found in
(a) Eastern Orissa
(b) North-Eastern Tamil Nadu
(c) Siwaliks and Terai region
(d) Western Andhra Pradesh
Ans. (d)
Explanation:
The tropical semi-desert type merges into the tropical thorn forests when
natural vegetation consists of open stunted forests with widely scattered trees
and bushes and having long roots and sharp thorns. Eastern Rajasthan,
Southwest Punjab and Haryana, Bundelkhand region of Madhya Pradesh,
Southern Karnataka and adjoining Andhra Pradesh have this type of
vegetation.
Which one of the following is not a lagoon ?
(a) Ashtamudi lake (b) Chilka lake
(c) Periyar lake
(d) Pulicat lake
Ans. (c)
Explanation:
Lagoon means a part of sea partially cut off from it by deposits of sand or coral
reefs. In the given optionals, Ashtamudi, Chilka and Pulicat are lagoons
whereas Periyar lake is not a lagoon.
With reference to Indian agriculture, which one of the following statements Is
correct ?
(a) About 90 percent of the area under pulses in India is rainfed
(b) The share of pulses in the gross cropped area at the national level has
doubled in the last two decades
(c) India accounts for about 15 percent of the total area under rice in the world
(d) Rice occupies about 34 percent of the gross cropped area of India
Ans. (a)
Explanation :
About 23% of gross cropped area of India is under Rice Cultivation. Gross
cropped area of pulses has increased from 19-09 million ha. in 1950-51 to 20-0
million ha. in 2000-01.
HINDALCO, an aluminium factory located at Renukut owes its site basically to
(a) proximity of raw materials
(b) abudant supply of power
(c) efficient transport network
(d) proximity to the market
Ans. (b)
Explanation:
HINDALCO owed its site basically to abundant supply of power. It obtains
power from Rihand power house, which is nearer to this aluminium factory. Its
bauxite minfes are situated at Lohardaga in Bihar, and Amarkantak region of
Madhya Pradesh. HINDALCO was set up in 1958 at Renukut (about 160 km
from Mirzapur) in Uttar Pradesh.
The correct sequence of the eastward flowing rivers of the peninsular India
from north to south is
(a) Subarnarekha, Mahanadi, Godavari, Krishna, Pennar, Cauvery and Vaigai
(b) Subarnarekha, Mahanadi, Krishna, Godavari, Cauvery, Vaigai and Pennar
(c) Mahanadi, Subarnarekha, Godavari, Krishna, Cauvery, Pennar and Vaigai
(d) Mahanadi, Subarnarekha, Krishna, Godavari, Cauveri, Vaigai and Pennar

102

123.

124.

125.

126.

Ans. (a)
Explanation:
Subarnarekha in Jharkhand and West Bengal, Mahanadi in Chattisgarh and
Orissa, Godavari in Maharashtra and Andhra Pradesh, Krishna and Pennar in
Karnataka and Andhra Pradesh, Kaveri and Vaigai in Tamil Nadu are the east
flowing rivers.
With reference to Indian transport systems, consider the following statements:
1. Indian railway system is the largest in the world.
2. National Highways cater to 45 percent of the total road transport demand.
3. Among the states, Kerala has the highest density of surface road.
4. National Highway No. 7 is the longest in the country.
Which of these statements are correct ?
(a) 1 and 2
(b) 1 and 3
(c) 2 and 3
(d) 2 and 4
Ans. (d)
Explanation:
National Highway No. 7 from Varanasi to Kanyakumari is the longest.
Among the states Goa has the high density (153.8 km) of surfaced roads.
National Highways are only 1.94% of the total length of roads, carries 40-45%
of the total
traffic across the length and breadth of the country.
Indian Railway network is the largest in Asia and fourth largest in the world.
The average annual temperature of a meteorological station is 26C, its
average annual rainfall is 63 cm and the annual range of temperature is 9 C.
The station in question is
(a) Allahabad
(b) Chennai
(c) Cherrapunji
(d) Kolkata
Ans, (a)
Explanation:
Allahabad Station has average annual temperature of 26C, average annual
rainfall of 63 cm and the annual range of. temperature of 9C. Remaining three
stations are located in high rainfall regions of more than 200 cm.
Indias population growth during the 20th century can be classified into four
distinct phases. Match List I with List II and select the correct answer using the
codes given below the lists:
List I (Period)
List II (Phase)
A. 1901-1921
1. Steady growth
B. 1921-1951
2. Rapid high growth
C. 1951-1981
3. Stagnant growth
D. 1981-2001
4.High growth with definite signs of
slowdown
A B C D
A B C D
(a) 3 1 4 2
(b) 1 3 2 4
(c) 3 1 2 4
(d) 1 3 4 2
Ans. (c)
Explanation:
T'ne Indian demographic history can be divided into the following four phases :
1. Period of stagnant growth rate - Before 1921.
2. Period of steady growth rate - 1921-1951.
3. Period of rapid growth rate - 1951-1981.
4. Period of declining growth rate - After 1981.
Match List I with List II and select the correct answer using the codes given
below the lists:

103

List I (Mangrove)
A. Achra Ratnagiri
B. Coondapur
C. Pichavaram
D. Vernbanad

List II (State)
1. Karnataka
2. Kerala
3. Andhra Pradesh
4. Maharashtra
5. Tamil Nadu
A B C D
(b) 4 5 3 2
(d) 4 1 5 2

A B C D
(a) 2 1 5 4
(c) 2 5 3 4
Ans. (d)
Explanation:
A. Achra Ratnagiri is located in Maharashtra.
B. Coondapur is located in Karnataka.
C. Pichavaram is located in Tamil Nadu.
D. Vembanad is located in Kerala.
2003
127. The thermal power plant of Bokaro-is located in
(a) Bihar
(b) Chattisgarh
(c) Jharkhand
(d) Orissa
Ans. (c)
Explanation:
Bokaro is situated at a point between Jamshedpur and Jharia coal fields and
near to Raniganj Coal fields in Jharkhand. Its establishment integrates the
entire steel-coal belt into one closely Knit industrial region.
128. Which amongst the following States has the highest population density as per
Census-2001 ?
(a) Kerala
(b) Madhya Pradesh (c) Uttar Pradesh
(d) West
Bengal
Ans. (d)
Explanation:
Density of population is expressed as man-land ratio, i.e., number of persons
per unit of geographical area. The state wise population density varies from 13
persons in Arunachal Pradesh to 904 persons in West Bengal.
Directions : The following items consist of two statements, one labelled as
'Assertion A' and the other labelled as 'Reason R'. You are to examine these
two statements carefully and decide if the Assertion A and the Reason R are
individually true and if so, whether the Reason is a correct explanation of the
Assertion. Select your answers to these items using the codes given below and
mark your answer sheet accordingly.
(a) Both A and R are true and R is the correct explanation of A
(b) Both A and R are true but R is NOT the correct explanation of A
(c) A is true but R is false
(d) A is false but R is true
129. Assertion (A) : The eastern coast of India produces more rice than the western
coast.
Reason (R) : The eastern coast receives more rainfall than the western coast.
Ans. (c)
Explanation:
Statement A is correct. West Bengal, Orisssa, Andhra Pradesh, Tamil Nadu,
Assam states are producing more than
75% of the Rice in the river delta
regions.

104

130.

131.

132.

133.

134.

Statement R is wrong. Because the 1000 mm isohyet runs southwards from the
Gujarat coast, roughly parallel to the crest of Western Ghats upto Kanyakurnsri.
The rainfall over the peninsular region lying to the east of this line drops
abruptly to below 600 mm, as it lies in the rain-shadow region. So, west coast
receives more rainfall than east coast.
Assertion (A) : Coal-based thermal power stations contribute to acid-rain.
Reason (R) : Oxides of carbon are emitted when coal burns.
Ans. (a)
Explanation:
Coal based thermal power stations emit the oxides of Carbon (like CO, C02
etc.) sulphur (S02), Nitrogen (N02, N20) etc with the combustion of coal. When
coal burns. After reacting with die atmospheric moisture these oxides produce
acids and contribute to acid rains. So, both statements are correct and R is the
correct explanation of A.
What is the correct sequence of the rivers-Godavari, Mahanadi, Narmada and
Tapi in the descending order of their lengths ?
(a) Godavari - Mahanadi - Narmada - Tapi (b) Godavari - Narmada - Mahanadi Tapi
(c) Narmada - Godavari - Tapi - Mahanadi (d) Narmada - Tapi - Godavari Mahanadi
Ans. (b)
Explanation:
Godavari with 1465 km length, Mahanadi with 828 km length, are flowing from
west to east. Narmada with 1312 km length, Tapi with 724 km length are
flowing from east to west. So, the descending order of their lengths is Godavari
> Narmada > Mahanadi > Tapi.
Which one of the following statements is NOT correct?
(a)India is the second largest producer of nitrogenous fertilizers in the world
(b) India is the ninth largest steel producing country in the world
(c) India is the second largest producer of silk in the world
(d) India ranks third in the world in coal production
Ans. (*)
Explanation:
India is the Eighth largest steel producing country in the world. India is the third
largest producer of nitrogenous fertilizers in the world after U.S.A. and China.
(Remaining two statements are correct).
Which one among the following has the largest shipyard in India ?
(a) Kolkata
(b) Kochi
(c) Mumbai
(d)
Visakhapatnam
Arts. (b)
Explanation:
Kochi (Cochin) Shipyard Limited is designed to construct ships of size upto
1,10,000 DWT and repair ships up to 1,25,000 DWT. It is the only shipyard with
ISO-9001 accreditation for ship building, ship-repair and marine engineering
training in India.
Match List I with List II and select the correct answer using the codes given
below the lists:
List I (Organisation)
List II (Location)
A. National Sugar Institute
1. Dehradun
B. Mishra Dhatu Nigam Ltd.
2. Kamptee

105

C. Institute of Military Law


D. Institute of National Integration

135.

136.

137.

138.

3. Pune
4. Hyderabad
5. Kanpur
A B C D
(b) 5 1 4 3
(d) 5 4 2 3

A B C D
(a) 3 1 4 2
(c) 3 4 5 2
Ans. (d)
Explanation:
A. National Sugar Institute - Kanpur
B. Mishra Dhatu Nigam Ltd. - Hyderabad
C. Institute of Military Law - Kamptee
D. Institute of National Integration - Pune
Nanda Devi peak forms a part of
(a) Assam Himalayas
(b) Kumaon Himalayas
(c) Nepal Himalayas
(d) Punjab Himalayas
Ans. (b)
Explanation:
Nanda devi peak (7817 mts) forms part of Kumaon Himalayas which extends
from the river SutSej to the Kali to a distance of 320 km.
Consider the following statements :
1. India is the original home of the cotton plant.
2. India is the first country in the world to develop hybrid cotton variety leading
to increased production.
Which of these statements is/are correct ?
(a) Only 1
(b) Only 2
(c) Both 1 and 2
(d) Neither
1
nor 2
Ans. (c)
Explanation:
Both statements are correct.
Among the four cultivated cotton species, Gossypium arboreum is supposed to
be native of India. India is the first country in the world to develop hybrid cotton
named H-4 in Gujarat Agricultural University.
Among the following cities, which one is at the highest altitude above mean sea
level ?
(a) Bangalore
(b) Delhi
(c) Jodhpur
(d) Nagpur
Ans. (a)
Explanation:
Bangalore is located on the Bangalore plateau, whereas remaining cities
(Jodhpur, Nagpur, Delhi) are located on the plains. So, Bangalore is at the
highest altitude above mean sea level.
Consider the following statements :
1. Molasses is a by-product of sugar production process.
2. Bagasse obtained in the sugar mills is used as a fuel in the boilers
to
generate steam for the sugar factories.
3. Sugar can only be produced from sugarcane as the raw material.
Which of these statements are correct ?
(a) 1 and 2
(b) 2 and 3
(c) 1 and 3
(d) 1, 2 and 3
Ans. (a)
Explanation:

106

139.

140.

141.

142.

143.

Statements 1 and 2 are correct. But Third statement is wrong. Because Sugar
can be produced from Sugarbeet (Beta vulgaris) also. Sugarbeet is the major
contributor for world's sugar production than Sugarcane.
Consider the following statements :
1. Longitude of Jabalpurs location is between those of indore and Bhopal.
2. Latitude of Aurangabads location is between those of Vadodara and Pune.
3. Bangalore is situated more southward than Chennai.
Which of these statements is/are correct ?
(a) 1 and 3
(b) Only 2
(c) 2 and 3
(d) 1, 2 and 3
Ans. (c)
Explanation:
Statements 2, 3 are correct.
Longitude of Jabalpur's location is eastwards of those of Indore and Bhopal; but
not in between them.
Which one among the following States is smallest in area ?
(a) Andhra Pradesh (b) Gujarat
(c) Karnataka
(d) Tamil Nadu
Ans. (d)
Explanation:
Tamil Nadu is the smallest in area, in the given optionals. The decreasing order
of the area is Andhra Pradesh > Gujarat > Karnataka > Tamil Nadu.
Among the following cities, which one is nearest to the Tropic of Cancer ?
(a) Delhi
(b) Kolkata
(c) Jodhpur
(d) Nagpur
Ans. (b)
Explanation:
Kolkata is the nearest city to the Tropic of Cancer (231/2 North latitude).
Kolkata is located just below the Tropic of Cancer.
Consider the following statements :
1. India ranks first in the world in fruit production.
2. India ranks second in the world in the export of tobacco.
Which of these statements is/are correct ?
(a) Only 1
(b) Only 2
(c) Both 1 and 2
(d) Neither
1
nor 2
Ans. (b)
Explanation:
India ranks second in the world in fruit production after China.
Which one of the following statements is correct ?
(a) Alliance Air is a wholly-owned subsidiary of Indian Airlines
(b) The Airports Authoritty of India manages seven of the countrys international
airports
(c) The Airports Authority of India is the regulatory organisation for enforcing
civil air regulations in India
(d) It is the function of Directorate General of Civil Aviation to plan and construct
runways and terminal buildings and to provide air safety services
Ans. (a)
Explanation:
b, c, d statements are wrong.
The Airports Authority of India (AAI) manages 126 air ports, including 11
international, 86 domestic airports and 29 civil enclaves at defence air fields.
AAI plans, develops constructs and maintains runways, taxiways, apron,
terminal building etc. (it is not the function of DG of Civil Aviation) Director

107

144.

145.

146.

147.

148.

General of Civil Aviation is the regulatory body for enforcing Civil air regulations
in India.
Consider the following animals of India :
1. Crocodile
2. Elephant
Which of these is/are endangered species ?
(a) Only 1
(b) Only 2
(c) Both 1 and 2
(d) Neither
1
nor 2
Ans. (c)
Explanation:
Both Crocodile and Elephant are endangered species. In order to protect these
species, crocodile breeding project and project elephant were launched in 1974
and 1992 respectively.
Which one among the following States has the highest female literacy rate as
per the Census 2001 ?
(a) Chattisgarh
(b) Madhya Pradesh (c) Orissa
(d) Rajasthan
Ans. (a)
Explanation:
The female literacy rates of Chattisgarh, Madhya Pradesh, Orissa and
Rajasthan are 51-85%, 50-29%, 50-51% and 43-85% respectively, according to
2001 Census.
Which one of the following is produced during the formation of photochemical
smog ?
(a) Hydrocarbons
(b) Nitrogen Oxides (c) Ozone
(d) Methane
Ans. (b)
Explanation:
The deep reddish brown NOa (Nitrogen oxide) gas, which is the only widely
prevalent coloured pollutant gas is the chief constituent of Photochemical
Smog, in metropolitan areas.
2004
In which one of the following Union Territories, do the people of the Onge tribe
live ?
(a) Andaman and Nicobar islands
(b) Dadra and Nagar Haveli
(c) Daman and Diu
(d) Lakshadweep
Ans. (a)
Explanation:
There are 5 primitive tribal groups in Andaman and Nicobar Islands.
Tribes
Race
Islands
Onges
Negroid
Little Nicobar
Sentinelese
Negroid
Sentinel Island
Jarawa
Negroid
Middle and South Andaman
Andamanese
Negroid
Strait Island
Shompen
Mongoloid
Great Nicobar
Nicobarese
Mongoloid
Great Nicobar
Consider the following crops :
1. Cotton
2. Groundnut
3. Maize
4. Mustard
Which of the above are Kharif crops ?
(a) 1 and 2
(b) 1, 2 and 3
(c) 3 and 4
(d) 1, 2, 3 and 4
Ans. (b)
Explanation :

108

149.

150.

151.

152.

Cotton. Groundnut Maize are Kharif crops. Whereas Mustard Bengsigram,


wheat are Rabi crops.
Which one of the following statements is not correct ?
(a) The largest Buddhist monastery in India is in Assam
(b) The language Konyak is spoken in Nagaland
(c) The largest river island in the world is in Assam
(d) Sikkim is the least - populated State of the Indian Union
Ans. (a)
Explanation:
The largest Buddhist monastery in India is in Arunachal Pradesh.
Consider the following statements:
1. Damodar Valley Corporation is the first multipurpose river valley project of
independent India.
2. Damodar Valley Corporation includes thermal and gas power stations.
Which of the statements given above is/are correct?
(a) 1 only
(b) 2 only
(c) Both 1 and 2
(d) Neither
1
nor 2
Ans. (c)
Explanation:
Damodar Valley Corporation (DVC) is the first multi-purpose river valley project
constituted in independent India, in 1948, on the river of Sorrow viz., Damodar.
The DVC's main projects include four dams at Maithon, Panchet and Konar,
with connected hydro-electric power stations (except at Konar), Thermal Power
Station at Bokaro
'A', Bokaro 'B!, Chandrapura, Durgapura. Mejia and also one gas turbine
station at Maithon.
Consider the following statements :
Among the Indian States
1. Andhra Pradesh has the longest coastline
2. Gujarat has the highest number of airports Which of the statements given
above is/are correct ?
(a) 1 only
(b) 2 only
(c) Both 1 and 2
(d) Neither 1 or
2
Ans. (d)
Explanation:
Along with 13 Major ports, 185 minor ports are there in India, in which
Maharashtra has 53, Gujarat has 40 airports. Gujarat has the longest coast line
(not Andhra Pradesh).
Match List 1 (Minerals) with List II (Location) and select the correct answer
using the codes given below the lists :
List I
List II
A. Coal
1. Giridih
B. Copper
2. Jayamkondam
C. Manganese
3. Alwar
D. Lignite
4. Dharwar
A B C D
A B C D
(a) 1 4 3 2
(b) 2 3 4 1
(c) 1 3 4 2
(d) 2 4 3 1
Ans. (c)
Explanation :

109

153.

154.

155.

156.

Coal - Jharia (largest), Bokaro, Giridih of Jharkhand. Copper - Jhunjhunu and


Alwar districts of Rajasthan. Manganese - is associated with Dharwar rocks in
Karnataka Lignite - Jayamkondam, Neyveli (Tamil Nadu).
Which among the following National Highway routes is the longest ?
(a) Agra - Mumbai
(b) Chennai - Thane
(c) Kolkata - Hajira
(d) Pune - Machilipatnam
Ans. (c)
Explanation:
Starting
Terminal
N.H. No.
Length
Station
Station
in km.
Kolkata
Hajira
6
1949
Chennai
Thane
4
1235
Agra
Mumbai
3
1161
Pune
Machilipatnam
9
841
Consider the following :
1. Mahadeo Hills
2. Sahyadri Parvat 3. Satpura Range
What is the correct sequence of the above from the north to the south ?
(a) 1, 2, 3
(b) 2, 1, 3
(c) 1, 3, 2
(d) 2, 3, 1
Ans. (c)
Explanation:
Satpura is a series of Seven mountains commensing from Rajpipla hills in west,
through Mahadeo hills, it extends Maikaia range in the east. Sahyadris are
Western Ghats running from Tapti valley to north of Kanyakumari in NorthSouth direction.
Lake Sambhar is nearest to which one of the following cities of Rajasthan ?
(a) Bharatpur
(b) Jaipur
(c) Jodhpur
(d) Udaipur
Ans. (b)
Explanation:
Jaipur is the nearest city to the Lake Sakbhar in Rajasthan.
Match List I (National Park/Sanctuary) with List II (State) and select the correct
answer using the codes .given below the lists :
List I
List II
A. Kanger Ghati National Park
1. Chhattisgarh
B. Nagerhole National Park
2. Haryana
C. Kugti Wildlife Sanctuary
3. Himachal Pradesh
D. Sultanpur Bird Sanctuary
4. Kamataka
A B C D
A B C D
(a) 3 2 1 4
(b) 1 4 3 2
(c) 3 4 1 2
(d) 1 2 3 4
Ans. (b)
Explanation:
A. Kanger Ghati National Park - Chhattisgarh
B. Nagerhole National Park - Karnataka
C. Kugti Wildlife Sanctuary - Himacha! Pradesh
D. Sultanpur Bird Sanctuary - Haryana
Directions : The following items consist of two statements, one labelled as the
Assertion (A) and the other as Reason (R). You are to examine these two
statements carefully and select the answers to these items using the code
given below :
(a) Both A and R are true and K is the correct explanation of A

110

157.

158.

159.

160.

161.

(b) Both A and R are true but R is NOT the correct explanation of A
(c) A is true but R is false
(d)
A is false but R
is true
Assertion (A) : Bangalore receives much higher average annual rainfall than
that of Mangalore.
Reason (R) : Banglore has the benefit of receiving rainfall both from south west and north - east monsoons.
Ans. (d)
Explanation:
Bangalore receives less rainfall (312 cm) than Mangalore (946 cm) because
Mangalore is located west side of the western ghats. This rainfall receives very
high rainfall than the regions located eastern side of the western ghats.
Assertion (A) : West-flowing rivers of Peninsular India have no deltas.
Reason (R) : These rivers do not cany any alluvial sediments.
Ans. (c)
Explanation:
West flowing rivers carry alluvial sediments but are not deposited at the mouth
of the river to form deltas because of the swiftness of flow.
Match List I (Biosphere Reserve) with List II (States) and select the correct
answer using the codes given below the lists :
List I
List II
A. Similipal
1. Sikkim
B. Dehong Deband
2. Uttaranchal
C. Nokrek
3. Arunachal Pradesh
D. Kanchenjunga
4. Orissa
5. Meghalaya
A B C D
A B C D
(a) 1 3 5 4
(b) 4 5 2 1
(c) 1 5 2 4
(d) 4 3 5 1
Ans. (d)
Explanation:
A. Similipal - Orissa
B. Dehong Deband - Arunachal
Pradesh
C. Nokrek - Meghalaya
D. Kanchenjunga - Sikkim
Consider the following statements:
As per 2001 Census
1. the two States with the lowest sex ratio are Haryana and Punjab
2. the two States with the lowest population per sq km of area are Meghalaya
and Mizoram
3. Kerala has both the highest literacy rate and sex ratio
Which of the statements given above is/are correct?
(a) 3 only
(b) 2 and 3
(c) 1 and 2
(d) 1 and 3
Ans. (d)
Explanation :
The States with lowest population per sq. km. of area (density) are Arunachal
Pradesh (13), Mizoram (42), Sikkim (76) and Jammu & Kashmir (99).
Amongst the following Indian States which one has the minimum total forest
cover ?
(a) Sikkim
(b) Goa
(c) Haryana
(d) Kerala
Ans. (c)

111

162.

163.

164.

165.

Explanation:
Haryana has the least total forest cover among the above given states. On the
basis of percentage of the actual forest cover, the above states are in the order
of : Sikkim (40.80%) > Goa (33.79%) > Kerala > Haryana.
What is the correct sequence of the rivers-Godavari, Mahanadi, Narmada and
Tapi in the descending order of their lengths ?
(a) Godavari - Mahanadi - Narmada - Tapi (b) Godavari - Narmada - Mahanadi Tapi
(c) Narmada - Godavari - Tapi - Mahanadi (d) Narmada - Tapi - Godavari Mahanadi
Ans. (b)
Explanation:
River
Length
(km)
River
Length
(km)
Godavari
2465
Narmada
1312
Mahanadi
828
Tapi
724
According to Census 2001, which one of the following Indian States has the
maximum population in India after Uttar Pradesh ?
(a) West Bengal
(b) Maharashtra
(c) Bihar
(d) Tamil Nadu
Ans. (b)
Explanaiion:
State
Population (000)
India
- 1,028,610
Uttar Pradesh
- 1,66,198
Maharashtra
- 96,879
Bihar
- 82,999
West Bengal
- 80,176
Andhra Pradesh
- 76,210
Tamil Nadu
- 62,406
Match List-I with List-II and select the correct answer using the codes given
below the lists:
List-I (National Park/Wild-life Sanctuary
List-II (State)
A. Bondla Wildlife Sanctuary
1. Orissa
B. Kangerghat National Park
2. Assam
C. Orang Sanctuary
3. Chhattisgarh
D. Ushakothi Wildlife Sanctuary
4. Goa
5. Tripura
A B C D
A B C D
(a) 2 1 5 3
(b) 4 3 2 1
(c) 2 3 5 1
(d) 4 1 2 3
Ans. (b)
Explanation:
A. Bondla Wildlife Sanctuary - Goa
B. Kangerghat National Park - Chhattisgarh
C. Orang Sanctuary - Assam
D. Ushakothi Wildlife Sanctuary - Orissa
Where do the Sahariya tribals, who were recently in the news, live ?
(a) Andhra Pradesh (b) Assam
(c) Rajasthan
(d) Orissa
Ans. (c)
Explanation:

112

166.

167.

168.

169.

170.

Rajasthan
Consider the following statements :
1. India is the only country in the world producing all the five known commercial
varieties of silk.
2. India is the largest producer of sugar in the world.
Which of the statements given above is/are correct?
(a) 1 only
(b) 2 only
(c) Both 1 and 2
(d) Neither
1
nor 2
Ans. (a)
Explanation:
India is second largest producer of Sugar after Brazil in the world. Maharashtra
contributes about 1/3rd of country's sugar production followed by Uttar
Pradesh.
2005
Match items in the List-I with List-II and select the correct answer using the
codes given below the lists;
List-I (Power Station)
List-II (State)
A. Kothagudem
1. A.ndhra Pradesh
B. Raichur
2. Gujarat
C. Mettur
3. Karnataka
D. Wanakbori
4. Tamil Nadu
A B C D
A B C D
(a) 4 2 1 3
(b) 1 3 4 2
(c) 4 3 1 2
(d) 1 2 4 3
Ans. (b)
Explanation:
A. Kothagudem - Andhra Pradesh
B. Raichur - Karnataka
C. Mettur - Tamil Nadu
D. Wanakbori - Gujarat
For which one of the following, is Satara well-known?
(a) Thermal power plant
(b) Wind energy plant
(c) Hydro-electric plant
(d) Nuclear power plant
Ans. (b)
Explanation:
Wind Energy Plant.
Which one of the following statements is not correct ?
(a) Rourkela Steel Plant, the first integrated steel plant in the Public Sector of
India was set up with the Soviet Union collaboration
(b) Salem Steel Plant is a premier producer of stainless steel in India
(c) Maharashtra Elektrosmeit Ltd. is a subsidiary of the Steel Authority of India
Ltd.
(d) Visakhapatnam Steel Plant is a unit of the Rashtriya Ispat Nigam Ltd.
Ans. (a)
Explanation:
Rourkela Steel Plant, located in Sundargarh district was set up in collaboration
with the German Firm Krupps and Demag and was commissioned In 1959.
Which one of the following statements is not correct?
(a) The Western Ghats are relatively higher in their northern region
(b) The Anai Mudi is the highest peak in the Western Ghats
(e) Tapi river lies to the south of Satpura
(d) The Narmada and the Tapi river valleys are said to be old rift valleys

113

171.

172.

173.

174.

Ans. (a)
Explanation:
The Western Ghats are relatively higher in their Southern region. The highest
peak (2,696 m) of Anaimudi is the Central point from where three ranges
radiate in three directions - the Cardamom hills to the South, the Anamalai to
the North and the Paini to the North-east. Tapti river lies to the south of Satpura
and North of Ajanta ranges. The Narmada and Tapi are two old rift valleys
formed due to faulting of the land and drained to the west to join in Arabian
Sea.
Gandhi Sagar Dam is a part of which one of the following ?
(a) Chambal Project
(b) Kosi Project
(c) Damodar Valley Project
(d) Bhakra Nangal Project
Ans. (a) __________________________________________________
Explanation:
C'nambal project is a joint venture of Rajasthan and Madhya Pradesh started in
1954 on Chambal river.
Rana Pratap Sagar and Jawahar Sagar in Rajasthan and Gandhi Sagar Dam in
Chautasigarh near Bhanpura (Madhya Pradesh) are part of Chambal project.
Which one of the following is the correct sequence of the given hills starting
from the north and going towards the south ?
(a) Nallamalai Hills - Nilgiri Hills - Javadi Hills -Anaimalai Hills
(b) Anaimalai Hills - Javadi Hills - Nilgiri Hills -Nallamalai Hills
(c) Nallamalai Hills - Javadi Hills -- Nilgiri Hills -Anaimalai Hills
(d) Anaimalai Hills - Nilgiri Hills - Javadi Hills -Nallamalai Hills
Ans. (c)
Explanation:
Nallamalai Hills - Javadi Hills - Nilgiri Hills - Anaimalai Hills.
Which one of the following is not a Biosphere Reserve?
(a) Agasthyamalai (b) Nallamalai
(c) Nilgiri
(d) Panchmarhi
Ans. (b)
Explanation:
Nallamalai. The Biosphere Reserves of India (14) are Nilgiri, Nanda Devi,
Nokrek, Manas, Sunderbans, Gulf of Mannar,
Great Nicobar, Simlipal, Dibru-Daikhowa, Dihang-Dibang, Panchmarhi,
Kanchenjunga, Achanakmar Amarkantak and Agasthyamalai. Out of these
Nilgiri, Sunderbans and Gulf of Mannar have been recognised on World
Network of Biosphere Reserves by UNESCO.
Consider the following statements :
1. Liquefied Natural Gas (LNG) is liquefied under extremely cold temperatures
and high pressure to facilitate storage or transportation in specially designed
vessels.
2. First LNG terminal in India was built in Hassan.
3. Natural Gas Liquids (NGL) are separated from LPG and these include
ethane, propane, butane and natural gasoline.
Which of the statements given above is/are correct?
(a) 1 only
(b) 1 and 3
(c) 2 and 3
(d) 1, 2 and 3
Ans. (b)
Explanation:
GAIL is implementing a 1246 km long LNG pipeline project from
Kandla/Jamnagar in Gujarat to Luni in Uttar Pradesh via Delhi.

114

175. Match List-I (Atomic Power Plants/Heavy Water Plants) with List-II .(State) and
select the correct answer using the code given below the lists :
List - I
List - II
A. Thai
1. Andhra Pradesh
B. Manuguru
2. Gujarat
C. Kakrapar
3. Maharashtra
D. Kaiga
4. Rajasthan
5. Karnataka
A B C D
A B C D
(a) 2 1 4 5
(b) 3 5 2 1
(c) 2 5 4 1
(d) 3 1 2 5
Ans. (d)
Explanation:
A. Thai - Maharashtra
B.
Manuguru
Andhra Pradesn
C. Kakrapar - Gujarat
D.
Kaiga
Karnataka
176. For which one of the following items, is Tirupur well-known as a huge exporter
to many parts of the world?
(a) Gems & Jewellery
(b) Leather goods
(c) Knitted garments
(d) Handicrafts
Ans. (c)
Explanation
Knitted garments.
177. Consider the following statements concerning the Indian Railways :
1. The Head Quarters of the North Western Railway are located at Jodhpur.
2. Mndrail pass - a travel-as-you-please ticket has been created especially for
freedom fighters and sportspersons who have represented India in any
game/sport.
3. Fairy Queen is a train using the worlds oldest working engine and the Indian
Railways conduct a journey of wildlife and heritage sites on it.
Which of the statements given above is/are correct?
(a) 2 only
(b) 3 only
(c) 1 and 2
(d) None
Ans. (b)
Explanation:
The Headquarters of the North Western Railways is located at Jaipur. 'Indrail
pass' - a travel as-you-please ticket has been issued and sold only to foreign
nationals and Indians living abroad holding valid passport.
178. Which of the following States border Uttar Pradesh ?
1. Punjab
2. Rajasthan
3. Chhattisgarh
4. Jharkhand
Select the correct answer using the codes given below :
(a) 1, 2, 3 and 4
(b) 2, 3 and 4
(c) 1 and 4
(d) 1 and 3
Ans. (b) ____________________________________________________
Explanation :
States bordering Uttar Pradesh are : Rajasthan, Chattisgarh, Jharkhand,
Haryana, Himachal Pradesh, Uttaranchal, Bihar, Madhya Pradesh and National
Capital Territory (NCT) of Delhi.
179. Which one of the following is the correct sequence of the given Indian cities in
the decreasing order of their normal annual rainfall?
(a) Kochi - Kolkata - Delhi - Patna
(b) Koikata - Kochi - Patna - Delhi

115

180.

181.

182.

183.

(c) Kochi - Kolkata - Patna - Delhi


(d) Kolkata -- Kochi - Delhi - Patna
Ans. (c)
Explanation:
Kochi - Kolkata - Patna - Delhi.
Consider the following statements :
1. Silent Valley National Park is in the Nallamalai range.
2. Pathrakkadavu Hydroelectric Project is proposed to be built near the Silent
Valley National Park.
3. The Kunthi river originates in Silent Valleys rainforests.
Which of the statements given above is/are correct?
(a) 1 and 3
(b) 2 only
(c) 2 and 3
(d) 1, 2 and 3
Ans. (c)
Explanation:
Silent Valley National Park is present in the Kudali Hills of the Western Ghats at
the south western corner of the Nilgiri Hills in Paighat district in Kerala.
Nallamalai range is present in Andhra Pradesh.
Consider the following statements :
1. Areawise, Chattisgarh is larger than West Bengal.
2. According to the Population 2001 Census, population of 'West Bengal is
larger than that of Chhattisgarh.
Which of the statements given above is/are correct?
(a) 1 only
(b) 2 only
(c) Both 1 and 2
(d) Neither I
nor 2
Ans. (c)
Explanation:
Population
Stale
Area (sq. km.)
Population (2001 Census)
Chattisgarh
1,36,034
2,07,95,956
West Bengal
88,752
8,02,21,171
Which one of the following is the correct statement on the basis of Census2001 ?
(a) Bihar has the highest percentage of the Scheduled Castes of its population
(b) The decadal growth of population of India (1991-2001) Tias been below 20%
(c) Mizoram is the Indian State with the least population
(d) Pondicherry has the highest sex ratio among the Union Territories
Ans. (d)
Explanation:
Punjab (28-85) has the highest percentage of the Scheduled Castes of its
population followed by Himacha Pradesh (24-72) and West Bengal (23-02).
Decadal Growth rate (%)
Census Period
21-54
1991-2001
23-87
1981-1991
24-66
1971-1981
Sikkim (540,851) has the least population followed by Mizoram (888,573)
and Arunachal Pradesh (10,97,968)
The Sex ratio is highest in Pondicherry (> 1,000) and lowest in Daman and
Diu (709) among Union Territories.
Consider the following statements :
1. India is the second country in the world to adopt a National Family Planning
Programme.

116

2. The National Population Policy of India 2000 seeks to achieve replacement


level of fertility by 2010 with a population of 111 crores.
3. Kerala is the first State in India to achieve replacement level of fertility.
Which of the statements given above is/are correct?
(a) I only
(b) 1 and 2
(c) 2 and 3
(d) 1, 2 and 3
Ans. (c)
Explanation:
India is the first country in the world to adopt a National Family Planning
Programme in the year 1952.
184. Which one of the following statements is not correct ?
(a) There is no definition of the Scheduled Tribe in the Constitution of India
(b) North-East India accounts for a little over half of the countrys tribal
population
(c) The people known as Todas live in the Nilgiri area
(d) Lotha is a language spoken in Nagaland
Ans. (b)
Explanation:
Of the total Scheduled Tribe Population (8-43 crores), more than 50% of them
are distributed in Middle and East Indian States viz., Madhya Pradesh (1.2
crore), Jharkhand (0.7 crore), Chattisgarh (0.66 crore), Orisssa (0.8 crore).
West Bengal (0.44 crore), Maharashtra (0.85 crore). The total S.T. population
of NE India (7 states) is 1.163 crores only.
185. Consider the following statements :
1. The forest cover in India constitutes around 20% of its geographical area.
Out of the total forest cover, dense forest constitutes around 40%.
2. The National Forestry Action Programme aims at bringing one-third of the
area of India under tree/forest cover.
Which of the statements given above is/are correct?
(a) 1 only
(b) 2 only
(c) Both 1 and 2
(d) Neither 1
nor 2
Ans. (b)
Explanation:
Type
Area (000 sq. km)
% of the
Geographical Area
in 2003
Total Forest Cover
678.33
20.64
Dense forests
51.29
1.58
Moderately dense forests
339.28
10.32
Open forests
287.67
8.76
Total Tree cover
81.47
2.48
Forest and Tree cover
757.01
23.03
186. Match List-I with List - II and select the correct answer using the code given
below the lists:
List-I (National Park/ Wildlife Sanctuary)
List-II (Nearby
Town)
A. Chandra Prabha
1. Jaipur
B. Karera
2. Jhansi
C. Jaisamand
3. Agra
D. Nahargarh
4. Varanasi
5. Udaipur

117

A B C D
A B C D
(a) 4 1 5 2
(b) 5 2 3 1
(c) 4 2 5 1
(d) 5 1 3 2
Ans. (c)
Explanation :
A. Chandra Prabha - Varanasi
B. Karera - Jhansi
C. Jaisamand - Udaipur
D. Nahargarh - Jaipur
187. Consider the following statements :
1. Sikkim has the minimum area among the 28 Indian States (Delhi and
Pondicherry not included).
2. Chandigarh has the highest literacy rate among Pondicherry, NCT of Delhi
and other Union Territories.
3. Maharashtra has the highest population after Uttar Pradesh among the 28
Indian States (Delhi and Pondicherry not included).
Which of the statements given above is/are correct?
(a) 1 and 2
(b) 2 and 3
(c) 1 only
(d) 3 only
Ans. (d)
Explanation:
Goa is the smallest state (3,702 sq. km.) followed by Sikkim (7,096 sq. km.)
and Tripura (10,492 sq.km.).
Lakshadweep (86.7%) has the highest literacy rate among the Union Territories
followed by Chandigarh (81.9%), Delhi (81.7%), Andaman and Nicobar Islands
(81.3%), Pondicherry (81:2%), Daman & Diu (78.2%) and Dadra & Nagar
Haveli (57.6%). Maharashtra has the highest population after Uttar Pradesh
followed by Bihar, West Bengal and Andhra Pradesh.
188. Match List-I with List-11 and select the correct answer using the code given
below the lists:
List-1 (Centre of Handicrafts)
List-II (State)
A. Mon
1. Arunachal Pradesh
B. Nalbari
2. Assam
C. Pasighat
3. Meghalaya
D. Tura
4. Nagaland
A B C D
A B C D
(a) 4 2 1 3
(b) 1 3 4 2
(c) 4 3 1 2
(d) 1 2 4 3
Ans. (a)
Explanation :
A. Mon - Nagaland
B. Malbari - Assam
C. Pasighat - Arunachal Pradesh
D. Tura - Meghalaya
189. Consider the following statements :
Assertion (A) : The percentage of net sown area in the total area of Andhra
Pradesh is less as compared to that of West Bengal.
Reason (R) : The soil of most of the Andhra Pradesh is laterite.
In the context of above two statements, which one of the following is correct ?
(a) Both A and R are true and R is the correct explanation of A
(b) Both A and R are true but R is NOT the correct explanation of A
(c) A is true but R is false
(d) A is false but R is true
Ans. (c)
Explanation:

118

Red Soils occupy over 66% of the cultivated area in Andhra Pradesh followed
by Black Soils (25%), alluvial loamy soils (5%), coastal sands (3%) and laterite
soils (1%). The Percentage of Net sown area in the total area of Andhra
Pradesh (38.94%) is less than West Bengal (62.2%).
190. Match List-1 with List-II and select the correct answer using the code given
below the lists:
List-I (Valley)
List-II (State)
A. Markha Valley
1. Sikkim
B. Dzukou Valley
2. Himachal Pradesh
C. Sangla Valley
3. Jammu and Kashmir
D. Yumthang Valley
4. Nagaland
A B C D
A B C D
(a) 2 4 3 1
(b) 3 1 2 4
(c) 2 1 3 4
(d) 3 4 2 1
Ans. (d)
Explanation:
A. Markha Valley - J & K
B.
Dzukou Valley
- Nagaland
C. Sangla Valley - Himachal Pradesh
D. Yumthang Valley - Sikkim
191. Which one of the following statements is not correct ?
(a) Mahanadi River rises in Chattisgarh
(b) Godavari
River
rises
in
Maharashtra
(c) Cauvery River rises in Andhra Pradesh (d) Tapti River rises in Madhya
Pradesh
Ans. (c)
Explanation:
Cauvery River originates from Brahmagiri of Western Plateau (Karnataka) while
Mahanadi originated from Dandakaranya (Chattisgarh), Godavari from
Triyambakeshwar (Nasik) and Tapti river from Muttai's Baitul district (Madhya
Pradesh).
192. Consider the following statements :
1. According to the Census 2001, Kerala has ihe smallest gap in male and
female literacy rates among the 28 states of India (Delhi and Pondicherry not
included).
2. According to the Census 2001, Rajasthan has literacy rate above the
national average literacy rate.
Which of the statements given above is/are correct?
(a) I only
(b) 2 only
(c) Both 1 and 2
(d) Neither 1
nor 2
Ans. (d)
Explanation:
According to the 2001 Census, Rajasthan has literacy rate (60.41%) below the
national average literacy rate (64.84%).
State
Literacy rate (%)
Total
Male
Female
Gap
Kerala
90.9
94.2
87.7
6.5
Mizoram
88.8
90.7
86.7
4.0
(smallest gap)

119

193.

194.

195.

196.

Meghalaya
62.6
65.4
59.6
5.8
Rajasthan
60.4
75.7
43.9
31.8
Consider the following statements :
1. Assam shares a border with Bhutan and Bangladesh.
2. West Bengal shares a border with Bhutan and Nepal.
3. Mizoram shares a border with Bangladesh and Myanmar.
Which of the statements given above are correct ?
(a) 1, 2 and 3
(b) 1 and 2, only
(c) 2 and 3, only
(d) 1 and 3,
only
Ans. (a)
Explanation:
Five states bordering Nepal : Uttaranchal, Uttar Pradesh, Bihar, West Bengal
and Sikkim.
Five states bordering Bangladesh : West Bengal, Assam, Meghalaya, Tripura
and Mizoram.
Five states bordering China : Jammu and Kashmir, Himachal Pradesh,
Uttaranchal, Sikkim and Arunachal Pradesh.
Four states bordering Myanmar : Arunachal Pradesh, Nagaland, Manipur, and
Mizoram.
Four states bordering Pakistan : Jammu and Kashmir, Punjab, Rajasthan and
Gujarat.
Which one of the following pairs is not correctly matched ?
Railway Zone
Headquarters
(a) North Eastern Railway
: Gorakhpur
(b) South Eastern Railway
: Bhubaneshwar
(c) Eastern Railway
: Kolkata
(d) South East Central Railway
: Bilaspur
Ans. (b)
Explanation:
Railway Zone
Headquarters
South Eastern Railway Kolkata
East coast Railway
Bhubaneswar
Where is the Central Water and Power Research Station located ?
(a) Khadakwasla
(b) Sileru
(c) Jamnagar
(d) Srisailam
Ans. (a)
Explanation:
Central Water and Power Research Station located near Khadakwasla Dam
(Pune). It was established in 1916 as a 'special Irrigation cell by the then
Bombay presidency but later in 1936, it was undertaken by Govememnt of
India,
Which of the following subjects are found in the beach sands of many parts cf
Kerala ?
1. llmenite
2. Zircon
3. Sillimanite
4. Tungsten
Select the correct answer using the code given below :
(a) 1, 2, 3 and 4
(b) 1, 2 and 3 only
(c) 3 and 4 only
(d) 1 and 2 only
Ans. (b)
Explanation:

120

197.

198.

199.

200.

201.

The beach sands in Koliam are rich in heavy minerals like Monozite, llmenite,
Rutile, Zircon and Silimanite. The Kerala Minerals and Metals Limited' is
engaged in mining of these minerals.
Between which of the following was the ancient town of Takshasila located ?
(a) Indus and Jhelum
(b) Jhelum and Chenab
(c) Chenab and Ravi
(d) Ravi and Beas
Ans. (a)
Explanation:
"Takshasila" was the capital of "Ambhi who was an ally of Alexander the
Great", is present between rivers Indus and Jhelum.
Which one of the following pairs is not correctly matched ?
Monastery
State
(a) Dhankar Monastery
: Himachal Pradesh
(b) Rumtek Monastery
:
Sikkim
(c) Tabo Monastery
: Himachal Pradesh
(d) Kye Monastery
: Arunachal Pradesh
Ans. (d)
Explanation:
Kye Monastery is situated in the Lahaul and Spiti district of Himachal Pradesh.
From north towards south, which one of the following is the correct sequence of
the given rivers in India ?
(a) Shyok - Spiti - Zaskar - Satluj
(b) Shyok - Zaskar - Spiti - Satluj
(c) Zaskar - Shyok - Satluj - Spki
(d) Zaskar - Satluj - Shyok - Spiti
Ans. (b)
Explanation:
The correct sequence of the rivers from North towards South : Shyok - Zaskar Spiti - Satluj.
2007
Match List-I with List-II and select the correct answer using the code given
below the lists :
List-I (Town)
List-II (River Nearer to it)
A. BetuL
1. Indravati
B. Jagdalpur
2. Narmada
C. Jabalpur
3. Shipra
D. Ujjain
4. Tapti
A B C D
A B C D
(a) 1 4 2 3
(b) 4 1 2 3
(c) 4 1 3 2
(d) 1 4 3 2
Ans. (b)
Explanation:
4, 1, 2, 3
Betui is the district of MP. The major rivers flowing in the district- are Ganjal
river that is tributary of Tapi river, Morand river and Tawa river.
Jagdalpur is a city in the Chattisgarh. The city is located on the south bank
of Indravati river.
Jabalpur is in MP. It has a beautiful collection of marble rocks surrounding the
holy Narmada river.
Ujjain is in Malwa region of MP, on the eastern bank of Shipra river.
Consider the following statements :
1. In India, Red Panda is naturally found in the Western Himalayas only.

121

202.

203.

204.

205.

2. In India, Slow Loris lives in the dense forests of the North East.
Which of the statements given above is/are correct ?
(a) 1 only
(b) 2 only
(c) Both 1 and 2
(d) Neither 1
nor 2
Ans. (b)
Explanation:
The Red Panda is found in mountaneous band from Nepal through NE India
and Bhutan and into China. Laos and northern Myanmar.
Match List-I with List-II and select the correct answer using the code given
below the lists :
List-I (Aluminium Company)
List-II (Location)
A. BALCO
1. Hirakud
B. HINDALCO
2. Korba
C. Indian Aluminium Company
3. Koraput
D. NALCO
4. Renukoot
A B C D
A B C D
(a) 3 1 4 2
(b) 2 4 1 3
(c) 3 4 1 2
(d) 2 1 4 3
Ans. (b)
Explanation:
BALCO is located at a distance of 10 km from the Korba city.
HINDALCO commissioned its aluminium factory at Renukoot in eastern UP in
1962.
NALCO was incorporated in 1981 in the Public sector, to exploit a part of large
deposits of bauxite discovered in East Coast.
Which one of the following is located in the Bastar region ?
(a) Bandhavgarh National Park
(b) Dundeli Sanctuary
(c) Rajaji National Park
(d) Indravati National Park
Ans. (d)
Explanation:
Bandhavgarh National Park - M.P.
Dandeli Sanctuary - Karnataka
Rajaji National Park - Uttarakhand
Indravati National Park - Bastar, Chattisgarh
Source : New Vishal's C.A. 2007 - Page 258, 259, 262
Which one of the following National Highways passes through Maharashtra,
Chhattisgarh and Orissa ?
(a) NH4
(b) NH5
(c) NH6
(d) NH7
Ans. (c)
Explanation :
NH4 - Thane - Chennai
NH5 - Jharpokharia - Chennai (Along east coast)
NH6 - Hajra, Kolkata via Nagpur, Sambalpur and Raipur
NH7 - Varanasi - Kanyakumari via Jabalpur, Nagpur, Hyderabad, Bangalore
and Madurai (longest in the country).
Consider the following statements :
1. Baiaghat is known for its diamond mines.
2. Majhgawan is known for its manganese deposits.
Which of the statements given above is/are correct ?

122

206.

207.

208.

209.

210.

(a) 1 only
(b) 2 only
(c) Both 1 and 2
(d) Neither 1
nor 2
Ans. (d)
Explanation:
Manganese reserves are found in Balaghat and Chindwara districts (MP).
Majhagawan area in Panna district is rich in Diamond reserves.
Which one among the following States of India has the lowest density of
population ?
(a) Himachal Pradesh
(b) Meghalaya
(c) Arunachal Pradesh
(d) Sikkim
Ans. (c)
Explanation:
State
Density of Population
Arunachal Pradesh 13
Sikkim
76
Himachai Pradesh 109
Meghalaya
103
Source : New Vishal's Current Affairs 2007 - 190
Which one of the following rivers originates at Amarkantak?
(a) Damodar
(b) Mahanadi
(c) Narmada
(d) Tapti
Ans. (c)
Explanation:
River
Place of Origin
Damodar
Chotanagpur plateau (Jharkhand)
Mahanadi
Dandakaranya (Chattisgarh)
Narmada
Amarkantak (Maikal Mountain)
Tapti
Muttai's Baitul dist (MP)
Which one among the following major Indian cities is most eastward located ?
(a) Hyderabad
(b) Bhopal
(c) Lucknow
(d) Benguiuru
(Bangalore)
Ans. (c)
Explanation:
Hyderabad is located almost in middle of India.
Bhopal is located in North eastern parts of MP in Central region of India.
Lucknow is. situated in the Upper Gangetic plains of India. Bangalore is
situated on the south-eastern corner of Karnataka.
Dalbergia species is associated with which one of the following ?
(a) Cashew nut
(b) Coffee
(c) Tea
(d) Rosewood
Ans. (d)
Explanation:
Palisander Rosewood - Dalbergia baroni
Siamese Rosewood - D, Cochinchinensis Indian Rosewood - D. latifblia etc.
Out of the four southern States Andhra Pradesh, Karnataka, Kerala and Tamil
Nadu, which shares boundaries with the maximum number of Indian States ?
(a) Andhra Pradesh only
(b) Karnataka only
(c) Each of Andhra Pradesh and Karnataka (d)
Each of Tamil
Nadu and Kerala
Ans. (c)
Explanation:
State
Boundary shared by

123

211.

212.

213.

214.

215.

Andhra Pradesh
Maharashtra, Karnataka,
Chattisgarh,
Orissa, Tamilnadu
Karnataka
Goa, Maharashtra, Andhra Pradesh, Tamilnadu, Kerala
Tamilnadu
Andhra Pradesh, Kerala,
Karnataka
Kerala
Karnataka, Tamilnadu
In which State is the Guru Shikhar Peak located ?
(a) Rajasthan
(b) Gujarat
(c) Madhya Pradesh (d)
Maharashtra
Ans. (a)
Explanation:
Guru Shikhar - On covering a distance of 3 miles Delwara Achagarh road, a
hilly coal-tarred road of nearly 7 kms, leads to highest peak of Mount-Abu popularly known as Guru Shikhar. It is the highest peak of Mt. Abu, at the
height of 5650 ft/1722 mts. On the peak is the temple of Guru Dattatreya, who
is supposed to be ab incarnation of God Brahma, Vishnu and Mshesh.
Where were Shevaroy Hills located ?
(a) Andhra Pradesh (b) Karnataka
(c) Kerala
(d) Tamil Nadu
Ans, (d)
Explanation:
Yercaud, popularly known as 'poor man's ooty is iocated in the Shevaroy Hills
of north central Tamilnadu about 4,500 ft. above sea level.
In which one of the following districts, have large reserves of diamond-bearing
kimberlite been discovered in the recent past ?
(a) Hoshangabad
(b) Raipur
(c) Sambalpur
(d) Warangal
Ans. (b)
Explanation:
Large reserves of diamond-bearing Kimberlite traits have been discovered in
Payalikhand and Behradin in Raipur district and Tokapal in Bastar.
Which one of the following is also known as Top Slip ?
(a) Simlipal National Park
(b) Periyar Wildlife Sanctuary
(c) Manjira Wildlife Sanctuary
(d) Indira Gandhi Wildlife Sanctuary and
National Park
Ans. (d)
Explanation:
Indira Gandhi Wildlife Sanctuary and National Park is also known as 'Top Slip'.
It lies in the Coimbatore District of Tamil Nadu at the southern part of the Nilgiri
Biosphere Reserve in the Anamalai Hills.
Assertion (A) : River Kalinadi is an east-flowing river in the southern part of
India.
Reason (E) : The Deccan Plateau is higher along its western edge and gently
slopes towards the Bay of Bengal in the east.
Ans. (d)
Explanation:
River Kalindi is a west flowing river.

6. WORLD GEOGRAPHY
1995
1.

A meteor is
(a) a rapidly moving star
124

2.

3.

4.

5.

(b) a piece of matter which has entered the earths atmosphere from outer
spalce
(c) part of a constellation
(d) a comet without tail
Ans. (b)
Explanation:
Meteors are the pieces of matter which have entered the earth's atmosphere
from outer space by the gravitational force of the earth, when they pass close to
the earth. Once these objects enter into the thick atmosphere of the earth, due
to the friction they burn before reaching the earth.
Given below is a map of some countries which were parts of the erstwhile
Soviet Union with water bodies shown by shaded areas :
The countries marked 1,2, 3, 4 and 5 are respectively
(a) Tajikistan, Turkmenia, Uzbekistan, Kirgizia, Kazakhstan
(b) Turkmenia, Kirgizia, Tajikistan, Uzbekistan, Kazakhstan
(c) Kazakhstan, Uzbekistan, Tajikistan, Kirgizia, Turkmenia
(d) Kazakhstan, Turkmenia, Uzbekistan, Kirgizia, Tajikstan
Ans. (d)
Explanation:
1. Kazakhstan, 2. Turkmenia (Turkmenistan), 3. Uzbekistan, 4. Kirgizia
(Kirgyzstan), 5. Tajikistan.
These were separated from U.S.S.R. in 1991.
The Standard time of the following countries is ahead or behind Greenwich
Mean Time depending on whether they are east or west of the longitude
passing through Greenwich.
1. Cube
2. Greece
3. Iraq
4. Costa Rica
5. Japan
Which one of the following sequential orders gives the correct arrangement of
the countries according to their standard time from ahead to behind GMT?
(a) 5, 3, 2, 1, 4
(b) 2, 4, 1, 3, 5
(c) 4, 1, 3, 2, 5
(d) 3, 5, 4, 1, 2
Ans. (a)
Explanation:
The Greenwich line which passes through London is called Prime Meridian. It is
0 longitude. The countries which lies east of the Prime Meridian has their
standard time ahead of the Greenwich Mean Time, and which lies west of the
Prime Meridian has their standard time behind the Greenwich Mean Time.
Accordingly from East to West, the order of the given countries is Japan, Iraq,
Greece, Cuba and Costa Rica.
The original home of the gypsies was
(a) Egypt
(b) Russia
(c) India
(d) Persia
Ans. (c)
Explanation:
Gypsies are the nomadic people, whose origin place is Noth-West India i.e.
Western Rajasthan and Punjab, and migrated and found mostly now in Central
Asia. 'Gypsy' is an Egyptian term.
'Which one of the following is the continent with the highest mean elevation in
the world?
(a) Antarctica
(b) North America
(c) Asia
(d)
South
America
Ans. (a)

125

6.

7.

8.

9.

Explanation:
Antarctica is having highest mean elevation. It is located in South pole region.
Its Eastern side is more elevated than the Western side.
The graph given below shows the mean average monthly temperatures (in C)
and mean monthly rainful (in cms) of a place :
The graph is indicative of which one of the climatic zones of the world
?
(a) Wet & dry tropical (b) Rainy tropical
(c) Semi-arid tropica! (d) Temperate
marine
Ans. (a)
Explanation:
As the graph indicates average temperature range 15-25C and average
rainfall range also 15 - 25 cm not exceeding 30 cm and heaviest rainfall in May,
June months, It is wet and dry tropical climate zone of the world.
Match List I with List II and select the correct answer using the codes given
below the lists:
List I
List II
A. European transcontinental railway
1. Paris to Istanbul
B. Trans-Andean railway
2.
Leningrad
to
Vladivostok
C. Trans-Siberian railway
3. Leningrad to Volgograd
D. Orient Express
4. Buenos Aires to Valparaiso
5. Paris to Warsaw
A B C D
A B C D
(a) 5 4 2 1
(b) 1 4 3 2
(c) 5 1 2 3
(d) 1 2 3 4
Ans. (a)
Explanation :
European transcontinental railway is from Paris to WarSaw Trans-Andean
railway is from Buenos Aires to Valparaiso Trans-Siberian railway is from
Leningrad to Vladivostok. Orient express is from Paris to Istanbul.
Willy willy is
(a) a type of tree grown in temperate regions
(b) a wind that blows in a desert
(c) a tropical cyclone of the north-west Australia
(d) a kind of common fish found near Lakshadweep islands
Ans. (c)
Explanation:
Willy-Willy is a tropical cyclone of the North-West Australia, occur during the
hottest period of the year and provide rainfall. It originates from Timor Sea and
hits the coast.
Consider the map given below indicating four places frequently figuring in the
news :
Which one of them is Chectienya ? Choose the correct answer.
(a) 1
(b) 2
(c)3
(d) 4
Ans. (c)
Explanation:
The place indicated is Chechenya. It is a part of the Russia, it is located in the
South-West region of that country. It was in news recently because the

126

separatist movement going on in there. Russia is trying to supress that


movement.
10. The clouds floods in the atmosphere because of their low
(a) temperature
(b) velocity
(c) pressure
(d) density
Ans. (d)
Explanation :
As the clouds are less denser than the below air, they can flood in the
atmosphere.
11. Comet shoemaker-Levy 9 hit the planet
(a) Piuto
(b) Mars
(c) Jupiter
(d) Saturn
Ans. (c)
Explanation:
Comet Shoemaker-Levy 9 hit the planet Jupiter. It was discovered by Keroshil
Shoemaker and David Levy on 24th March 1994.
12. Consider the map given below ;
The dotted (broken) line in the map is the :
(a) Durand line
(b) MacMahon line
(c) line of demarcation between India and Pakistan suggested by the Boundary
Commission (1947)
(d) route followed by the Youghhusband Expedition
Ans. (a)
Explanation:
It is the Durand Sine which is demarcated by a Commission j under Mr.
Durand, between India and Afghanistan.
13. Which one of the following layers of the atmosphere Is responsible for the
deflection of ! radio waves ?
(a) Troposphere
(b) Stratosphere
(c) Mesosphere
(d) Ionosphere
Ans. (d)
Explanation:
In the lower part of the Thermosphere region between 100 to 400 km, ionisation
of atmospheric gases takes place. This layer is called Ionosphere. There is a
peak concentration of ionized particles at 250 km, that is why this layer deflects
radio waves.
14. Consider the following statements :
Assertion (A) : Areas near the equator receive rainfall throughout the year.
Reason (R) : High temperatures and high humidity cause Convectional rain in
most afternoons near the equator.
In the contest of the above two statements, which one of the following is correct
?
(a) Both A and R are true and R is the correct explanation of A
(b) Both A and R are true but R is not a correct explanation of A
(c) A is true but R is false
(d)
A is false but R
is true
Ans. (a)
Explanation:
These areas near equator are called Tropica! Rain Forest regions. Here
generally the type of rainfall is convectional rainfall, because of constant high
temperature and high humidity produces rainfall almost daily in the afternoon
throughout the year.

127

15. Which one of the following sets of conditions is necessary for a good cultivation
of wheat ? (a) Moderate temperature and moderate rainfall
(b) High temperature and
heavy rainfall
(c) High temperature and
moderate rainfall
(d) Low temperature and low rainfall
Ans. (a)
Explanation:
Wheat is a temperate crop. So it requires moderate ranges of temperature
(between 15 to 20C) and rainfaii (between 25 - 75 cm). That Is why, if grown
in tropical region, it is grown as Rabi crop.
16. The rough outline map shows a portion of the Middle East. The countries
labelled A, B, C and D are respectively :
(a) Syria, Iraq, Jordan and Saudi Arabia
(b) Syria, Iraq, Saudi Arabia and
Jordan
(c) Iraq, Syria, Saudi Arabia and Jordan
(d) Iraq, Syria, Jordan and Saudi
Arabia
Ans. (c)
Explanation:
The regions touching Euphrates river are Iraq and Syria. Beside Syria is Jordan
and in Southwards Is Saudi Arabia.
17. Diamond Ring is a phenomenon observed
(a) at the start of a total solar eclipse
(b) at the end of a total solar eclipse
(c) only along the peripheral regions of the totality trail
(d) only in the central regions of the totality trail
Ans. (c)
Explanation:
Diamond ring is a phenomenon observed only along the peripheral regions of
the totality trail.
18. A rough outline map of Northern Sri Lanka is shown in the given figure. Jaffna
is located at the point marked :
(a) A
(b) B
(c)C
(d) D
Ans. (d)
Explanation
The Jaffna region of Srilanka is located in North-West region. It is a stronghold
of Tamil separatist group L.T.T.E.
1997
19. About 50% of the world population is concentrated between the latitudes of
(a) 5 N and 20 N (b) 20 N and 40 N (c) 40 N and 60 N (d) 20 S and
40 S
Alts. (b)
Explanation:
Between 20N and 40N latitudes, most of the population of the world is
situated. It is in this region most populous countries like China, India, U.S.A.
and North African countries and southern countries of the EUROPE sre
located.
20. The tail of a comet is directed away from the sun because
(a) as the comet rotates around the sun, the lighter mass of the comet is pushed
away due to the centrifugal force alone

128

(b) as the comet rotates, the lighter mass of the comet is attracted by some star
situated in the direction of its tail
(c) the radiation emitted by the sun exerts a radial pressure on the comet
throwing its tail away from the sun
(d) the tail of the comet always exists in the same orientation
Ans. (a)
Explanation :
As the comet rotates around the Sun, due to the centrifugal force, the lighter
mass of the comet is pushed away and appears as a tail. It is directed away
from the Sun.
21. Consider the map given below :
The river shown on the map is
(a) Irrawaddy
(b) Mekong
(c) Chao Phraya
(d) Salween
Ans. (d)
Explanation :
The river shown in the Map is Salween, it originates from plateau of Tibet and
flows through Shan plateau in Eastern region of Myanmar.
22. Which one of the following factors is responsible for the change in the regular
direction of the ocean currents in the Indian Ocean ?
(a) Indian Ocean is half an ocean
(b) Indian Ocean has Monsoon drift
(c) Indian Ocean is a land-locked Ocean
(d) Indian Ocean has greater variation in salinity
Ans. (b)
Explanation:
Indian Subcontinent (in north), Africa (in west) and Australia (in South East) are
the land regions that are located around the Indian ocean. Due to the monsoon
drift, ocean current in the Indian ocean becomes eastward and westward in
June and November respectively. It is affected by wind circulation.
23. The Prime Minister of which one of the following countries is chosen by the
ruling prince from a slate of three candidates put up to him by the President of
France ?
(a) San Marius
(b) Liechtenstein
(c) Malta
(d) Monaco
Ans. (d)
24. Match List I with List II and select the correct answer using the codes given
below the lists:
List I (Crops)
List II (Geographical conditions)
A. Barley
l. Hot and dry climate with poor soil
B. Rice
2. Cool climate with poorer soil
C. Millets
3. Warm and Moist climate with high altitude
D. Tea
4. Hot and Moist climate with rich soil
A B C D
A B C D
(a) 2
4 1
3 (b)
3 4 12
(c) 2
1 4
3 (d)
3 2 4
1
Ans. (a)
Explanation:
Barley is a temperate crop. It requires cooler climate.
Rice requires high temperature and high rainfall and nutrient rich soil.
Millets can be grown in less water and poor soil conditions. Tea can be grown
in high altitudes with warm and moist climate.

129

25. From Aceh in the far north west to Torres Strait in the east is 5000 miles,
almost as far as from London to Baghdad. The archipelago has 14,000 islands,
some mere equatorial rocks, others some of the largest in the world. This
description best fits
(a) West Indies
(b) Japan
(c) Philippines
(d) Indonesia
Ans. (d)
Explanation:
The description best fits to Indonesia as the country sprawled across, with
14,000 islands in the equatorial region and North-western tip of the country is
Banda Aceh.
26. The group of small pieces of rock revolving round the sun between the orbits of
Mars and Jupiter are called
(a) meteors
(b) comets
(c) meteorites
(d) asteroids
Ans. (d)
Explanation:
The group of small pieces of rocks revolving round the Sun, between the orbits
of Mars and Jupiter are called Asteroids. They may be about 1,00,000 in
numbers but their total mass is only a few hundredths the mass of Moon. They
include Ceres (the largest - 940 kms in diametre), Vesta (has a light coloured
surface and brightest as seen from the Earth), Eros and Icarus.
27. If the earths direction of rotation is reversed, what would be the 1ST when it is
noon at the International Date Line ?
(a) 06.30 hrs
(b) 05.30 hrs
(c) 18.30 hrs
(d) 17.30 hrs
Ans. (a)
Explanation:
Generally, 1ST is 5:30 hrs. ahead of the Greenwich mean time. So if its noon at
the International Date line (Greenwich line) then 1ST is 12 : 00 + 5 :30 = 17:30
hrs.
If the earths direction of rotation is reversed, the 1ST will be behind Greenwich
mean time. So if its noon at the International Date line then 1ST will be 12 : 005 :30= 6: 30 hrs.
28. Which one of the following stars is nearest to the Earth ?
(a) Polaris
(b) Alpha Centauri (c) Sun (d)
Sirius
Ans. (c)
Explanation:
Sun is the nearest star of earth, with its average distance being 149-6 million
km and proxima centauri is the second nearest star of the earth.
29. Which one of the following conditions is most relevant for the presence of life
on Mars ?
(a) Atmospheric composition
(b) Thermal conditions
(c) Occurrence of ice caps and frozen water (d)
Occurrence of
ozone
Ans. (c)
Explanation:
The most relevant condition for the presence of life on Mars is occurence of ice
caps and frozen water. Because the same situations, on the earth also, are
supposed to support the 'primitive forms of life.
30. Consider the geographical details given in the following figure :
The point marked by A in the above figure indicates a country in

130

(a) North America


(b) South America (c) Europe
(d) Asia
Ans. (c)
Explanation:
As the longitudes 5 EL, 6 EL are just beside the prime meridian, and in the
Northern Hemisphere, the point marked indicates the parts of Europe.
31. Consider the map given below :

32.

33.

34.

35.

Of the four shaded areas in the map, that which is characterised by hot dry
summers, mild and moist winters and seasonal reversal of winds is the area
labelled
(a) 1
(b) 2
(c) 3
(d) 4
Ans. (a)
Explanation:
The hot dry summers, mild and moist winters and seasonal reversal of winds is
the charateristic of Mediterranean type of climate, found between 30-45
latitude in both the hemispheres on the western side of each continent. So in
the given map 2, 3, 4 are in eastern side. Only 1 which is indicated for
California (U.S.A.) region is possessing that type of climate.
Daily weather map showing isobars is an example of
(a) Choropleth map
(b) Isopleth map
(c) Chorochromatic map
(d) Choroschematic map
Ans. (b)
Explanation:
Daily weather map showing isobars i.e. places having equal pressure is an
example of Isopleth map.
Which one of the following countries is the largest producer of fuel wood in the
world ?
(a) Indonesia
(b) Russia
(c) India
(d) China
Ans. (b)
Explanation:
Russia has the largest cover of forests.
One will NOT have to pass through the Suez Canal while going from Bombay
to
(a) Alexandria
(b) Suez
(c) Port Said
(d) Benghazi
Ans. (b)
Explanation:
Suez is at the eastern entry point of the Suez canal which is nearest to
Bombay, where as Port Said is in the North western point of Suez canal.
Alexandria (Egypt), Benghazi (Libya) are away from
Suez
canal,
and in order
to reach these ports, one has to travel through the Suez
canal.
During a flight from Delhi to Tokyo the following are the landing airports :
1. Hongkong
2. Hanoi
3. Taipei
4. Bangkok
The correct sequence of the landing at these airports during an onward journey
is
(a) 1, 2, 3, 4
(b) 4, 2, 1, 3
(c) 3, 4, 1, 2
(d) 4, 1, 2, 3
Ans. (b)
Explanation:
During the flight from Delhi to Tokyo, the Sequence of ariports is - Bangkok
(Thailand), Hanoi (Vietnam), Hongkong, Taipei (Taiwan).

131

36. Which one of the following is present in the largest amount in terms of per cent
by mass in the earths crust ?
(a) Silicon
(b) Oxygen
(c) Carbon
(d) Calcium
Ans. (b)
Explanation:
Oxygen, in the form of oxides, is present in the largest amounts, second
abundant is Silicon, third abundant is Aluminium, in the earth's crust.
1998
37. Consider the following statements regarding asteroids :
1. Asteroids are rocky debris of varying sizes orbiting the Sun.
2. Most of the asteroids are small but some have diameter as large as 1000
km.
3. The orbit of asteroids lies between orbits of Jupiter and Saturn.
Of these statements
(a) l, 2 and 3 are correct
(b) 2 and 3 are correct
(c) l and 2 are correct
(d) 1 and 3 are coiject
Ans. (c)
Explanation:
Asteroids are rocky debris of varying sizes from smaller to as large as 1000 km.
But the orbit of asteroids (tier orbit) lies between the orbits of mars and Jupiter,
but not between Jupiter and Saturn. So, 1 & 2 are correct but not 3.
38. The correct sequence of different layers of the atmosphere from the surface of
the Earth upwards is:
(a) Troposphere, Stratosphere, Ionosphere, Mesosphere
(b) Stratosphere, Troposphere, Ionosphere, Mesosphere
(c) Troposphere, Stratosphere, Mesosphere, Ionosphere
(d) Stratosphere, Troposphere, Mesosphere, Ionosphere
Ans. (c)
Explanation:
The correct sequence is Tropo - Strato - Meso - Iono spheres.
Troposphere extends upto 8-18 km, after that, Stratosphere upto 50 km, after
that Mesosphere extends upto 80 km and beyond that, upto 400 km Ionosphere
exists.
39. Match List I with List II and select the correct answer using the codes given
below the lists:
List-I
List-II
A. Ringgit
1. Indonesia
B. Baht
2. South Korea
C. Rupiah
3. Thailand
D. Won
4. Malaysia
A B C D
A B C D
(a) 1 3 4 2
(b) 4 3 1 2
(c) 1 2 4 3
(d) 4 2 1 3
Ans. (b)
Explanation:
Ringgit is the, National Currency of Malaysia.
Baht is the National Currency of Thailand.
Rupiah is the National Currency of Indonesia.
Won is the National Currency of S. Korea.

132

40. Match river labelled A, B, C and D on the given map with their names given in
the list and select the correct answer :
List I
1. St. Lawrence
2. Orinoco
3. Mackenzie
4. Amazon
5. Yukon
A B C D
A B C D
(a) 4 3 2 1
(b) 5 3 1 2
(c) 5 4 1 3
(d) 3 1 4 2
Ans. (b)
Explanation:
'a' is Yukon river in Alaska region.
'b1 is Mackenzie river in Northern Canada.
'c' is St. lawrence river in Eastern Canada.
'd is Orinoco river in Venizuela.
41. Estuaries possess distinct blooms of excessive growth of a pigmented
dianoflagellates. These blooms are called
(a) red tides
(b) sea tides
(c) black tides
(d) sea flowers
Ans. (a)
Explanation:
The growth of pigmented dianoflagellates causes red colour to water. So these
blooms are called Red tides.
42. Consider the following climatic conditions (northern hemisphere) :
Temperature (C)
Rainfall (Cm.)
J
3.9
4.7
F
4.4
5.7
M
8.3
8.2
A
14.4
9.2
M
20.0
9.2
J
23.3
17.7
J
27.8
14.5
A
27.8
14.0
S
22.3
12.7
O
18.3
7.0
N
12.2
5.0
D
6.7
3.5
These are most likely to be found in the natura regions of
(a) China type
(b) Equatorial type
(c) Hot desert type
(d) Monsoon type
Ans. (a)
Explanation:
This is most likely to be found in the natural region of China type; which is
typified by a warm moist summer and a cool, dry winter. Temperatures strongly
modified by maritime influence.
43. When there is noon at I.S.T. meridian people on another place of the Earth are
taking their at 6 Oclock morning tea. The longitude of the place is :
(a) 17 30' E
(b) 7 30' W
(c) 172 30' E
(d) 90 W

133

44.

A.
B.
C.

45.

46.

47.

Ans. (b)
Explanation:
Since the earth makes one complete rotation of 360 in one day or 24 hrs, it
passes through 15 in one hour, or, 1 in 4 minutes. The earth rotates from
west to east. In the given problem, it is noon at 1ST (Indian Standard Time).
GMT is 5:30 hrs westwards to 1ST. The longitude passes through that place is
"prime meridian". It is 0 longitude. Still we have to travel 30 minutes
westwards. If we convert this time into longitudes - 30/4 = 730' westwards. So
the answer is 'b'.
Match List-I with List-II and select the correct answer using the codes given
below the lists:
List I (Special characteristic)
List II (Name of planet)
Smallest planet of the solar system
1. Mecury
Largest planet of the solar system
2. Venus
Planet second from the
3. Jupiter
Sun in the solar system
D. Planet nearest to the Sun
4. Pluto
5. Saturn
A B C D
A B C D
(a) 2 3 5 1
(b) 3 5 1 2
(c) 4 1 2 3
(d) 4 3 2 1
Ans. (d)
Explanation:
The smallest planet of the Solar system is Pluto. Largest is Jupiter, planet
second from the Sun is Venus and planet nearest to the Sun is Mercury.
1999
If it is 10.00 a.m. I.S.T., then what would be the local time at Shillong on 92 E
longitude ?
(a) 9.38 a.m.
(b) 10.38 a.m.
(c) 10.22 a.m.
(d) 9.22 a.m.
Ans. (b)
Explanation:
1ST is 5:30 hrs ahead of GMT that is 8230'E longitude. If Shinong is on 92E
longitude, it is 930' ahead of 1ST. To travel 1, it takes 4 minutes for earth. So
for 9 30', it takes 38 minutes. If 1ST is 10:00 a.m./ the local time at Shillong is
10:00 + 38min. = 10:38 a.m.
Which one of the following scholars suggests the earth's origin from gases and
dust particles?
(a) James Jeans
(b) H. Alfven
(c) F. Hoyle
(d) O. Schmidt
Ans. (d)
Explanation:
The Scholar, suggested the earth's origin from gases and dust particles is O.
Schmidt. He propounded "Inter Stellar dust hypothesis".
James Jeans - Tidal hypothesis
H. Alfven - Intestellar cloud hypothesis F. Hoyle - Super nova hypothesis.
Match List with List II and select the correct answer using the codes given
below the lists :
List I (Volcanic Mountain)
List II (Country)
A. Mt. Rainier
1. Italy
B. Etna
2. Mexico
C. Paricutin
3. Philippines

134

D. Taal
4. U.S.A.
A B C D
A B C D
(a) 4 2 1 3
(b) 4 1 2 3
(c) 2 1 4 3
(d) 4 3 2 1
Ans. (b)
Explanation:
Volcanic mount Rainier is located in North-Western part of U.S.A. Mt. Etna in
Sicili region of Italy.
Mt. Paricutin in Mexico. Mt. Taal in Philippines.
48. In the given map, which one of the following map of ocean currents is shown ?
(a) Benguela & Falkland
(b) Canary & Humboldt
(c) Agulhas & Guinea
(d) Benguela & Guinea
Ans. (d)
Explanation :
In the given map, the Guinea and Benguela ocean currents are shown, which
flows in the central western and south western sides of Africa.
49. At which one of the following positions shown in the diagram will the height of
the ocean tide be maximum?
(a)M1
(b) M2
(c)M3
(d) M4
Ans. (d)
Explanation:
The tide will be maximum, when the position of Sun and Moon are in straight
line i.e. in opposition (here M2) and in Conjunction (here M4). But when we
compare these two, the position of Conjunction tend to have maximum
.gravitational pull. So answer is M4.
50. Which one of the areas marked as A, B, C and D in the given figure of the
cyclone, witnesses heavy torrential short-duration rainfall accompanied by
thunderstorms ?
(a) A
(b) B
(c) C
(d) D
Ans. (b)
Explanation:
The area marked by 'B' witnesses torrential downpours accompanied by
thunder and lightning because in this area, maximum upliftment of warm air
mass takes place.
51. Consider the following temperature and rainfall data:
Month
Temperature (C)
Rainfall (cm)
January
6.7
44:0
February
6.7
13.2
March
7.2
11.4
April
8.9
9.4
May
11.1
8.1
June
13.9
8.1
July
15.0
9.6
August
15.0
12.2
September
13.9
10.4
October
11.1
14.4
November
8.9
14.0

135

52.

53.

54.

55.

December
7.8
16.8
The climate to which this data pertains is
(a) St. Lawrence type
(b) China type
(c) West European type
(d) Mediterranean type
Ans. (d)
Explanation:
According to the given data - the, climate is Mediterranean type. In this type,
winter rainfall is a typical character. Maximum rainfall occurs from October to
February.
Match List I with List II and select the correct answer using the codes given
below the lists:
List I (Timber)
List II (Country)
A. Cedar
1. Myanmar
B. Douglas Fir
2. Canada
C. Mahogany
3. Mexico
D. Teak
4. Honduras
A B C D
A B C D
(a) 3 2 1 4
(b) 3 2 4 1
(c) 2 3 4 1
(d) 2 3 1 4
Ans. (b)
Explanation :
Cedar is found in Mexico.
Douglas fir is found in Canada.
Mahogany in Honduras and Teak in Myanmar.
Match List I with List II and select the correct answer using the codes given
below the lists:
List I
List II
A. Cotton
1. Rainfall 1000-1500mm; Temperature 40 - 60C
B. Flax
2. Rainfall 1500-2000 mm; Temperature 25 - 35C
C. Sugarbeet
3. Rainfall 600-800 mm; Temperature 5 - 18C
D. Jute
4. Rainfall 500-1000 mm; Temperature 18 - 22C
5. Rainfall 500 - 600 mm; Temperature 18 - 22C
A B C D
A B C D
(a) 1 3 4 2
(b) 2 3 5 4
(c) 4 5 2 1
(d) 4 3 5 2
Ans. (d)
Explanation:
Cotton requires 500-1000 mm rainfall Flax (Linseed) requires 600-800 mm
rainfall Sugarbeet requires 500-600 mm rainfall Jute requires 1500-2000 mm
rainfall
Which one of the following port cities in Venezuela has been developed as an
oil port ?
(a) Caracas
(b) Maracaibo
(c) Maracay
(d) Carupano
Ans. (b)
Explanation:
The port city Maracaibo in Venezuela has been developed as an oil port.
Match the cities labelled as A, B, C and D in the given map with the names of
cities and select the correct answer using the codes given below the names of
cities :

136

Names of Cities :
1. Darwin
2. Kuala Lumpur
3. Lagos
4. Nairobi
5. Singapore
A B C D
A B C D
(a) 1 2 4 3
(b) 2 1 4 3
(c) 1 4 5 2
(d) 4 3 5 2
Ans. (a)
Explanation
City marked 'A1 is Darwin in Australia City marked 'B' is Kuala Lumpur in
Malaysia City marked 'C1 is Nairobi in Kenya City marked 'D' is Lagos in
Nigeria
56. The physical regions marked as 1, 2, 3 and 4 on the given map are respectively
(a) Andes, Brazilian Shield, Guyana Highlands and Amazon Basin
(b) Andes, Guyana Highlands, Brazilian Shield and Amazon Basin
(c) Amazon Basin, Guyana Highlands, Brazilian Shield and Andes
(d) Guyana Highlands, Brazilian Shield, Andes and Amazon Basin
Ans. (a)
Explanation:
The physical region ranked '1' is Andes, '2' is Brazilian Shield '3' is Guyana
Highlands and '4' is Amazon Basin.
57. Which one of the following types of coal contains a higher percentage of carbon
than the rest ?
(a) Bituminous coal (b) Lignite
(c) Peat
(d) Anthracite
Ans. (d)
Explanation:
Anthracite contains 80-90% carbon. It is the type of coal which contains higher
percentage of carbon.
Bituminous coal - 70-90% carbon
Lignite/Brown coal - 45-75% carbon
58. Which one of the following statements is not correct ?
Dinar/New Dinar is the currency of
(a) Sudan
(b) Yugoslavia
(c) U.A.E.
(d) Tunisia
Ans. (a)
Explanation:
Sudan's currency is Sudanese pound, whereas remaining countries (i.e.
Yugoslavia, U.A.E., Tunisia) are associated with Dinar/New Dinar.
Directions : The following items consist of two statements, one labelled as the
Assertion (A) and the other as Reason (R). You are to examine these two
statements carefully and select theu answers to these items using the code
given below :
(a) Both A and R are true and R is the correct explanation of A
(b) Both A and R are true but R is NOT the correct explanation of A
(c) A is true but R is false
(d) A is false but R is true
59. Assertion (A) : Chile continues to be an important producer of copper in the
world.
Reason (R) : Chile is endowed with the world's largest deposit of porphyry
copper.
Ans. (a)
Explanation:

137

60.

61.

62.

63.

64.

Chile continues to be an important producer of copper in the world as it is


endowed with the world's largest deposit of porphyry copper in Chugni Camata
region.
A ship sailing from the eastern extremity of the Aleutian Islands to Dutch
crosses 180 meridian at 23.30 hrs on January 1, 1999. What time and date will
be recorded by the captain of the ship in his diary after one-hour journey from
the point of crossing of the meridian ?
(a) January 1, 0030 hrs
(b) January 2, 0030 hrs
(c) January 3, 0030 hrs
(d) January 4, 0030 hrs
Ans. (c)
A person of mixed European and Indian blood in Latin America is called a
(a) Mulatto
(b) Mestizo
(c) Meiji
(d) Mau Mau
Ans. (b)
2000
Match List I with List II and select the correct answer using the codes given
below the lists:
List I (Minerals)
List II (Major producer)
A. Mineral Oil
1. Zambia
B. Copper
2. Guyana
C. Manganese
3. Venezuela
D. Bauxite
4. Gabon
A B C D
A B C D
(a) 3 1 4 2
(b) 3 1 2 4
(c) 1 3 2 4
(d) 1 3 4 2
Ans. (a)
Explanation:
The major producer of Mineral oil is Venezuela.
The major producer of Copper is Zambia in Africa, (also Chile in South
America).
The major producer of Manganese is Gabon.
The major producer of Bauxite is Guyana.
Which one of the following is knows as the Coffee port of the world ?
(a) Sao Paulo
(b) Santos
(c) Rio de Janeiro
(d) Buenos
Aires
Ans. (b)
Explanation:
'Santos' is considered as the "Coffee port" of the world. It is in Brazil. It is also
an out port of Sao Paulo.
At which one of the cities labelled as A, B, C and D on the given map of Europe
was the historic treaty between NATO and Warsaw Pact countries signed in
1998 ?

(a) A
(b) B
(c) C
(d) D
Ans. (c)
Explanation:
In 1998, the historic treaty between NATO and Warsaw pact countries was
signed at Brussels, the headquarters of NATO. With this the NATO member
countries increased to 19. (At present total NATO member countries are 26).
So, 'C is Brussels in Belgium.
65. Consider the following provinces of former Yugoslavia:

138

66.

67.

68.

69.

1. Bosnia
2. Croatia
3. Slovenia
4. Yugoslavia
The correct sequence of these provinces from the east to the west is:
(a) 4, 1, 3, 2
(b) 4, 1, 2, 3
(c) 1, 4, 3,2
(d) 1, 4, 2, 3
Ans. (b)
Explanation :
The correct sequence from East to West is Yugoslavia, Bosnia Herzgovina,
Croatia, Slovenia. These were former Yugoslavia provinces and divided in
1999.
Which one of the following lakes forms an international boundary between
Tanzania and Uganda ?
(a) Chad
(b) Malawi
(c) Victoria
(d) Zambezi
Ans. (c)
Explanation :
Lake Victoria (Biggest lake in Africa) forms the international boundary between
Tanzania (South) and Uganda (North).
A 'black hole1 is a body in space which does not allow any radiation to come
out. This property is due to its
(a) very small size (b) very large size
(c) very high density (d) very
low
density
Ans. (c)
Explanation:
Because of its high density, a black hole has very high gravitational force. So
no object, whether it is particle or light, can escape from it.
Directions : The following items consist of two statements, one labelled as the
Assertion (A) and the other as Reason (R). You are to examine these two
statements carefully and select the answers to these items using the code
given below :
(a) Both A and R are true and R is the correct explanation of A
(b) Both A and R are true but R is NOT the correct explanation of A
(c) A is true but R is false
(d) A is false but R is true
Assertion (A) : In Australia, cattle rearing is done more for meat than for milk.
Reason (R) : Australians are traditionally non-vegetarians.
Ans. (b)
Explanation:
Both A and R are correct. But R is not the correct reason for A. Though they
are traditionally non-vegetarian, they rear cattle for meat, for exporting.
Match List 1 with l.ist II and select the correct answer using the codes given
below the lists:
List I (Oceanic Trench)
List II (Location)
A. Aleutian
1. Indian Ocean
B. Kermadec
2. North Pacific Ocean
C. Sunda
3. South Pacific Ocean
D. S. Sandwich
4. South Atlantic Ocean
A B C D
A B C D
(a) 2 4 1 3
(b) 2 3 1 4
(c) 1 3 2 4
(d) 1 4 2 3
Ans. (b)
Explanation:

139

A-Aleutian trench in North Pacific ocean near Bering Sea B-Kermadec trench in
South Pacific ocean near New Zealand C-Sunda trench in Indian ocean near
Sumatra Island D-South Sandwich trench in Atlantic ocean
70. Match the drainage basins labelled as A, B, C and D with the names listed
below and select the correct answer using the codes given below the drainage
basins
Names of drainage basins
1. Ganga-Brahmaputra 2. Indus
3. Parana
4. Zambeski
A B C D
A B C D
(a) 3 1 2 4
(b) 1 3 4 2
(c) 1 3 2 4
(d) 3 I 4 2
Ans. (d)
Explanation:
A - Parana (3)
B - Ganga-Brahmaputra (1)
C - Zambeski (4)
D - Indus (2)
71. Which one of the following statements is false ?
(a) All countries other than India have over 80% literacy
(b) Malaysia and Korea have life expectancy higher than all other countries
(c) Higher the adult literacy lower is the infant mortality
(d) The life expectancy at birth in India is almost the same as that of Indonesia
Ans. (c)
2001
72. Consider the following statements :
1. Most magmas are a combination of liquid, solid and gas.
2. Water vapour and carbon dioxide are the principal gases dissolved in a
magma.
3. Basaltic magma is hotter than the silicic magma.
4. The magma solidified between sedimentary rocks in a horizontal position is
known as
dike.
Which of these statements are correct ?
(a) 1, 2 and 3
(b) 2, 3 and 4
(c) 1 and 4
(d) 1, 2 and 4
Arts. (a)
Explanation:
1, 2, 3 statements are correct but not 4. Because, the magma solidified
between sedimentary rocks in a horizontal position is known
as "Sill", but
not Dyke (Dyke - solidification in vertical position).
73. Consider the following statements regarding the earthquakes :
1. The intensity of earthquake is measured on Mercalli Scale.
2. The magnitude of an earthquake is a measure of energy released.
3. Earthquake magnitudes are based on direct measurements of the amplitude
of seismic waves.
4. In the Richter Scale, each whole number demonstrates a hundredfold
increase in the amount of energy released.
Which of these statements are correct ?
(a) 1, 2 and 3
(b) 2, 3 and 4
(c) 1 and 4
(d) 1, and 3
Arts. (a)
Explanation:
1, 2, 3 statements are correct but not 4. Because in the Richter scale, each
whole number demonstrates a ten fold increase in the amount of energy.

140

74. Cloudy nights are warmer compared to clear cloudless nights, because clouds
(a) perevent cold waves from the sky from descending on earth
(b) reflect back the heat given off by earth
(c) produce heat and radiate it towards earth
(d) absorb heat from the atmosphere and send it towards earth
Ans. (b)
Explanation:
Cloudy nights are warmer compared to clear cloudless nights, because clouds
reflects back the heat given off by the earth.
75. Which one of the following weather conditions is indicated by a sudden fall in
barometer reading ?
(a) Stormy weather
(b) Calm weather
(c) Cold & dry weather
(d) Hot & sunny weather
Ans. (a)
Explanation:
Stormy weather condition is indicated by a sudden fall in barometer reading.
Because decrease of pressure indicates the advent of storms.
76. The high density of population in Nile Valley and Island of Java is primarily due
to
(a) intensive agriculture
(b) industrialization
(c) urbanization
(d) topographic constraints
Ans. (a)
Explanation :
Fertile alluvial soil beds in these region lead to intensive agriculture, due to
which the high density of population is seen in those areas.
77. Consider the following statements made about the sedimentary rocks :
1. Sedimentary rocks are formed at earth's surface by the hydrological system.
2. The formation of sedimentary rocks involves the weathering of pre-existing
rocks.
3. Sedimentary rocks contain fossils.
4. Sedimentary rocks typically occur in layers.
Which of these statements are correct ?
(a) 1 and 2
(b) 1 and 4
(c) 2, 3 and 4
(d) 1, 2, 3 and 4
Ans. (d)
Explanation:
All the statements are correct.
The weathered material of the pre-existing rocks is transformed by the
hydrolised systems and is deposited elsewhere in layers. They contain fossils.
78. Volcanic eruptions do not occur in the
(a) Baltic Sea
(b) Black Sea
(c) Caribbean Sea (d) Caspian
Sea
Ans. (a)
Explanation:
Volcanic eruptions do not occur in Baltic Sea.
79. Quartzite is metamorphosed from
(a) Limestone
(b) Obsidian
(c) Sandstone
(d) Shale
Ans. (c)
Explanation :
Quartzite is metamorphosed from Sandstone while Shale is from Slate.
80. Identify the correct order of the processes of soil-erosion from the following :

141

(a) Splash erosion, Sheet erosion, Rill erosion, Gully erosion


(b) Sheet erosion, Splash erosion, Gully erosion, Rill erosion
(c) Rill erosion, Gully erosion, Sheet erosion, Splash erosion
(d) Gully erosion, Rill erosion, Sheet erosion, Splash erosion
Ans. (a)
Explanation:
The correct order of soil erosion process is Splash ^Sheet-Rill - Gully and
Ravine erosion.
81. The temperature and rainfall of a meteorological station are given below :
Temperature (C)
Rainfall (cm)
J
9.4
12.2
F
10.6
9.1
M
11.7
7.9
A
12.2
2.5
M
13.3
1.8
J
13.9
0.3
J
1379

A
14.4

S
15.6
0.8
O
15.0
2.5
N
13.3
6.1
D
10.6
11.7
Average Temperature : 12.8 C, Average Rainfall : 54.9 cm per annum
Identify the region having the above climatic pattern from amongst the following
:
(a) Mediterranean region
(b) Monsoon region
(c) Steppe region
(d) N. W. European region
Ans. (a)
Explanation:
Concentration of rainfall in winter and dry. warm summers is the characteristic
of Mediterranean or western margin climate.
82. Match List I with List II and select the correct answer using the codes given
below the lists:
List I (Local Wind)
List II (Region)
A. Fohn
1. Argentina
B. Samun
2. Kurdistan
C. Santa Ana
3. California
D. Zonda
4. Alps
A B C D
A B C D
(a) 2 4 1 3
(b) 4 2 3 1
(c) 2 4 3 1
(d) 4 2 1 3
Ans. (b)
Explanation:
The local wind Fohn in Alps region, Samun in Kurdistan, Santa Ara in California
and Zonda in Argentina.
83. The given map shows four towns of Central Asian region marked as I, 2, 3 and
4. Identify these from the following list and select the correct answer using the
codes given below :
Towns :

142

84.

85.

86.

87.

88.

A. Bishkek
B. Ashkhabad
C. Tashkent
D. Dushanbe
A B C D
A B C D
(a) 3 1 2 4
(b) 3 1 4 2
(c) 1 3 2 4
(d) 1 3 4 2
Ans. (a)
Explanation :
In the map marked T is Ashikabad (now Ashgabat) capital of Turkmenistan, '2'
is Tashkent, capital of Uzbekistan, '3' is Bishkek, capital of Kyrgyzstan, 4 is(
Dushanbe, capital of Tajikstan.
Directions : The following items consist of two statements, one labelled as the
Assertion (A) and the other as Reason (R). You are to examine these two
statements carefully and select the answers to these items using the code
given below :
(a) Both A and R are true and R is the correct explanation of A
(b) Both A and R are true but R is NOT the correct explanation of A
(c) A is true but R is false
(d) A is false but R is true
Assertion (A) : Anticyclonic conditions are formed in winter season when
atmospheric pressure is high and air temperatures are low.
Reason (R) : Winter rainfall in Northern India causes development of
anticyclonic conditions with low temperatures
Ans. (b)
Explanation:
Both A and R are correct but R is not the explanation of A
Assertion (A) : During the Neap Tides, the high tide is lower and the low tide
is higher than usual.
Reason (R) : The Neap Tide, unlike the Spring Tide, occurs on the New Moon
instead of on the Full Moon.
Ans. (b)
Explanation:
The reason for lower high tide and higher low tide is - as it occurs on the new
moon. So the gravitational pull of the new moon is less than the full moon.
If the stars are seen to rise perpendicular to the horizon by an observer, he is
located on the
(a) Equator
(b) Tropic of Cancer (c) South Pole
(d) North Pole
Ans. (a)
2002
Consider the following statements :
1. In equatorial regions, the year is divided into four main seasons.
2. In Mediterranean region, summer receives more rain.
3. In China type climate, rainfall occurs throughout the year.
4. Tropical highlands exhibit vertical zonation of different climates.
Which of these statements are correct ?
(a) 1, 2, 3 and 4
(b) 1, 2 and 3
(c) 1, 2 and 4
(d) 3 and 4
Ans. (d)
Explanation:
Only 3, 4 are correct. Because - in equatorial regions there is no clear
distinction of main seasons. In Mediterranean area there is rainfall during the
winter but not in summer.
Consider the following ecosystems
1. Taiga
2. Tropical evergreen
3.Tropical decidous 4. Tundra

143

89.

90.

91.

92.

The correct sequence in decreasing order of the albedo values of these


ecosystems is
(a) 1, 4, 3, 2
(b) 4, 1, 2, 3
(c) 4, 1, 3, 2
(d) 1, 4, 2, 3
Ans. (c)
Explanation:
The earth's average albedo is 34%. But it varies according to the texture and
colour of the surface. For fresh snow, the albedo is 85%, for forests, it is 5-10%.
Accordingly, the order is Tundra - Taiga - Tropical evergreen - Tropical
deciduous.
Directions : The following items consist of two statements, one labelled as the
Assertion (A) and the other as Reason (R). You are to examine these two
statements carefully and select the answers to these items using the code
given below :
(a) Both A and R are true and R is the correct explanation of A
(b) Both A and R are true but R is NOT the correct explanation of A
(c) A is true but R is false
(d) A is false but R is true
Assertion (A) : 60 - 65 latitudes in both the hemispheres have a low
pressure belt instead of high pressure.
Reason (R) : The low pressure areas are permanent over oceans rather than
on land.
Ans. (b)
Explanation:
Both statements are correct. But R is not the correct reason for A. It is due to
the rotational movement of the earth that these low and high pressure belts are
formed.
Assertion (A) : The surface winds spiral inwards upon the centre of the
cyclone.
Reason (R) : Air descends in the centre of the cyclone.
Ans. (c)
Explanation:
'A' is correct but 'R' is wrong. The surface winds spiral inwards upon the centre
of the cyclone, in the eye of the cyclone air ascends. However, the central
portion of the eye has a small counter current where air descends.
Consider the following climatic and geographical ph enomena :
1. Condensation
2. High temperature and humidity
3. Orography
4. Vertical wind
Thunder Cloud development is due to which of these phenomena ?
(a) 1 and 2
(b) 2, 5 and 4
(c) 1, 3 and 4
(d) 1, 2, 3 and 4
Ans. (d)
Explanation:
The thunder cloud develops by all the above climatic and geographical
phenomena.
High temperature and humidity causes the wind to rise vertically up, due to
Orography or pressure of mountains.
These winds get condensated and form Cumulonimbus clouds or thunder
clouds.
For short-term climatic predictions, which one of the following events, detected
in the last decade, is associated with occasional weak monsoon rains in the
Indian sub-continent ?
(a) La Nina
(b) Movement of Jet Streams

144

(c) El Nino and Southern Oscillations


(d) Greenhouse effect on global level
Ans. (c)
Explanation:
El Nino and Southern oscillation are the reasons associted with occasional
weak monsoon rains in Indian sub-continent. El Nino is a warm oceanic current
of pacific ocean, near Peru coast of S. America.
93. In the map given below, four islands of Indian Ocean region i.e., (A) Seychelles,
(B) Chagos, (C) Mauritius and (D) Socotra are marked as 1, 2, 3 and 4. Match
them and select the correct answer from the codes given below :

94.

95.

96.

97.

A B C D
A B C D
(a) 1 3 4 2
(b) 3 1 2 4
(c) 1 3 2 4
(d) 3 1 4 2
Ans. (d)
Explanation :
The four islands of Indian ocean are - T is Chagos, '2' is socotra''(Yemen), '3' is
Seychelles and '4' is Mauritius.
Which one of the following statements is correct with reference to - our solar
system ?
(a) The Earth is the densest of all the planets in our solar system
(b) The predominant element in the composition of Earth is silicon
(c) The Sun contains 75 percent of the mass of the solar system
(d) The diameter of the Sun is 190 times that of the Earth
Ans. (a)
Explanation:
The predominant element in the earth's composition is 'Fe' (35%), while in the
earth's crust is Oxygen (46%). The Sun contains more than 99% of the mass of
the Solar System. The diameter of the Sun is 109 times that of the Earth.
Consider the following statements :
1. Ocean currents are the slow-surface movement of water in the ocean.
2. Ocean currents assist in maintaining the Earth's heat balance.
3. Ocean currents are set in motion primarily by prevailing winds.
4. Ocean currents are affected by the configuration of the ocean.
Which of these statements are correct ?
(a) 1 and 2
(b) 2, 3 and 4
(c) 1, 3 and 4
(d) 1, 2, 3 and 4
Arts. (b)
Explanation:
The ocean currents are not only slow but also could be fast. So, statements 2,
3 and 4 are correct.
Consider the following countries :
1. Brazil
2. Indonesia
3. Japan
4. Russia
What is the descending order of the size of the following countries populationwise ?
(a) 1, 2, 4, 3
(b) 2, 3, 1, 4
(c) 2, 1, 4, 3
(d) 1, 2, 3, 4
Ans. (c)
Explanation :
Indonesia (217 million) > Brazil (170 million) > Russia (146 million) > Japan
(127 million).
Match List I with List II and select the correct answer using the codes given
below the lists:

145

98.

99.

100.

101.

102.

List I (Ethnic Community)


List II (Country)
A. Apatani
1. China
B. Dayak
2. India
C. Dinka
3. Indonesia
D. Uighur
4. Sudan
A B C D
A B C D
(a) 2 3 4 1
(b) 3 2 4 1
(c) 2 3 1 4
(d) 3 2 1 4
Ans. (a)
Explanation :
Apatani is the ethnic community of India (Arunachai Pradesh). Dyak is the
ethnic community of Indonesia.
Dinka is the ethnic community of Sudan (Africa).
Uighur is the ethnic community of China.
Consider the- following countries :
1. Albania
2. Bosnia Herzegovina 3. Croatia
4. Macedonia
Which of these countries has/have Adriatic Sea as a boundary ?
(a) 1 and 2
(b) 1, 2 and 3
(c) 4 only
(d) 3 and 4
Ans. (b)
Explanation:
Macedonia is a land locked country, whereas remaining 3 countries are having
Adriatic sea as their boundary.
Consider the following countries of South Asia:
1. Bangladesh
2. India
3. Pakistan
4. Sri Lanka
The descending order of literacy status of these countries is :
(a) 4, 2, 1, 3
(b) 2, 4, 3, I
(c) 4, 2, 3, 1
(d) 2, 4, 1, 3
Ans. (c)
Explanation:
The literacy percentage of the given Asian countries are -Srilanka (90-7%) >
India (64-8%) > Pakistan (40-9%) > Bangladesh (38-9%).
A hollow sphere of radius R, a hollow cube of side R and a thin circular plate of
radius R, made up of the same material, are ail heated to 20 C above room
temperature. When left to cool in the room, which of them will reach the room
temperature first ?
(a) Circular plate
(b) Cube
(c) Sphere
(d) All of them will reach the room temperature at the same time
Ans. (c)
Explanation:
The sphere will reach room temperature first becauge it holds maximum area
that is exposed.
Berber speaking community, frequently in the news, lives in
(a) Afghanistan
(b) Algeria
(c) Argentina
(d) Australia
Arts. (b)
2003
Life expectancy is -highest in the world in
(a) Canada
(b) Germany
(c) Japan
(d) Norway
Ans. (c)
Explanation :
Highest life expectancy in the world is in Japan (80-9 years) next to it, Canada
(79-7 years), Norway (78-9 years) and Germany (77-8 years).

146

103. Which among the following countries lias the largest population ?
(a) Indonesia
(b) Japan
(c) Pakistan
(d) Sudan
Ans. (a)
Explanation:
In the given countries, Indonesia has the largest population (23,20,73,071) next
to it Pakistan (14,76,00,000), Japan (12,70,00,000), Sudan (3,71,00,000).
Directions : The following items consist of two statements, one labelled as the
Assertion (A) and the other as "Reason (R). You are to examine these two
statements carefully and select the answers to these items using the code
given below :
(a) Both A and R are true and R is the correct explanation of A
(b) Both A and R are true but R is NOT the correct explanation of A
(c) A is true but R is false
(d) A is false but R is true
104. Assertion (A) : The amount of moisture in the atmosphere is related to latitude.
Reason (R) : The capacity to hold moisture in the form of water vapour is
related to temperature.
Ans. (a)
Explanation:
The amount of moisture in the atmosphere is related to latitude. Because,
according to the change in latitudes, temperature also varies on the earth.
Temperature directly influences the moisture holding capacity of the
atmosphere.
105. Assertion (A) : Unlike temperate forests, the tropical rain forests, if cleared,
can yield productive farmland that can support intensive agriculture for several
years even without chemical fertilizers.
Reason (R) : The primary productivity of the tropical rain forest is very high
when compared to that of temperate forests.
Ans. (d)
Explanation:
A is wrong and R is correct.
In tropical rainforests because of the rainfall throughout the year, leaching of
minerals occur. Due to high temperatures bacterial activity is enormous, there
by humous cantent in the soil decreases. To raise crops in this region requires
application of chemical fertilizers.
106. Assertion (A) : Areas lying within five to eight degrees latitude on either side of
the equator receive rainfall throughout the year.
Reason (R) : High temperatures and high humidity cause convectional rain to
fall mostly in the afternoons near the equator.
Ans. (a)
Explanation:
Areas lying within 5-8 latitude on either side of the equator receives
convectional rainfall, mostly in the afternoons throughout the year, due to high
humidity and high temperatures.
107. Consider the following statements :
1. In Macedonia, ethnic Albanians are a minority.
2. In Kosovo, Serbians are a majority.
Which of these statements is/are correct ?
(a) Only 1
(b) Only 2
(c) Both 1 and 2
(d) Neither
1
nor 2
Ans. (c)

147

108.

109.

110.

111.

112.

113.

114.

Explanation:
Both are correct.
In Macedonia, ethnic Albanians are a minority.
In Kosovo, Serbians are a majority.
Israel has common borders with
(a) Lebanon, Syria, Jordan and Egypt
(b) Lebanon, Syria,
Turkey and
Jordan
(c) Cyprus, Turkey. Jordan and Egypt
(d) T urkey, Syria, Iraq and Yemen
Ans. (a)
Explanation:
Israel has common borders with Lebanon and Syria in the north and north-east,
Jordan in the east, and Egypt in the west.
Which one among the following covers the highest percentage of forest area In
the world ?
(a) Temperate Coniferous forests
(b) Temperate Deciduous forests
(c) Tropical Monsoon forests
(d) Tropical Rain forests
Ans. (a)
Explanation :
Temperate coniferous forests cover the highest percentge of forest area in the
world.
In which one of the following countries is Rupee its currency ?
(a) Bhutan
(b) Malaysia
(c) Maldives
(d) Seychelles
Ans. (d)
Explanation :
Seychelles currency is Rupee.
Bhutan's currency is Ngultrum.
Malaysia's currency is Ringgit.
Maldives currency is Rufiyaa.
Which one of the following countries is land locked ?
(a) Bolivia
(b) Peru
(c) Suriname
(d) Uruguay
Ans. (a)
Explanation:
Bolivia is Sand locked among Brazil, Peru, Chile, Argentina and Paraguay.
Which one of the following countries does NOT border the Caspian Sea ?
(a) Armenia
(b) Azerbaijan
(c) Kazakhstan
(d)
Turkmenistan
Ans. (a)
Explanation:
Armenia is land locked among Georgia, Azerbaijan, Iran and Turkey.
Among the following which planet takes maximum time for one revolution
around the Sun?
(a) Earth
(b) Jupiter
(c) Mars
(d) Venus
Ans. (b)
Explanation :
Jupiter takes 12 years for one revolution where as earth takes 1 year, Mars
takes 687 days and Venus takes 263 days.
Which one among, the following languages has largest number of speakers in
the world ?
(a) Bengali
(b) French
(c) Japanese
(d) Portuguese
Ans. (a)

148

115.

116.

117.

118.

119.

Explanation :
(In the given optionals) Bengali has largest number of speakers in the world i.e.
207 million, French - 79 million speakers, Japanese - 125 million speakers,
Portuguese ^ 176 million speakers.
2004
The great Asian river Mekong does not run through
(a) China
(b) Malaysia
(c) Cambodia
(d) Laos
Ans. (b)
Explanation:
Mekong river does not run through Malaysia but runs through China, Laos,
Cambodia and Vietnam.
Latvia does not share its borders with which one of the following countries?
(a) Russia
(b) Estonia
(c) Lithuania
(d) Poland
Ans. (d)
Explanation:
Latvia shares its borders with Estonia, Russia, Belarus and Lithuania.
Match List I with List SI and select the correct answer using the codes given
below the lists:
List I (Sea)
List II (Country)
A. Black Sea
1. Bulgaria
B. Red Sea
2. China
C. Yellow Sea
3. Eritrea
D. Caspian Sea
4. Kazakhstan
A B C D
A B C D
(a) 1 4 2 3
(b) 2 3 1 4
(c) 1 3 2 4
(d) 2 4 1 3
Ans. (c)
Explanation:
Black sea shares its borders with Bulgaria in the west.
Red sea shares its borders with Eritrea (Africa) in south west.
Yellow sea is located in the east of China, and Caspian sea'is located in the
south west region of Kazakhstan.
Which one of the following statements is correct ?
(a) Cirrus clouds are composed of ice crystals
(b) Cirrus clouds exhibit a flat base and have the appearance of rising domes
(c) Cumulus clouds are white and thin and form delicate patches and give a
fibrous and feathery appearance
(d) Cumulus clouds are classified as high clouds
Ans. (a)
Explanation :
Cirrus clouds look fibrous and appears like wisps in the blue sky, and
composed of ice crystals. Cumulus clouds are with a round top and horizontal
base, and these are classified as clouds with no definite height.
Which one of the following statements is not correct ?
(a) Gulfs with narrow fronts and wider rears experience high tides
(b) Tidal currents take place when a gulf is connected with the open sea by a
narrow channel
(c) Tidal bore occurs when a Lide enters the narrow and shallow estuary of a
river

149

120.

121.

122.

123.

(d) The tidal nature of the mouth of the river Hooghly is of crucial importance to
Kolkata as port
Ans. (a)
Explanation:
Gulfs with wide fronts and narrow rears experience high tides. Options (b), (c)
and (d) are correct.
Directions : The following items consist of two statements, one labelled as the
Assertion (A) and the other as Reason (R). You are to examine these two
statements carefully and select the answers to these items using the code
given below :
(a) Both A and R are true and R is the correct explanation of A
(b) Both A and R are true but R is NOT the correct explanation of A
(c) A is true but R is false
(d) A is false but R . is true
Assertion (A) : The thickness of the atmosphere is maximum over the
Equator.
Reason (R) : High insolation and strong convection currents occur over the
Equator.
Ans. (a)
Explanation:
High insolation and strong convection currents result in expansion of air, at the
equator. Thereby the thickness of the atmosphere is maximum over the
equator.
Consider the following geological phenomena :
1. Development of a fault
2. Movement along a fault
3. Impact produced by a volcanic eruption 4. Folding of rocks
Which of the above cause earthquakes ?
(a) 1, 2 and 3
(b) 2 and 4
(c) 1, 3 and 4
(d) 1, 2, 3 and 4
Ans. (d)
Explanation:
Faults, thrusts, folds are associated with earth's crust. Impact produced by a
volcanic eruption also causes earthquake.
Which one of the following does not border Panama ?
(a) Costa Rica
(b) Pacific Ocean
(c) Colombia
(d) Venezuela
Ans. (d)
Explanation:
Venezuela does not border Panama. Colombia on the eastern side, Costa Rica
on the western side, Pacific Ocean on the southern side, are having border with
Panama.
Match List I (State / Province / Overseas Territory) with List II (Country) and
select the correct answer using the codes given below the lists :
List 1
List II
A. British Colombia 1. USA
B. Bavaria
2. UK
C. Gibraltar
3. Canada
D. Rhode Island
4. Germany
5. Denmark
A B C D
A B C D
(a) 1 2 5 3
(b) 3 4 2 1
(c) 1 4 2 3
(d) 3 2 5 1
Ans. (b)

150

124.

125.

126.

127.

Explanation:
British Colombia belongs to Canada. Bavaria belongs to Germany. Gibraltar
belongs to U.K.
Rhode Island belongs to U.S.A.
Match List I (New Names of the Countries) with List II (Old names of the
Countries) and select the correct answer using the codes given below the lists :
List 1
List II
A. Benin
1. Nyasaland
B. Belize
2. Basutoland
C. Botswana
3. Bechuanaland
D. Malawi
4. British Honduras
5. Dohomey
A B C D
A B C D
(a) 3 1 2 4
(b) 5 4 3 1
(c) 3 4 2 1
(d) 5 1 3 4
Ans. (b)
Explanation:
Benin is the new name of Dohomey Belize is the new name of British Honduras
Botswana is the new name of Bechuanaland Malawi is the new name of
Nyasaland
Which one of the following countries does NOT border the Caspian Sea ?
(a) Armenia
(b) Azerbaijan
(c) Kazakhstan
(d)
Turkmenistan
Ans. (a)
Explanation:
The Caspian Sea is bordered by Russia, Khazakhstan, Turkmenistan,
Azerbaijan and Iran.
Consider the following statements :
1. Montenegro and Serbia agreed to a new structure for the Yugoslav
Federation.
2. Croatia remained under the Hungarian Administration until the end of First
World War.
3. Claims to Macedonia Territory have long been a source of contention
between Belgium and Greece.
4. In 1991, Slovenia declared independence from Czechoslovakia.
Which of the statements given above are correct ?
(a) 1 and 2
(b) 1, 2 and 3
(c) 2. 3 and 4
(d) 1, 3 and 4
Ans. (b)
Explanation:
In 1991, Slovenia declared independence from Yugoslavia and not from
Czechoslovakia.
Itaipu Dam built on the River Parana is one of the largest dams in the world.
Which one of the following two countries have this as a joint project ?
(a) Brazil and Peru
(b) Paraguay and Ecuador
(c) FSrazil and Paraguay
(d) Colombia and Paraguay
Ans. (c)
Explanation:
Itaipu Dam built on the River Parana is a joint project between Brazil and
Paraguay.

151

128. Which one of the following cities is not a former capital of the given country
(Country given in the brackets) ?
(a) Karachi (Pakistan)
(b) Auckland (New Zealand)
(c) Kyoto (Japan)
(d) Brisbane (Australia)
Ans. (d)
Explanation:
Karachi was the capital of newly independent Pakistan from 1947 until 1959. In
1959, Rawalpindi became the second capital, and in 1967 Islamabad became
permanent capital. Auckland was colonial capital of New Zealand in 1840 and
in 1865 the capital was transferred to Wellington.
Upto 1185, Kyoto was the capital of Japan. In 1185, it was transferred to Tokyo
(Kamakura).
Brisbane was the capital of Queensland. It never was the capita! of Australia.
129. Consider the following statements :
1. Total land area of Earth is approximately 1475 lakh square kilometres.
2. Ratio of land area to water area of Earth is approximately 1:4.
3. Maximum percentage of Earths water is in the Pacific Ocean.
Which of the statements given above is/are correct?
(a) 1 and 3
(b) 2 and 3
(c) 1 only
(d) 3 only
Ans. (a)
Explanation:
Ratio of land area to water area of earth is 29 : 71 (nearly 1 : 2-5).
Surface area of Earth : 510,100,500 sq. km.
Land area of Earth : 148,950,800 sq. km.
Ocean Surface of Earth : 361,149,700 sq. km.
130. Which one f the following pairs is not correctly matched ?
(a) Seikan Rail Tunnel :
China
(b) Petronas
Towers
: Malaysia
(c) Appalachian Trail : United States of America
(d) Rogun Dam
: Tajikistan
Asn. (a)
Explanation:
Petronas Towers - Malaysis Appalachian Trail - U.S.A.
Rogun Dam - Tajikistan
Seikan Rail Tunnel, 34 miies in length is the world's longest and most costly
tunnel linking all the islands of Japan by train.
131. Which one of the following Is the correct sequence of the given Continents in
the decreasing order of their percentage of Earths land ?
(a) North America - Africa - South America -Europe
(b) Africa - North America - South America - Europe
(c) North America - Africa - Europe - South America
(d) Africa - North America - Europe - South America
Ans. (b)
Explanation:
Continent
Area (sq. km.)
% area
Asia
44,579,000
29-5
Africa
30,065,000
20-0
North America
24,256,000
16-3
South America
17,819,000
11-8
Antarctica
13,209,000
9-6

152

Europe
9,938,000
6-5
Oceania
7,687,000
5-2
132. Match items in the List-I.with List-II and select the correct answer using the
codes given below the lists:
List-1 (Country)
List-II (Name of Parliament)
A. Netherlands
1. Diet
B. Ukraine
2. States General
C. Poland
3. Supreme Council
D. Japan
4. Sejm
A B C D
A B C D
(a) 4 1 2 3
(b) 2 3 4 1
(c) 4 3 2 1
(d) 2 1 4 3
Ans. (b)
Explanation :
Parliament of Netherlands is States General Parliament of Ukraine is Supreme
Council Parliament of Poland is Sejm Parliament of Japan is Diet
133. For which one of the following countries, is Spanish not an official language ?
(a) Chile
(b) Columbia
(c) Republic of Congo (d)
Cuba
Ans, (c)
Explanation :
French, Kiswahili, Lingala are the official languages in the
Republic of Congo.
134. Consider the following statements :
1. The axis of the earths magnetic field is inclined at 2314 to the geographic
axis of the earth.
2. The earths magnetic pole in the northern hemisphere is located on a
peninsula in northern Canada.
3.
Earths magnetic equator passes through Thumba in South India.
Which of the statements given above is/are correct?
(a) 1, 2 and 3
(b) 2 and 3
(c) 2 only
(d) 3 only
Ans. (b)
Explanation:
An imaginary line joining the magnetic poles would be inclined by
approximately 11-3 from the planet's axis of rotation. The axis of the earth is
Inclined to the plane of the ecliptic (the plane in which the earth orbits round the
sun) at an angle of 66 V20, giving rise to different seasons and varying lengths
of day and night. Earth rotates on its own axis, tilted at an angle of 23-4 every
23 hr. 56 giving rise to Day and Night.
135. Which one of the following pairs is not correctly matched ?
(a) Bahamas
: Nassau
(b) Costa Rica
: San Jose
(c) Nicaragua
: Belmopan
(d) Dominican Republic : Santo Domingo
Ans. (c)
Explanation:
Bahamas : Nassau, Costa Rica : San Jose, Nicaragua : Managua, Belize :
Belmopan, Dominican Republic : Santo Domingo
136. Which one of the following is the correct sequence of the given towns of
Pakistan while moving from the North towards the South ?

153

137.

138.

139.

140.

141.

(a) Islamabad - Gujranwala - Peshawar - Multan


(b) Peshawar - Gujranwala - Multan - Islamabad
(c) Peshawar - Islamabad - Gujranwala - Multan
(d) Islamabad Multan - Peshawar - Gujranwala
Ans. (c)
Explanation:
Sequence of towns of Pakistan moving from North to South: Peshawar Islamabad - Gujranwala - Multan.
Where are the Balearic Islands located ?
(a) Mediterranean Sea
(b) Black Sea
(c) Baltic Sea
(d) North Sea
Ans. (a)
Explanation:
Balearic Islands are located in Mediterranean Sea. These islands are divided
into Gimnesias (Malloria, Menoria and Cabrera) in the North and Pitiuass (Ibiza
Formentera) in the South west.
Which one of the following countries does not border Lithuania ?
(a) Poland
(b) Ukraine
(c) Belarus
(d) Latvia
Ans. (b)
Explanation :
Lithuania is not bordered by Ukraine.
Consider the following statements :
1. Great Britain comprises England, Wales, Scotland and Northern Ireland.
2. England covers less than 60% of the total area of the United Kingdom.
Which of the statements given above is/are correct?
(a) 1 only
(b) 2 only
(c) Both 1 and 2
(d) Neither 1
nor 2
Ans. (b)
Explanation:
Great Britain comprises England, Wales and Scotland on the island of Great
Britain, while the United Kingdom includes Great Britain as well as Northern
Ireland on the Island of Ireland. The Isle of Man in the Irish sea and the channel
islands in the English channel, are dependencies of the crown, with their own
systems of government.
Where is the volcanic mountain, Mount St. Helens located ?
(a) Chile
(b) Japan
(c) Philippines
(d) United States of America
Ans. (d)
Explanation:
Mount St. Helens is located in United States of America.
Directions : The following items consist of two statements, one labelled as the
'Assertion (A) and the other as Reason (R). You are to examine these two
statements carefully and select the answers to these items using the code
given below :
(a) Both A and R are true and R is the correct explanation of A
(b) Both A and R are true but R is NOT the correct explanation of A
(c) A is true but R is false
(d) A is false but R is true
Assertion (A) : The same face of the Moon is always presented to the Earth.
Reason (R) : The Moon rotates about its own axis in 2314 days which is about
the same \time that it takes to orbit the Earth.

154

142.

143.

144.

145.

146.

Ans. (c)
Explanation:
Moon rotates more slowly than the Earth and takes little over 27 days to rotate
once. Since it takes about the same time to revolve around the earth, it always
presents the same face or hemisphere to the Earth. Hence R is not correct
statement.
Assertion (A) : Existence of human life on Venus is highly improbable.
Reason (R) : Venus has extremely high level of carbon dioxide in its
atmosphere.
Ans. (a)
Explanation:
Venus second ciosest planet after Mercury to the Sun is often considered
'Earth's twin' because of their close proximity in size, mass (weight) and
density. Existence of human life is highly improbable because of high levels
C02 in its atmosphere.
Which one of the following cities is the seat of Parliament of Sooth Africa ?
(a) Pretoria
(b) Durban
(c) Johannesburg
(d) Cape Town
Ans. (d)
Explanation:
Legislative Capital (Seat of Parliament) - Cape town Administrative Capital Pretoria Judiciary Capital - Johannesburg
Match List-I with List-II and select the correct answer using the code given
below the lists :
List - I (City)
List - II (River)
A. Washington D.C. 1. River Manzanares
B. Berlin
2. River Seine
C. Paris
3. River Spree
D. Madrid
4. River Potomac
A B C D
A B C D
(a) 2 3 4 1
(b) 4 1 2 3
(c) 2 1 4 3
(d) 4 3 2 1
Ans. (d)
Explanation :
Washington D.C. - River Potomac Berlin - River Spree Paris - River Seine
Madrid - River Manzanares
Which one of the following pairs is not correctly matched ?
Current Name
Old Name
(a) Harare
: Salisbury
(b) Ethiopia
: Abyssinia
(c ) Ghana
: Dutch Guiana
(d) Kinshasa
: Leopoldville
Ans. (c)
Explanation:
Harare - Salisbury Suriname - Dutch Guiana Ethiopia - Abyssinia Ghana - Gold
coast Kinshasa - Leopoldville
2006
What is the new administrative capital proposed for Myanmar ?
(a) Bassein
(b) Mandalay
(c) Myitkyina
(d) Pyinmana
Ans. (d)
Explanation:

155

Pyinmana is the new administration capital with strategic importance proposed


for Myanmar. Present capital is 'Yangon'.
Directions : The following items consist of two statements, one labelled as the
Assertion
(A) and the other as Reason (R). You are to examine
these two statements carefully and select the answers to these items using the
code given below :
(a) Both A and R are true and R is the correct explanation of A
(b) Both A and R are true but R is NOT the correct explanation of A
(c) A is true but R is false
(d) A is false but R is true
147. Assertion (A) : To orbit around Sun, the planet Mars takes lesser time than
time taken by the Earth.
Reason (R) : The diameter of the planet Mars is less than that of the Earth.
Ans. (d)
Explanation :
To orbit around the Sun, the planet Mars (Red planet) takes more time (687
days) than time taken by Earth (365 1A days). The diameter of the planet Mars
(6,787 km) is less than that of the Earth (12,756 km)
148. In which one of the following oceans is Biamantina Trench situated ?
(a) Pacific Ocean
(b) Atlantic Ocean (c) Indian Ocean
(d)
Arctic
Ocean
Ans. (c)
Explanation :
Diamantina Trench is situated in the South east Indian Ocean basin. It is the
deepest trench of the Indian Ocean (8,047 m).
149. Other than India and China, which of the following groups of countries border
Myanmar?
(a) Bangladesh, Thailand and Vietnam
(b) Cambodia, Laos and Malaysia
(c) Thailand, Vietnam and Malaysia
(d) Thailand, Laos and Bangladesh
Ans. (d)
Explanation :
Myanmar is bordered by Thailand, Laos, Bangladesh, China and India.
150. Which one of the following pairs is not correctly matched ?
Well-known Place
Country
(a) Baikonour
: Russia
(b) Kourou
: French Guiana
(c) Borobudur
: Indonesia
(d) Cannes
: France
Ans. (a)
Explanation:
Baikonour (Leninsk) - Kazakhsthan Kourou - French Guiana Borobudur Indonesia Cannes - France
151. Through which one of tSie following groups of countries does the Equator pass
?
(a) Brazil, Zambia and Malaysia
(b) Colombia, Kenya and Indonesia
(c) Brazil, Sudan and Malaysia
(d) Venezuela,
Ethiopia
and
Indonesia
Ans. (b)
Explanation:

156

152.

153.

154.

155.

156.

157.

Equator passes through Columbia,, Kenya and Indonesia. The other countries
through which the equator passes through includes Gabon, Congo, Democratic
Republic of Congo, Uganda, Somalia, Equador and Brazil.
What is the approximate mean velocity with which the Earth moves round the
Sun in its orbit ?
(a) 20 km/s
(b) 30 km/s
(c) 40 km/s
(d) 50 km/s
Ans. (b)
Explanation:
The approximate mean velocity with which the Earth moves round the Sun in its
orbit is 30 km/s. (29.8 km/s).
Which one of the following countries is not a member of the Commonwealth of
Independent States (CIS) ?
(a) Armenia
(b) Belarus
(c) Estonia
(d) Georgia
Ans. (c)
Explanation:
Commonwealth of Independent States (CIS), (1991), Minsk (Belarus). CIS is
made up of 12 of the 15 former Soviet Constituent Republics. Members of the
CIS in 1995 includes Armenia, Azerbaijan, Belarus, Georgia, Kazakhstan,
Kyrgyzstan, Moldova, Russia, Tajikistan, Turkmenistan, Ukraine and
Uzbekistan.
Which one of the following pairs is not correctly matched ?
(a) Slovenia
: Bratislava
(b) Seychelles
: Victoria
(c) Sierra Leone
: Freetown
(d) Uzbekistan
: Tashkent
Ans, (a)
Explanation :
Slovenia - Ljubljana, Seychelles - Victoria, Sierra Leone - Freetown, Uzbekistan
- Tashkent, Bratislava is the capital city of Slovakia.
Consider the following statements :
1. Length of a terrestrial mile is lesser than that of a nautical mile.
2. Harmattan is a dusty land-wind of the East African Coast.
3. Greece and Albania form a part of the Iberian Peninsula.
Which of the statements given above is/are correct?
(a) 1, 2 and 3
(b) 2 and 3, only
(c) 3 only
(d) 1 only
Ans. (d)
Explanation:
Harmattan is a dusty land-wind of the West African coast. The length of
terrestrial mile (1-6 km) is lesser than that of a nautical mile (1-85 km). Greece
and Albania forms a part of Balkan peninsula, Portugal and Spain forms Iberian
peninsula.
Where is Davos - the venue of the annual meeting of World Economic Forum,
located ?
(a) France
(b) Germany
(c) Switzerland
(d) Luxembourg
Ans. (c)
Explanation:
Davos, the venue of the annua! meeting of World Economic Forum is located in
Switzerland.
Consider the following statements :

157

158.

159.

160.

161.

1. The Richter scale is a logarithmic scale and so an increase of 1 magnitude


unit represents a factor of 10 times in amplitude.
2. Each integer reading of the Richter scale has an energy 100 times that of
the previous integer reading.
Which of the statements given above is/are correct?
(a) 1 only
(b) 2 only
(c) Both I and 2
(d) Neither
1
nor 2
Ans. (a)
Explanation:
Richter scale is a logarithmic scale that runs from 1 to 9, though no upper limit
exists. An increase of 1 magnitude unit represents a factor of 10 times in
amplitude i.e. a magnitude of 8 quake is 10 times more powerful than
magnitude of 7 quakes. Every increase of one number in magnitude means the
energy release of the quake is 32 times greater. For example, an earthquake of
magnitude 6-0 releases 32 times as much energy as an earthquake measuring
5-0.
Claims to the historical Macedonian territory have been a bone of contention
between which of the following countries ?
(a) Portugal and Spain
(b) Bulgaria and Greece
(c) Romania and Bulgaria
(d) Portugal and Greece
Ans. (b)
Explanation:
Claims to the historical Macedonian territory have been a bone of contention
between Bulgaria and Greece. The old European civilization flourished in
Macedonia between 7000 and 3500 B.C. The existence of Macedonian nation
was permanently distributing the neighbouring countries viz., Greece, Bulgaria,
Serbia and Albania.
Huangpu River flows through which one of the following cities ?
(a) Beijing
(b) Ho Chi Minh City (c) Shanghai
(d) Manila
Ans. (c)
Explanation:
Huangpu river flows through Shanghai city, is the most important Shipping
artery from the mouth of the Yangtzekiang river in Wusong to the East China
sea. Huangpu river is a tributary of Yangtzekiang river (3 rd longest river in the
world after Nile and Amazon).
Which one of the following countries is the leading producer of uranium ?
(a) United States of America
(b) Canada
(c) Germany
(d) Zambia
Ans. (b)
Explanation:
Canada is the leading producer of Uranium in the world mainly from the
deposits situated in Saskatchewan. Australia has the largest reserve of
Uranium followed by S. Africa and Nigeria.
Bermuda triangle extends up to which of the following places ?
1. Southern Florida 2. Puerto Rico
3. Hawaii islands
Select the correct answer using the code given below :
(a) 1, 2 and 3
(b) 1 and 2, only
(c) 2 and 3, only
(d) 1 and 3,
only
Ans. (b)
Explanation:

158

162.

163.

164.

165,

Bermuda triangle extends upto southern Florida San Juan and Puerto Rico in
the Atlantic ocean. Hawaii islands are present in Pacific ocean. This triangle is
noted for a high incidence of unexplained losses of ships, small boats and
aircraft.
2007
Match List-I with List-II and select tlie correct answer using the code given
below the lists :
LIst-I (City)
List-II (River)
A. Bangkok
1. Irrawaddy
B. Phnom-Penh
2. Mekong
C. Hanoi
3. Menam (Chao Phraya)
D. Yangon
4. Red River
A B C D
A B C D
(a) 3 2 4 1
(b) 4 1 3 2
(c) 3 1 4 2
(d) 4 2 3 1
Ans. (a)
Explanation :
Bangkok is the capita! of Thailand. It is found at the banks of Chao Phraya
river near the Gulf of Thailand, a Phnom-Penh is located is south-central region
of Cambodia, at the confluence of Mekong river.
Hanoi is the capital of Vietnam. It is located on the right bank of the Red river,
a Yangon is the largest city and former capita! of Myanmar. It is located at the
convergence of the Yangon and Bago rivers.
Consider the following statements ;
1. Either of the two belts over the oceans at about 30 to 35 N and S Latitudes
is known as Horse Latitude.
2. Horse latitudes are low pressure belts.
Which of the statements given above is/are correct ?
(a) 1 only
(b) 2 only
(c) Both 1 and 2
(d) Neither
1
nor 2
Ans. (a)
Explanation:
'Horse latitudes' are sub tropica! high pressure belts extends between 30-35
(25-35) latitudes in both the hemispheres.
Which one among the following rivers is the longest ?
(a) Amazon
(b) Amur
(c) Congo
(d) Lena
Ans. (a)
Explanation:
Amazon - 6570 km (longest river of the world)
Amur - 2825 km (A river in N-E Asia)
Congo - 4667 km (Second largest river of Africa)
Lena - 4410 km (A river in Siberia)
Source : New Vishal's Stimulating Geography - Page 44-48
Through which one of the following Straits, does a tunnel connect the United
Kingdom and France ?
(a) Davis Strait
(b) Denmark Strait (c) Strait of Dover
(d) Strait of
Gibraltar
Ans. (c)
Explanation:

159

166.
1.
2

167.

168.

169.

170.

Dover Strait is between English Channel and North Sea. Dover is the strait at
the narrowest part of the English Channel. The shortest distance across the
strait is from the South Foreland, some 6 km north-east of Dover in the country
of Kent, England. The strait lies at the east end of the English Channel, where it
joins the North Sea. Its width is the shortest distance between France and
England. Source : New Vishai's India at a Glance 2007 - Page 172 Stimulating
Geography - Page 61
Consider the following statements :
The annual range of temperature is greater in the Pacific Ocean than that in the
Atlantic Ocean.
The annual range of temperature is greater in the Northern Hemisphere than
that in the Southern Hemisphere.
Which of the statements given above is/are correct ?
(a) 1 only
(b) 2 only
(c) Both 1 and 2
(d) Neither
1
nor 2
Ans. (b)
Explanation:
Bigger the size of ocean (pacific), lesser the annual range of temperature.
The largest coral reef in the world is found near the coast of which one of the
following countries ?
(a) Australia
(b) Cuba
(c) Ghana
(d) Philippines
Ans. (a)
Explanation :
President Bush designated an Island chain spanning nearby 2250 km of the
Pacific Ocean as a national monument. The region contains 70% of the tropical
shallow water coral reefs in U.S. The reserve is the world's largest protect
marine area.
Source : New Vishai's Current Affairs 2007 - Page 262
Where is Copacabana Beach located ?
(a) Buenos Aires
(b) Hawaiian Islands (c) Rio de Janeiro
(d) Valletta
Ans. (c)
Explanation :
Copacabana Beach is located right in the heart of Rio de Janeiro, it is also one
of the most lively beaches of the world. It is famous for its 4 km beach. It piays
host to millions of revellers during the annual New Year's Eve celebrations.
Which one of the following countries is planning to construct a rival to the
Panama Canal to link the Pacific and Atlantic Oceans ?
(a) Colombia
(b) Costa Rica
(c) Guatemala
(d) Nicaragua
Ans, (d)
Explanation:
Nicaragua is likely to pursue an ambitious $20 b project to create an alternative
to Panama canal. This project would redraw the map of the world trade by
opening an east coast of North America, Europe, Brazil to large scale sea traffic
from pacific Rim countries like China and South Korea.
What is the average distance (approximate) between the Sun and the Earth ?
(a) 70 x io5 km
(b) 100 x 105 km
(c) 110 x 106 km
(d) 150 x 106
km
Ans. (d)
Explanation:

160

One Astronomical; Unit (AU) is the approximately the mean distance between
the Earth and the Sun, or, 149,598,000 km.
171. Which one of the following cities does not have the same clock time as that of
the other three cities at any given instant ?
(a) London (U.K.)
(b) Lisbon (Portugal)
(c) Accra (Ghana)
(d) Addis Ababa (Ethiopia)
Ans. (d)
Explanation:
London, Lisbon and Accra cities are situated approximately | on the same
longitude away from Addis Ababa showing | the same clock time.
172. Assertion (A) : There are no tea plantations in any African country.
Reason (R) : Tea plants need fertile soil with high humus.
Ans. (d)
Explanation:
Major Tea producing countries in Africa are Kenya, Tanzania, Malawi,
Zimbabwe and Mozambique.

161

You might also like